Вы находитесь на странице: 1из 130

Solutions to

Mathematics
PULLOUT WORKSHEETS
FOR CLASS X
First Term

By
Surender Verma
M.Sc. (Mathematics), B.Ed.
Delhi Public School,
Dwarka, New Delhi

Since 1950
SARASWATI HOUSE PVT. LTD.
(An ISO 9001:2008 Company)
EDUCATIONAL PUBLISHERS
New Delhi-110002
Published by:
Atul Gupta
Saraswati House Pvt. Ltd.
9, Daryaganj, Near Telephone Office, New Delhi-110002
Post Box: 7063
Phone: 43556600 (100 lines), 23281022
Fax: 43556688
E-mail: delhi@saraswatihouse.com
Website: www.saraswatihouse.com
Import-Export Licence No. 0507052021

Branches:
1. 48, V Main Road, Chamrajpet, Bengaluru-560018
Phone: (080) 26619880, 26672813
Fax: 26619880
2. SCO 31, Sector 31-D, Chandigarh-160030
Phone: (0172) 2624882
Fax: 5086882
3. 10/34, Mahalakshmi Street, T. Nagar, Chennai-600017
Phone: (044) 24343740, 24346531, 24333508
Fax: 24333508
4. Vinayak Royal, 80, Mahadev Nagar, Vaishali Nagar, Jaipur-302019
Phone: 9672987282
5. 39/741, Sudarshanam, Karikkamuri Cross Road, Ernakulam South,
Kochi-682011
Phone: (0484) 3925288, 3062576
6. 16 A, Jopling Road, Hazratganj, Lucknow-226001
Phone: (0522) 4062517
7. 001, Vastu Siddhi, Wing-A, Vastu Enclave, R.J. Road, Pump House,
Andheri (East), Mumbai-400093
Phone: (022) 28343022
8. 4, Sitayan Apartments, Vivekanand Marg, North S.K. Puri, Patna-800013
Phone: (0612) 2570403

New Edition

© Author
All rights reserved. No part of this publication may be reproduced, stored in a retrieval
system, or transmitted, in any form or by any means without the prior written permission
of the publishers.
CONTENTS

1. Real Numbers ............................................................................................... 7-15

> Worksheets (1 to 6) >Assessment Sheets (1 and 2)>Chapter Test

2. Polynomials ................................................................................................ 16-28

> Worksheets (10 to 15) >Assessment Sheets (3 and 4)>Chapter Test

3. Pair of Linear Equations in Two Variables ...................................... 29-50

> Worksheets (18 to 29) >Assessment Sheets (5 and 6) >Chapter Test

4. Triangles ...................................................................................................... 51-72

> Worksheets (33 to 45) >Assessment Sheets (7 and 8)>Chapter Test

5. Introduction to Trigonometry ............................................................... 73-87

> Worksheets (50 to 58) >Assessment Sheets (9 and 10)>Chapter Test

6. Statistics ...................................................................................................... 88-97

> Worksheets (62 to 65) >Assessment Sheets (11 and 12)>Chapter Test

PRACTICE PAPERS (1 to 5) ......................................................................... 99-132

–3–
Solutions to
PULLOUT WORKSHEETS
[Summative Assessments]
[FIRST TERM]
Chapter

1 REAL NUMBERS

WORKSHEET–1 As RHS of this equation is rational, but


LHS is an irrational so a contradiction.
125 53 1 10. Let a be any odd positive integer and b
1. (B) 4. 3
= = = 0.0625
2 5 16 × 5 3 16 = 4. By Euclid’s lemma there exist integers
125 q and r such that
Clearly, the decimal form of 4 3 termi-
2 .5 a = 4q + r, 0 ≤ r < 4
nates after four places. .
.. a = 4q or 4q + 1 or 4q + 2 or 4q + 3.
2. (C) We know that the factors of a prime Therefore, for a to be odd, we have to take
are 1 and the prime itself only.
a = 4q +1 or 4q +3.
Therefore, the common factor of p and q
will be 1 only. Hence, HCF (p, q) = 1. 11. The maximum capacity of a bag will be
the HCF of 490, 588 and 882. Let us find
3. (A) As prime factors of 1005 are:
out the required HCF by prime factorisa-
1005 = 5 × 3 × 67.
tion method.
∴ 7 is not a prime factor of 1005.
490 = 2 × 5 × 72
24 192 588 = 22 × 3 × 72
4. Hint: = = 0.192 .
125 1000 882 = 2 × 32 × 72
First number × Second number ∴ HCF = 2 × 72 = 98
5. LCM =
HCF Thus, the maximum capacity of a bag is
96 × 404 98 kg.
= = 24 × 404 = 9696.
4
6. (i) 660; (ii) 330 WORKSHEET–2
Hint: Going in opposite direction to the
1. (A) HCF (p, q) = 1 ⇒ p and q are coprime.
factor tree, we obtain
2 × 165 = 330 (ii) and 2 × 330 = 660 (i). p
If p and q are coprime with q ≠ 0 and is
7. HCF = 3; LCM = 420 q
Hint: 12 = 22 × 3; 15 = 3 × 5; 21 = 3 × 7. a rational number, then q has only 2 and 5
as prime factors.
8. (i) Terminating
i.e., q = 2m × 5n where, m and n are non-
543 543 negative integers.
Hint: = 1 3.
250 2 × 5
2. (B) Going to opposite direction to the
(ii) Non-terminating repeating.
factor tree, we obtain
9 1 1
Hint: = = . 3 × 7 = 21 (ii) and 2 × 21 = 42 (i).
108 12 22 × 31
23×1449
a 3. (A) Required number =
9. Hint: Let 5 – 2 3 = ;b≠0 161
b
5b – a 1449
⇒ 3 = = = 207.
2b 7

R E A L N U M B E R S 7
4. 2 = 1.414... and 3 = 1.732... 11. Length = 6 m 30 cm = 630 cm
3 Breadth = 5 m 85 cm = 585 cm
Therefore, we can take 1.5 = Height = 3 m 60 cm = 360 cm
2
3 The required length of the tape will be the
as 2 < < 3.
2 Highest Common Factor (HCF) of the
5. Hint: As 12576 > 4052 numbers 630, 585 and 360.
Let us find out the HCF.
.. . 12576 = 4052 × 3 + 420
Further 4052 = 420 × 9 + 272 630 = 2 × 32 × 5 × 7
Further 420 = 272 × 1 + 148 585 = 32 × 5 × 13
Further 272 = 148 × 1 + 124 360 = 23 × 32 × 5
Further 148 = 124 × 1 + 24 ∴ HCF = 32 × 5 = 45
Hence, the length of the tape will be 45 cm.
Further 124 = 24 × 5 + 4
Further 24 = 4 × 6 + 0.
WORKSHEET–3
In the last equation, remainder is zero.
Hence, the required HCF = 4. 43 43 × 5 215
1. (C) 4 3
= 4
= 4 = 0.0215
6. First given number is composite as 2 ×5 (2 × 5) 10
5 × 3 × 11 + 11 = 11 (15 + 1) = 11 × 16 Hence, the number terminates after four
= 11 × 2 × 8 places of decimal.

( )( ) ( 2) −( 3)
2 2
But second given number is prime as 2. (A) 2− 3 2+ 3 =
5 × 7 + 7 × 3 + 3 = 35 + 21 + 3 = 59.
= 2 – 3 = – 1.
7. No. Prime factors of 6n will be of type 2n × 3n.
– 1 is a rational number.
As it doesn't have 5 as a prime factor, so 6n
can't end with the digit 5. 3. (C) 128 = 27; 240 = 24 × 3 × 5.
8. Hint: Let a be any positive integer Now, HCF (128, 240) = 24 = 16.
.. . a = 3q or 3q + 1 or 3q + 2 HCF × LCM
4. Hint: First number = = 232.
.
.. a2 = 9q2 = 3m; m = 3q2 Second number
or a2 = (3q +1)2 = 3m + 1, m = q (3q + 2) 5. No.
Hint: Prime factors of 15 n will not be of
or a2 = (3q + 2)2 = 3m + 1, m = 3q2 + 4q + 1.
type 2n × 5n.
9. We represent 6, 72 and 120 in their prime
factors. 6. Rational number = 0.27
6 =2 × 3 Irrational number = 0.26010010001... .
72 = 23 × 32 145 29 8 232
7. (i) = × = = 0.232 .
120 = 23 × 3 × 5 625 125 8 1000
Now, HCF = 2 × 3 = 6 7 125 875
And LCM = 23 × 32 × 5 = 360. (ii) × = = 0.0875 .
80 125 10000
10. Hint: Let 2 − 5 = x, a rational number 8. Let us assume, to the contrary that 2 is
⇒ 2 =x+ 5 rational. We can take integers a and b ≠ 0
Squaring both sides, we get such that
a
2 = x2 + 5 + 2x 5 3 = , where a and b are coprime.
b
– x2 – 3 ⇒ 3b2 = a2
⇒ 5 =
2x ⇒ a2 is divisible by 3
RHS of this last equation is rational, but ⇒ a is divisible by 3 ...(i)
LHS is an irrational which is a contradiction. We can write a = 3c for some integer c

8 M A T H E M A T I C S – X
⇒ a2 = 9c2 The remainder has now become zero, so
∴ our procedure stops. Since the divisor at
⇒ 3b2 = 9c2 [ a2 = 3b2]
⇒ b2 = 3c2 this stage is 8, the HCF of 616 and 32 is 8
⇒ 2
b is divisible by 3 Hence, the maximum number of columns
⇒ b is divisible by 3 ...(ii) is 8.
From (i) and (ii) we observe that a and b
have atleast 3 as a common factor. But this WORKSHEET– 4
contradicts the fact that a and b are co-
( )( ) ( 6) − ( 5)
2 2
prime. This means that our assumption is 1. (B) 6− 5 6+ 5 =
not correct. = 6 – 5 = 1 = Rational number.
Hence, 3 is an irrational number. 2. (B)
Hint: Denominator is not in the exact form
9. As: 1032 = 408 × 2 + 216 ...(i)
of 2m × 5n, where m, n are non-negative
408 = 216 × 1 + 192 ...(ii) integers.
216 = 192 × 1 + 24 ...(iii)
3. (C) 0 ≤ r < b.
192 = 24 × 8 + 0 ...(iv)
4. Hint: 107 = 4 × 26 + 3.
⇒ HCF = 24
5. Hint: 7 × 13 = (ii) and (ii) × 11 = (i).
∴ From (iii)
6. Let us represent each of the numbers 30,
⇒ 24 = 216 – 192
72 and 432 as a product of primes.
= 216 – [408 – 216] {... Use (ii)}
30 = 2 × 3 × 5
= 2 × 216 – 408
72 = 23 × 32
= 2[1032 – 2 × 408] – 408
432 = 24 × 33
{... Use (i)}
Now, HCF = 2 × 3 = 6
24 = 1032 × 2 – 5 × 408
⇒ m = 2. And LCM = 24 × 33 × 5 = 2160.
7. Here, 396 > 82.
10. Hint: Let x be any positive integer.
∴ 396 = 82 × 4 + 68
Then it is of the form 3q or 3q + 1 or 3q + 2. Further 82 = 68 × 1 + 14
If x = 3q, then Further 68 = 14 × 4 + 12
x3 = (3q)3 = 9m; m = 3q3 Further 14 = 12 × 1 + 2
If x = 3q + 1, then Further 12 = 2 × 6 + 0
x3 = (3q + 1)3 In the last equation, the remainder is zero
= 9m + 1; m = q(3q2 + 3q + 1). and the divisor is 2.
Hence, the required HCF = 2.
If x = 3q + 2, then a
x3 = (3q + 2)3 8. Hint: Let 3 + 2 5 = ; b ≠ 0
b
= 9m + 8; m = q (3q2 + 6q + 4).
a – 3b
11. The maximum number of columns must ⇒ = 5 = Rational
2b
be the highest common factor (HCF) of Which is a contradiction as 5 is an irra-
616 and 32. Let us find out the HCF by the tional number.
method of Euclid's division lemma.
Since 616 > 32, we apply division lemma Hence, 3 + 2 5 is an irrational number.
to 616 and 32, to get 9. (i) The given fraction can be written as
616 = 32 × 19 + 8 43 43 × 5
= = 0.0215
Since the remainder 8 ≠ 0, we apply the 24 ·53 104
division lemma to 32 and 8, to get Hence, the given number terminates after
32 = 8 × 4 + 0 four places of decimal.

R E A L N U M B E R S 9
(ii) The given fraction can be written as 5 = 1 × 5; 6 = 1 × 2 × 3; 7 = 1 × 7;
359 4
2 × 359 5744 8 = 1 × 23; 9 = 1 × 32; 10 = 1 × 2 × 5
= 5 =
2×5 5
2 × 55 100000 Now,
= 0.05744 LCM = 1 × 23 × 32 × 5 × 7
Hence, the given number terminates after = 8 × 9 × 5 × 7 = 2520 is required
five places of decimal. number.
10. The required number of students will be
the highest common factor (HCF) of 312, 5− 3
5. Hint: = 2x – 15
260 and 156. Let us find out the HCF by 5+ 3
the method of prime factorisation.
312 = 23 × 3 × 13 ⇒ 4 – 15 = 2x – 15
260 = 22 × 5 × 13 ⇒ x = 2, which is a rational number.
156 = 22 × 3 × 13 6. Hint: Any odd positive integer will be
∴ HCF = 22 × 13 = 52 type of 4q + 1 or 4q + 3
Number of buses required ∴ (4q + 1)2 = 16q2 + 8q + 1
Total number of students = 8 (2q2 + q) + 1
=
Number of students in one bus
= 8n + 1
312 + 260 + 156 Also, (4q + 3)2 = 16q2 + 24q + 9
= = 14
52 = 8 (2q2 + 3q + 1) + 1
Thus, the maximum number of students in
a bus and number of buses required are = 8n + 1.
52 and 14 respectively. 7. 35 cm
Hint: Find HCF.
11. Hint: Let x = any positive integer
a
x = 5m, 5m + 1, 5m + 2, 5m + 3 or 5m + 4 8. Hint: Let 5−3 2 =
b
Now take square of all form.
where a, b are integers and b ≠ 0
Squaring on both sides,
WORKSHEET–5
a2
1. (C) 5 + 18 − 6 10 =
b2
Hint: LCM of 18, 24, 30, 42 = 2520
∴ Required number = 2520 + 1 = 2521. a2
⇒ 23 − = 6 10
2. (C) Let the quotient is m when n2 – 1 is b2
divided by 8. 23b 2 − a 2
⇒ = 10 ... a contradiction.
∴ n2 – 1 = 8 × m 6b 2
⇒ n2 – 1 = An even integer. 9. (i) Terminating. (ii) Terminating.
⇒ n2 = An even integer + 1 = Odd 10. The required number of burfis will be the
integer highest common factor of 420 and 130.
∴ n = An odd integer. Let us find out the HCF using Euclid's
3. (B) division lemma.
It is clear that 420 > 130. We apply Division
Hint: HCF (65, 117) = 13
lemma to 420 and 130, to get
Now, 65m – 117 = 13. 420 = 130 × 3 + 30
∴ m = 2 will satisfy this equation. Since the remainder 30 ≠ 0, so we apply
4. Prime factors of numbers 1 to 10 are: Division lemma to 130 and 30, to get
1 = 1; 2 = 1 × 2; 3 = 1 × 3; 4 = 1 × 22 130 = 30 × 4 + 10

10 M A T H E M A T I C S – X
Again the remainder 10 ≠ 0, so we apply 6. Irrational
Division lemma to 30 and 10, to get
2− 3 x
30 = 10 × 3 + 0 Hint: =
Now, the remainder is zero. So the HCF of 2+ 3 3
420 and 130 is the divisor at the last stage x
⇒ 7–4 3 =
that is 10. 3
Hence, the required number of burfis is 10. ⇒ 7 3 – 12 = x = Irrational.
11. Let n = 3q, 3q + 1 or 3q + 2. 7. Rational Number = 0.55
Case I: If n = 3q, then Irrational number = 0.5477477747... .
n = 3q divisible by 3 8. 15
n + 2 = 3q + 2 ⇒ Not divisible by 3 Hint: HCF (1380, 1455, 1620) = 15.
n + 4 = 3q + 4 = 3(q + 1) + 1 9. (i) 0.052. (ii) 5.8352.
⇒ Not divisible by 3. 10. We know that any positive integer is either
Case II: If n = 3q + 1 then only of the form 3q, 3q + 1 or 3q + 2 for some
integer q.
n + 2 = 3q + 1 + 2 = 3q + 3
Now, three cases arise.
= 3(q + 1) is divisible by 3.
Case I. When p = 3q,
and if n = 3q + 2 then only
p + 2 = 3q + 2 and p + 4 = 3q + 4
n + 4 = 3q + 6 = 3(q + 2)
Here, p = 3q is exactly divisible by 3
is divisible by 3.
p + 2 = 3q + 2 leaves 2 as remain-
WORKSHEET– 6 der when it is divided by 3
p + 4 = 3q + 4 or 3 (q + 1) + 1 leaves
1. (C) 3825 = 52 × 32 × 17 1 as remainder when it is
So, 11 is not a prime factor of 3825. divided by 3.
2. (C) As p and p + 1 are two consecutive Case II. When p = 3q + 1,
natural numbers, HCF = 1 and p + 2 = 3q + 3 and p + 4 = 3q + 5
LCM = p (p + 1).
Here, p = 3q + 1 leaves 1 as remainder
3. (A) when it is divided by 3
51 17 p + 2 = 3q + 3 or 3 (q + 1) is exactly
Hint: The given number is or
1500 500 divisible by 3
∴ Denominator = 500 = 22 × 53 p + 4 = 3q + 5 or 3(q + 1) + 2 leaves 2
Clearly, the denominator is exactly in the as remainder when it is
form 2m × 5n, where m and n are non- divided by 3.
negative integers; so the given number is Case III. When p = 3q + 2, p + 2 = 3q + 4
a terminating decimal expansion. and p + 4 = 3q + 6
Here, p = 3q + 2 leaves 2 as remainder
4. Hint: ... 8 = 23; 9 = 32; 25 = 52 when it is divided by 3.
∴ HCF (8, 9, 25) = 1 p + 2 = 3q + 4 or 3(q + 1) + 1 leaves 1 as
LCM (8, 9, 25) = 1800. remainder when it is divided by 3
5. Hint: HCF (210, 55) = 5 p + 4 = 3q + 6 or 3(q + 2) is exactly divisible
∴ 210 × 5 + 55y = 5 by 3.
⇒ 55y = 5 – 1050 Hence, in all the cases, one and one number
−1045 out of p, p + 2 and p + 4 is divisible by 3,
⇒ y= = – 19.
55 where p is any positive integer.

R E A L N U M B E R S 11
OR 5. The required number would be the HCF of
Any positive odd integer is type of 2q + 1 967 – 7 = 960 and 2060 – 12 = 2048.
where q is a whole number. Let us find the HCF of 960 and 2048 by
∴ (2q + 1)2 = 4q2 + 4q + 1 = 4q (q + 1) + 1 using Euclid’s algorithm.
...(i) Since 2048 > 960
Now, q(q + 1) is either 0 or even ∴ 2048 = 960 × 2 + 128
So it is 2m, where m is a whole number.
960 = 128 × 7 + 64
∴ from (i) ⇒ (2q + 1)2 = 8m + 1.
128 = 64 × 2 + 0
11. Since, height of each stack is the same,
Since the remainder becomes zero and the
therefore, the number of books in each stack
divisor at this stage is 64, the HCF of 960
is equal to the HCF of 96, 240 and 336.
and 2048 is 64.
Let us find their HCF
96 = 24 × 2 × 3 Hence, the required number is 64.
240 = 24 × 3 × 5 6.
336 = 24 × 3 × 7
So, HCF = 24 × 3 = 48.
Now, number of stacks of English books
96
= =2
48
Number of stacks of Hindi books
240
= =5
48 Clearly, 456 = 23 × 3 × 19
Number of stacks of Mathematics books and 360 = 23 × 32 × 5
336 ∴ HCF = 23 × 3 = 24
= = 7. and LCM = 23 × 32 × 5 × 19 = 6840.
48
7. Let us assume the contrary that 3 is a
ASSESSMENT SHEET–1 rational number.
a
1. (D) The denominator of each fraction in the So, 3 = , where a and b are coprime.
options (A), (B) and (C) can be expressed in b
the form 2n 5m, where m, n being whole a2
∴ 3= (Squaring both sides)
numbers. b2
2. (A) Let x be any positive integer then it is ⇒ 3b2 = a2
of the form 3q or 3q + 1 or 3q + 2. So, x2 can ⇒ a2 is divisible by 3
be written in the form 3m or 3m + 1. ⇒ a is divisible by 3 because 3 is a prime.
3. HCF × LCM = Product of the two numbers We can write a = 3c for some integer c
⇒ 40 × 252 × p = 2520 × 6600 Substituting a =3c in 3b2 = a2, we get
2520 × 6600 3b2 = 9c2 ⇒ b2 = 3c2
⇒ p= = 1650.
40 × 252 ⇒ b2 is divisible by 3
4. True, If the number 3n ends with the digit 0, ⇒ b is divisible by 3.
then its prime factorisation contains the Therefore, both a and b are divisible by 3.
prime 5. But by the Fundamental Theorem But this contradicts the fact that a and b are
of Arithmetic, there is no prime other than 3 coprime that is, no common factor other
in the factorisation of 3n. than 1.

12 M A T H E M A T I C S – X
Consequently, we arrive at the result that 5000 = 5 × 103 = 5 × (2 × 5)3
our assumption that 3 is rational, is wrong. = 23 × 54.
Hence, 3 is an irrational number. 257 257 257 × 2
Further, = 3
=
8. Let a be any odd positive integer. Then, it is 5000 5 × 10 5 × 10 3 × 2
of the form 6p + 1, 6p + 3 or 6p + 5. 514
Here, three cases arise. = = 0.0514.
10 4
Case I. When a = 6p + 1,
6. Let x = 2p + 1 and y = 2q + 1
∴ a2 = 36p2 + 12p + 1
∴ x2 + y2 = (2p + 1)2 + (2q + 1)2
= 6p(6p + 2) + 1 = 6q + 1,
= 4p2 + 4p + 1 + 4q2 + 4q + 1
where q = p(6p + 2).
= 4(p2 + p + q2 + q) + 2
Case II. When a = 6p + 3,
=S+T
∴ a2 = 36p2 + 36p + 9
= 36p2 + 36p + 6 + 3 where S = 4(p2 + p + q2 + q) and T = 2
= 6(6p2 + 6p + 1) + 3 S is divisible by 4 and so an even integer.
= 6q + 3, T is not divisible by 4 but an even integer.
where q = 6p2 + 6p + 1. Therefore, S + T is even, as sum of any two
Case III. When a = 6p + 5, evens is even, and not divisible by 4.
∴ a2 = 36p2 + 60p + 25 7. Let us assume the contrary that 5 is a
= 36p2 + 60p + 24 + 1 rational number.
= 6(6p2 + 10p + 4) + 1 We can take coprime a and b (say) such that
= 6q + 1, a
5 = ;b≠0
b
where q = 6p2 + 10p + 4.
Hence, a is of the form 6q + 1 or 6q + 3. ⇒ b 5=a
Square both the sides to get
ASSESSMENT SHEET–2 5b2 = a2
⇒ a2 is divisible by 5
14587
1. (D) = 11.6696. ⇒ a is divisible by 5 because if square of a
1250
number is divisible by a prime, then the
Clearly, the decimal expansion terminates number is divisible by the prime.
after four decimal places.
Let us take some integer c such that
2. (C) LCM (p, q) = x3 y2 z3.
a = 5c
3. HCF × LCM = Product of the two numbers.
Square both the sides to get
⇒ 9 × LCM = 306 × 657
a2 = 25c2
306 × 657 Substitute a2 = 25c2 in 5b2 = a2 to get
⇒ LCM = = 22338.
9 5b2 = 25c2
4. The maximum number out of 3, 5, 15, 25,
b2 = 5c2
75 is 75. Therefore, the HCF of 525 and 3000
is 75. ⇒ b2 is divisible by 5
⇒ b is divisible by 5
257
5. The denominator of is 5000. Therefore, both a and b are divisible by 5.
5000

R E A L N U M B E R S 13
This contradicts the fact that a and b are 3. (C)
coprime that is a and b have no common As, 8q is even and 6 is even, 8q + 6 is even.
factor.
56125 449
∴ Our assumption is false. 4. ... 0.56125 = =
100000 800
So, we conclude that 5 is an irrational 449 449
number. = = 5
32 × 25 2 × 52
8. Any positive integer n can be written in the ∴ 2n × 5 m = 2 5 × 5 2
form 3q, 3q + 1 or 3q + 2. n = 5, m = 2.
Here, three cases arise as follows:
( )
2
Case I. When n = 3q, 5. 2– 9 = 2 – 2 18 + 9
∴ n3 = (3q)3 = 27q3
= 11 – 2 18
∴ n3 + 1 = 27q3 + 1 = 9 × 3q3 + 1
= irrational.
= 9m + 1, where m = 3q3. 6. Yes.
Case II. When n = 3q + 1, 2 × 3 × 5 × 13 × 17 + 13
∴ n3 = (3q + 1)3 = 13 × (2 × 3 × 5 × 17 + 1)
= 27q3 + 1 + 3(3q + 1) × 3q = 13 × 511
= a composite number.
= 27q3 + 27q2 + 9q + 1
∴ n3 + 1 = 27q3 + 27q2 + 9q + 2 7. No.
Hint: Prime factors of 9n will be type of
= 9 (3q3 + 3q2 + q) + 2
= 9m + 2, 32n, i.e., 3 × 3 × ... 3
 
where m = 3q3 + 3q2 + q Even no.
of times.
Case III. When n = 3q + 2,
8. 120 = 23 × 3 × 5
∴ n3 = (3q + 2)3 = 27q3 + 8 +
105 =3× 5 × 7
3 × 6q (3q + 2) 150 = 2 × 3 × 52
= 27q3 + 8 + 54q2 + 36q ∴ HCF = 3 × 5 = 15
∴ n3 +1= 27q3 + 54q2 + 36q + 9 And LCM = 23 × 3 × 52 × 7
= 8 × 3 × 25 × 7
=9 (3q3 + 6q2 + 4q + 1)
= 4200.
= 9m, where m = 3q3 + 9. Hint:
6q2 + 4q + 1.
Let 2 – 3 3 = x where x is rational.
Hence, n3 + 1 can be expressed in the form
( )
2
9m, 9m + 1 or 9m + 2 for some integer m. ⇒ 2–3 3 = x2

CHAPTER TEST ⇒ 2 + 27 – 6 6 = x2
⇒ 29 – x2 = 6 6
1. (D) Since 32844 = 2 × 2 × 3 × 7 × 17 × 23
So, 11 is not prime factor of 32844. 29 – x 2
⇒ = 6 6.
2. (A) 6

.. . 306 × 1314 Since 6 is not a perfect square. So 6 is


LCM =
18 always irrational.
= 22338. ∴ It's a contradiction.

14 M A T H E M A T I C S – X
10. We know that any positive integer is of the n3 + 1 = 27q3 + 54q2 + 36q + 9
form 3q or 3q + 1 or 3q + 2. = 9(3q3 + 6q2 + 4q + 1)
Case I: n = 3q = 9m,
⇒ n3 = (3q)3 = 9 × 3q3 = 9m where m = 3q3 + 6q2 + 4q + 1.
⇒ n3 + 1 = 9m + 1, where m = 3q3. Hence, n3 + 1 can be expressed in the form
9m, 9m + 1 or 9m + 2, for some integer m.
Case II: n = 3q + 1
11. Length = 8.25 m = 825 cm
⇒ n3 = (3q + 1)3
Breadth = 6.75 m = 675 cm
= 27q3 + 1 + 27q2 + 9q Height = 4.50 m = 450 cm
= 9q (3q2 + 3q + 1) + 1 The required length of the rod will be the
= 9m + 1 highest common factor of 825 cm, 675 cm
⇒ n3 + 1 = 9m + 2, where and 450 cm.
m = q(3q2 + 3q + 1). Now, 825 = 3 × 52 × 11
Case III: n = 3q + 2 675 = 33 × 52
450 = 2 × 32 × 52
⇒ n3 = (3q + 2)3
So, HCF (825, 675, 450) = 3 × 52 = 75
= 27q3 + 8 + 54q2 + 36q Hence, length of the rod is 75 cm.
❑❑

R E A L N U M B E R S 15
Chapter

2 POLYNOMIALS

WORKSHEET–10 8. Solving α + β = 3 and α – β = –1,


we get α = 1, β = 2
1. (C)
... Polynomial is x2 – (α + β) x + αβ
Hint: put x2 + 2x + 1 = 0 and solve for x.
⇒ p(x) = x2 – 3x + 2.
2. (C) Since the given graph of y = p(x) cuts
x-axis at three points, so the number of 9. According to the division algorithm,
zeroes of p(x) are 3. p(x) = g(x) × q(x) + r(x)
3. (A) ⇒ x3 – 3x2 + x + 2 = g (x) × (x – 2) + (– 2x + 4)
1 1 α+β (As given in question)
Hint: + = .
α β αβ x3 – 3x2 + 3x – 2
⇒ g(x) =
4. Let one zero be α, then the other one will x–2
1 To find g(x), we proceed as given below.
be .
α
1 k
∴ α. =–
α 15
⇒ k = – 15.

2 3
5. Sum of zeroes (S) = – +
3 4
3–8 5
= = –
4 3 4 3

2 3 1
Product of zeroes (P) = – × =–
3 4 2 Thus, g(x) = x2 – x + 1.
Now, required polynomial will be 1 3
5 1 10. − ;
x2 – Sx + P, i.e., x2 + x – 3 2
4 3 2
Hint: 6x2 – 7x – 3 = 0
or 4 3 x2 + 5x – 2 3 . ⇒ 6x2 – 9x + 2x – 3 = 0
6. Let f (x) = 2x2 + 2ax + 5x + 10 ⇒ 3x (2x – 3) + 1 (2x – 3) = 0
If x + a is a factor of f (x), then f (– a) = 0 ⇒ (2x – 3) (3x + 1) = 0
Therefore, 2a2 – 2a2 – 5a + 10 = 0
3 1
⇒ a = 2. ... x= or −
2 3
7. x3 – 4x2 + x + 6
Hint: If the roots are α, β and γ of a cubical −1 3 7 –b
... α+β= + = =
polynomial, then the polynomial will be 3 2 6 a
(x – α) (x – β) (x – γ) –1 3 –1 c
= (x – 3) (x – 2) (x + 1) = x3 – 4x2 + x + 6. ... α.β= . = = .
3 2 2 a

16 M A T H E M A T I C S – X
11. Let p(x) = x4 + x3 – 34x2 – 4x + 120 5. p = 2
Given zeroes of p(x) are 2 and – 2 Hint: (2)3 – 3(2)2 + 3(2) – p = 0
... (x – 2) (x + 2) = x4 – 4 is a factor of p(x). ⇒ 8 – 12 + 6 – p = 0
We divide p(x) by x2 – 4, ⇒ 2 – p =0
x2 + x – 30 ∴ p = 2.
x2 – 4 x4 + x3 – 34x2 – 4x + 120 6. Let α and β be the two zeroes of
x4 – 4x2 f(x) = ax2 + 2x + 3a
– +
2 3a
x3 – 30x2 – 4x +120 Then, α + β = – and αβ = =3
a a
x3 – 4x According to the question,
– +
– 30x2 + 120 –2
=3
– 30x2 + 120 a
+ –
0 –2
⇒ a= .
3
... p(x) = (x2 – 4) (x2 + x – 30)
. 7. Let the third zero be α, then
. . Other zeroes of p(x) are given by
x2 + x – 30 = 0 coefficient of x 2
sum of the zeroes = –
2
⇒ x + 6x – 5x – 30 = 0 coefficient of x 3
⇒ x(x + 6) – 5(x + 6) = 0 –6
⇒ 2 + 3 + α= –
⇒ (x – 5) (x + 6) = 0 1
x = 5, – 6 ⇒ α= 1
Hence, all the zeroes are 2, – 2, 5 and – 6. Hence, the third zero is 1.
8. Let us divide 6x4 + 8x3 + 17x2 + 21x + 7 by
WORKSHEET– 11 3x2 + 4x + 1.
1. (A) ∵ p(x) = 2x2 – 2x + 1
∴ Sum of zeroes = 1
1
Product of zeroes = .
2
2. (A) Let α = 5 and β = – 5, then the quad-
ratic polynomial will be x2 – (α + β)x + αβ
or x2 – 25.
Clearly, the remainder is x + 2.
3. (D) Let us take option (D)
Now, ax + b = x + 2
p(x) = (x2 – 2) – (x2 + 3x) = – 3x – 2
Comparing like powers of x both the sides,
This is a linear polynomial. we obtain
4. For zeroes of p(x), put p(x) = 0 a = 1, b = 2.
⇒ 4x2 – 4x + 1 = 0 9. We know that,
⇒ 2
4x – 2x – 2x + 1 = 0 Dividend = (Divisor × Quotient) + Remainder
⇒ 2x (2x – 1) – 1(2x – 1) = 0 ⇒ 4x3 – 8x2 + 8x + 1 = g(x) × (2x – 1) + x + 3
⇒ (2x – 1) (2x – 1) = 0
⇒ g(x) × (2x – 1) = 4x3 – 8x2 + 7x – 2
∴ 2x – 1 = 0
1 1 4x 3 – 8x 2 + 7 x – 2
∴ x= , . g(x) =
2 2 2x – 1

P O L Y N O M I A L S 17
Now, 2. (C) Sum of zeroes = 6
2
2x – 3x + 2 – 3k
⇒ 6= –
2x – 1 4x3 – 8x + 7x – 2
2 1
6
4x3 – 2x2 ∴ k= = 2.
– + 3
– 6 x2 + 7 x – 2 3. (D) Let one zero be α, then the other one
– 6x2 + 3x 1
+ – will be .
α
4x – 2
4x – 2 1 4a
– + So, α.=
α a2 + 4
0 a2 – 4a + 4 = 0
Hence, g(x) = 2x2 – 3x + 2. ⇒ (a – 2)2 = 0
⇒ a = 2.
10. 3 and 1 4. (A) Let the zeroes be α, β, γ. If γ = – 1, then
Hint: x2 – 3 x – x + 3 = 0 c
αβγ = –
1
⇒ x= 3, 1 If γ = – 1, then αβ = c ...(i)
Now, sum of zeroes = 3+ 1 Further, (– 1)3 + a (– 1)2 + b (– 1) + c = 0
⇒ –1 + a–b +c = 0
Coefficient of x
=– ⇒ c=b–a+1 ...(ii)
Coefficient of x 2
From equations (i) and (ii), we have
And product of zeroes = 3 αβ = b – a + 1.
Constant term 5. Given polynomial is:
= .
Coefficient of x 2 f (x) = x2 – px – 2p – c
.
.. α + β= p
(
11. p(x) = x 2 − 2x − 12; − 2 2, 3 2 ) and α . β = – 2p – c
2
Hint: For zeroes: x – 2x – 12 = 0 ... (α + 2) (β + 2) = αβ + 2 (α + β) + 4
= – 2p – c + 2p + 4
⇒ x 2 + 2 2 x − 3 2 x − 12 = 0
= (4 – c).
⇒ (x – 3 2 ) (x + 2 2 ) = 0 6. λ = 6
⇒ x = – 2 2 or x = 3 2 . Hint: (α + β)2 = (α − β)2 + 4αβ.
7. x = –1 or 3; f(x) = x2 – 2x – 3
WORKSHEET–12 Hint: x= – 1 or 3,
∴ Sum of zeroes = 2
– (– 5) Product of zeroes = –3
1. (C) Sum of zeroes = = 15
 1 ∴ p(x) = x2 – (α + β)x + αβ
 
3 = x2 – 2x – 3.
3 47
8. x2 – x –
2 9 4
Product of zeroes = = .
1 2 Hint: f (x) = {x2 – (sum of roots) x + (product
3 of roots)}

18 M A T H E M A T I C S – X
9. The number which to be subtracted is the ⇒ 3(x + 1)2 = 0
remainder when 4x4 + 2x3 – 8x2 + 3x – 7 is ⇒ x = – 1 and x = – 1
divided by 2x2 + x – 2. To find the remainder, 5,
we proceed as following. Hence, all the zeroes of p(x) are
3
2x2 – 2 5
2x2 + x – 2 4x4 + 2x3 – 8x2 + 3x – 7 – , – 1 and – 1.
3
4x4 + 2x3 – 4x2
– – + WORKSHEET–13
– 4x2 + 3x – 7 1. (A)
2
– 4x – 2x + 4 Hint: Given polynomial can be written as:
+ + –
p(x) = 2x2 + 3x – 11
5x – 11
–b
Hence, 5x – 11 must be subtracted from Sum of zeroes =
a
4x4 + 2x3 – 8x2 + 3x – 7 so that it becomes c
Product of zeroes = .
exactly divisible by 2x2 + x – 2. a
10. g(x) = x2 + 2x + 1 2. (B) Sum of zeroes = – 99 = –ve
Hint: p(x) = g(x) × q(x) + r(x) Product of zeroes = 127 = +ve
p ( x ) – r( x ) If the sum of both zeroes is negative, then
⇒ g(x) = the zeroes would be either both negative
q( x )
or one negative and other one positive. If
where, p(x) = 3x3 + x2 + 2x + 5 the product of both the zeroes is positive,
q(x) = 3x – 5 then the zeroes would be either both
and r(x) = 9x + 10. positive or both negative.
5 5 Consequently, we obtain that both the
11. Since x = and x = – are zeroes of zeroes are negative.
3 3
p(x) = 3x4 + 6x3 – 2x2 – 10x – 5, so p(x) is 3. (D) We know that the degree of the remain-
der is less than the degree of divisor.
 5  5  5
divisible by  x –  x +  , i.e., x2 – 3 . Here, degree of the divisor is 3, therefore,
 3  3 the possible degree of the remainder can
be any out of 0, 1 and 2.
4. Hint: Substitute x = – 2 in x2 + 2x + k = 0.
5. Since α, β are the zeroes of x2 + px + q, then
α + β = –p; αβ = q
1 1 α +β p
Now, + = =–
α β αβ q
1 1 1 1
And × = =
α β αβ q
1 1
So the polynomial having zeroes and
α β
will be
 1 1  1 1 p 1
Here, other two zeroes of p(x) are the other p(x) = x2 –  +  x +  ×  = x 2 + x +
two zeroes of quotient 3x2 + 6x + 3 α β α β q q
Put 3x2 + 6x + 3 = 0 or p(x) = qx2 + px + 1.

P O L Y N O M I A L S 19
6. g(x) = x2 + 2x + 7.
1 1 α 2 + β2
Hint: Divide x3 + 3x – 14 by x – 2. 11. Hint: + =
α2 β2 α 2β2
7. p(x) = 3x2 – 3x + 12.
g(x) = x2 – x + 4 =
(α + β )2 − 2αβ = b2 – 2ac .
2 2 2
... q(x) = 3 αβ c
r(x) = 0. OR
Let us divide x4 + 2x3 + 8x2 + 12x + 18 by x2 + 5.
1 1
8. ,–
7 7
Hint: For zeroes: 21x2 – 3 = 0
1
x2 =
7
1
... x= ± .
7
9. Since a = 2 is a zero of a3 – 3a2 – 10a + 24,
therefore a3 – 3a2 – 10a + 24 is divisible by
a – 2. Further the obtained quotient will
provide the other two zeroes.
Clearly, the remainder is 2x + 3.
Now, px + q = 2x + 3
Comparing like powers of x both the sides,
we get
p = 2, q = 3.

WORKSHEET–14
1. (C) If a quadratic polynomial has equal
roots , then its discriminant must be zero.
So, b2 – 4ac = 0
⇒ b2 = 4ac
This last equation holds if a and c have
Put a2 – a – 12 = 0 for other zeroes. same sign.
⇒ (a – 4) (a + 3) = 0 2. (D) Sum of zeroes = – 3 + 7 = 4,
⇒ a = –3, 4 Product of zeroes = (– 3) × 7 = – 21
A polynomial may be k(x2 – 4x – 21)
Thus, the other two zeroes are – 3 and 4.
where k has infinitely many real values.
10. g(x) = x + 1. Hence, infinitely many number of poly-
Hint: Applying division algorithm, we get nomials can be.
x4 + 1 = g(x) × (x3 – x2 + x – 1) + 2 3 1
3. (A) α + β = , αβ =
x –14 2 2
⇒ g(x) =
x – x2 + x – 1
3
... (α – β)2 = (α + β)2 – 4αβ
( x + 1 )( x – 1 )( x 2 + 1 ) 9 1
= =
–2=
( x – 1 )( x 2 + 1 )
4 4
1
= x + 1. ⇒ α–β= ±
2

20 M A T H E M A T I C S – X
1 1 (i) p(x) = 6x2 + 3x + 2, g(x) = 3
... α = , β = 1 or α = 1, β = q(x) = 2x2 – x, r(x) = – 2
2 2
(ii) p(x) = 8x3 + 6x2 – x + 7, g(x) = 2x2 + 1
5
... α + 2 = , β + 2 = 3 or α + 2 = 3, q(x) = 4x + 3, r(x) = – 5x + 4
2
(iii) p(x) = 9x2 + 6x + 5, g(x) = 3x + 2,
5 q(x) = 3x, r(x) = 5.
β+2= .
2
10. Given quadratic polynomial is
Hence, the required polynomial can be
5 5 x2 + 30x + 8 5
5  5 11 15 To find its zeroes, put
x2 –  + 3 x + × 3, i.e., x2 – x+ .
2  2 2 2
5 5 x2 + 30x + 8 5 = 0
4. Let zeroes be α and β.
⇒ 5 5 x2 + 20x + 10x + 8 5 = 0
α + β = 6, αβ = 4
Using (α – β)2 = (α + β)2 – 4αβ, we get ⇒ 5x ( )
5x + 4 + 2 5 ( )
5x + 4 = 0
(α − β)2 = 62 – 4 × 4 = 20 ⇒ α – β, = ± 2 5
⇒ ( 5x + 2 5) ( 5x + 4) = 0
Thus, the difference of zeroes is ± 2 5 .
2 –4
α β α 2 + β2 ⇒ x=– or x =
5. Hint: + = 5 5
β α αβ
2 5 4 5
(α + β )2 − 2αβ i.e., x=– or x = –
= 5 5
αβ
25 − 12 13 –2 5 4 5 6 5
= = . So, sum of zeroes = – =–
6 6 5 5 5
6. Hint: x2 – 1 = (x + 1) (x – 1) And product of zeroes
... x = – 1 or 1, both will satisfy with the  2 5  4 5 8
given polynomial. = –  × –  = .
 5   5  5
... we get, p + q + r + s + t = 0 ...(i)
Coefficient of x
and p–q+r–s+t=0 ...(ii) Also, sum of zeroes = –
Coefficient of x 2
From (ii),
p+r+t=q+s 30 6 5
=– =–
From (i), 5 5 5
2 (q + s) = 0 ⇒ q + s = 0 Constant term
.. . p + r + t = q + s = 0. And product of zeroes =
Coefficient of x 2
7. No.
8 5 8
Hint: Divide q(x) by g(x). If the remainder = = .
obtained is zero, then the g(x) is a factor of 5 5 5
q(x) otherwise not. Hence verified.
8. a = 1, b = 7 OR
Hint: Put remainder = 0 and equate coefficient α−1 β−1
of x in the remainder and constant term Hint: Let S = +
α+1 β+1
with zero.
9. According to division algorithm,  α − 1  β − 1 
P=  
p(x) = g(x) × q(x) + r(x)  α + 1  β + 1 

P O L Y N O M I A L S 21
... Required polynomial p(x) = x2 – Sx + P. 2. (A)
1 1 1 α+β+ γ
3 and – 3 are the zeroes of the given Hint: + + = .
11. As αβ βγ γα αβγ
2 2
3. (D) Let zeroes be α and β, then
 3
quadratic polynomial, so  x –  and (α – β)2 = 144
 2
⇒ α – β = + 12 ...(i)
 3 α + β= – p ...(ii)
x +  will be the factors of that, Conse- αβ = 45 ...(iii)
 2
Also, we have
 3  3  2 3 (α – β)2 = (α + β)2 – 4αβ
quently,  x –  × x +  , i.e.,  x – 
 2  2 2 144 = p2 – 180
⇒ p = ± 18.
must be the factor of that. Let us divide
3 4. Let the given linear polynomial be
2x4 – 10x3 + 5x2 + 15x – 12 by x2 – . y = ax + b ....(i)
2
This passes through points (1, –1), (2, 1) and
2x2 – 10x + 8
3 2x4 – 10x3 + 5x2 + 15x – 12 3 
x2 –  , 0

2 2
2x4 – 3x2
–– + ∴ –1= a+ b ...(ii)
– 10x3 + 8x2 + 15x – 12 1 = 2a + b ...(iii)
– 10x3 + 15x 3
+ – 0= a+b ...(iv)
2
8x2 – 12
Solving equations (ii) and (iii), we get a = 2,
8x2 – 12 b = – 3 which satisfy to equation (iv).
– +
0 Consequently, using equation (i), we get
y = 2x – 3
Now, 2x4 – 10x3 + 5x2 + 15x – 12 ∴ Polynomial is p(x) = 2x – 3
 3 3
=  x2 – 2 Since p(x) = 0 if x =
  (2x – 10x + 8) 2
2
3
By splitting –10x, we factorise 2x2 – 10x + 8 ⇒ x= is zero of p(x).
as (x – 4) (2x – 2). So, its zeroes are given 2
by x = 4 and x = 1. 5. Let us divide ax3 + bx – c by x2 + bx + c by
Therefore, the zeroes of the given poly- the long division method.

3, – 3 , 1 and 4. ax – ab
nomial are 3
2 2 x2 + bx + c ax + bx – c
ax + abx2 + acx
3
– – –
WORKSHEET–15 – abx2 + (b – ac) x – c
1. (B) – abx2 – ab2 x – abc
+ + +
Hint: f (x) = x2 – px – ( p + c) (ab2 + b – ac)x + abc – c
(α + 1) (β + 1) = αβ + (α + β) + 1 .
Put remainder = 0

22 M A T H E M A T I C S – X
⇒ (ab2 + b – ac)x + (abc – c) = 0
⇒ ab2 + b – ac = 0 and abc – c = 0
10. If 2 ± (
3 are zeroes of p(x), then x – 2 + 3 )
Consider abc – c = 0 ⇒ (ab – 1) c = 0 ( )
and x – 2 – 3 are factors of p(x).
⇒ ab = 1 or c = 0. Hence, ab = 1.
6. Hint: Let p(x) = x3 – mx2 – 2npx + np2 Consequently {x – (2 + 3 )} {x – (2 – 3 )}
(x – p) is a factor of p(x) i.e., (x – 2)2 – 3, i.e., x2 – 4x + 1 is factor of
⇒ p(x) = 0 at x = p. p(x).
⇒ p3 – p2m – p2n = 0 Further,
⇒ p2 [(p – (m + n)] = 0
⇒ p = m + n where p ≠ 0. x2 – 2x – 35
7. x3 – 4x2 + x + 6 x2 – 4x + 1 x4 – 6x3 – 26x2 + 138x – 35
Hint: The required cubic polynomial is given x4 – 4x3 + x2
– + –
by (x – 3) (x – 2) (x + 1) or x3 – 4x2 + x + 6
– 2x3 – 27x2 + 138x – 35
This is the required polynomial.
– 2x3 + 8x2 – 2x
8. – 2, 3, 4 + – +
Hint: α+β+γ=5
– 35x2 + 140x – 35
αβ + βγ + αγ = –2
– 35x2 + 140x – 35
αβγ = –24 + – +
Let αβ = 12 0
... γ = –2
Clearly x2 – 2x – 35 is a factor of p(x)
... α+β =7
⇒ (x – 7)(x – 5) is a factor of p(x)
⇒ (α – β)2 = 1
⇒ x – 7 and x + 5 are factors of p(x)
⇒ α–β =±1
⇒ x – 7 = 0 and x + 5 = 0 give other zeroes
... α + β = 7 and α – β = 1
of p(x)
⇒ α =4
⇒ x = 7 and x = – 5 are other zeroes of p(x).
β =3
Hence, 7 and – 5 are required zeroes.
or α + β = 7 and α – β = –1
⇒ α =3 α2 β 2 α 4 + β4
β = 4. 11. Hint: 2
+ 2 =
β α α 2β 2
9. f (x) would become exactly divisible by g(x)
if the remainder is subtracted from f(x). {(α + β)2 − 2αβ}2 − 2α 2β2
Let us divide f(x) by g(x) to get the remainder. = .
α 2 β2
x2 + 6x + 8
OR
x2 – 4x + 3 x4 + 2x3 – 13x2 – 12x + 21
Given polynomial is:
x4 – 4x3 + 3x2
– + – f (x) = pqx2 + (q2 – pr)x – qr
6x3 – 16x2 – 12x + 21 Put f (x) = 0 to find roots.
6x3 – 24x2 + 18x pqx2 + (q2 – pr) x – qr = 0
– + –
2
8x – 30x + 21 ⇒ pqx2 + q2x – prx – qr = 0
8x2 – 32x + 24 ⇒ qx(px + q) – r(px + q) = 0
– + – ⇒ (px + q)(qx – r) = 0
2x – 3 q r
⇒ x=– or x =
Hence, we should subtract 2x – 3 from f(x). p q

P O L Y N O M I A L S 23
Sum of roots =–
q r
+ =
pr – q 2 Let us divide f(x) by x – 2 . ( )
p q pq
6x2 + 7 2 x + 4
Coefficient of x
=– 2 x – 2 6x3 +  2x2 – 10x – 4  2
Coefficient of x
6x 3 – 6 2 x 2
– + 
q r r qr
Product of roots = – p × q = – p = – pq 7  2x2 – 10x – 4  2

Constant term 7  2x2 – 14x


= . – +
Coefficient of x 2 4x – 4  2
4x – 4  2
ASSESSMENT SHEET– 3 – +
0
1. (C) Discriminant = 0
⇒ b2 – 4ac = 0 ∴ f (x) = x – 2( ) (6 x 2 + 7 2x + 4 )
⇒ b2 = 4ac
b2
LHS = = positive sign
(
= x– 2 ) (6x2 + 3 2x + 4 2x + 4 )
⇒ RHS = 4ac must be positive sign.
(
= x– 2 ) (3 2x + 4 )( 2x + 1 )
⇒ c and a have same signs.
2 2
2. (D) Required quadratic polynomial (
Hence, 3 2 x + 4 )( )
2 x + 1 gives x = –
3
= x2 – (sum of zeroes)x + product of zeroes
1
= x2 – 2 3x – 5 3 . or x = –
2
3. p(x)= x2 – ax – (a + 1)
2 2
At x = – 1, p(x) = (–1)2 – a( –1) – (a + 1) Therefore, other two zeroes are – and
3
= 1 + a – a –1 = 0
q(x) = ax2 – x – (a + 1) 2
– .
at x = – 1, q(x) = a( –1)2 – ( –1) – (a + 1) 2
= a + 1 – a –1 = 0 3 5
6. p(y) = y2 + y–5
Therefore, x + 1 is the common factor of 2
p(x) and q(x).
3 5
4. Correct, Here, a = 1, b = ,c=–5
2
f(x) = x2 – p(x +1) – c = x2 – px – (c + p) Discriminant
∴ α + β = p; αβ = – (c + p) 2
3 5 
Now, (α + 1) (β + 1) = αβ + (α + β) + 1 D = b2 – 4ac =   – 4 × 1 × (– 5)
 2 
= – (c + p) + p + 1
= –c–p+p+1 45 125
= + 20 =
= 1 – c. 4 4

5. Let f(x) = 6x3 + 2 x2 – 10x – 4 2 –3 5 125


±
As 2 is a zero of f (x), (x – 2 ) is a factor –b± D 2 4
Now, y = =
of f(x). 2a 2×1

24 M A T H E M A T I C S – X
Here, remainder is (20 + a)x + b + 25.
–3 5 5 5
± If the polynomial p(x) is exactly divisible
2 2 by 2x2 – 5, the remainder must be zero.
=
2 ∴ (20 + a)x + (b + 25) = 0
Comparing the coefficients of like powers
2 5 –8 5
of x between both the sides, we have
= 2 or 2
2 2 20 + a = 0 and 25 + b = 0
⇒ a = – 20 and b = – 25.
5
= or – 2 5 ASSESSMENT SHEET–4
2

5 –3 2
Hence, the zeroes are and – 2 5 .
2 1. (C) Sum of zeroes = – = 2
3
7. α and β are zeroes of f (x) = x2 – x – 2 1
Product of zeroes = .
–1 3
Sum of roots = α + β = – =1 ...(i)
1 2. (B) At x = 2, p(x) = 0, i.e., p(2) = 0
–2 ∴ a (2)2 – 3 × 2 (a – 1) – 1 = 0
Product of roots = αβ = =–2 ...(ii)
1 ⇒ 4a – 6a + 6 – 1 = 0
∴ (2α + 1) + (2β + 1) = 2(α + β) + 2 5
⇒ a= .
= 2(1) + 2 [Using (i)] 2
3. Sum of zeroes = α + β = 5
=4 ...(iii)
Product of zeroes = αβ = 4
And (2α + 1) (2β + 1) = 4αβ + 2α + 2β + 1
1 1 α +β
= 4αβ + 2 (α + β) + 1 Now, + – 2αβ = – 2αβ
α β αβ
= 4 (– 2) + 2 (1) + 1
[Using (i) and (ii)] 5
= –2×4
= –5 ...(iv) 4
Now, required polynomial can be given by 27
= – .
x2 – {(2α + 1) + (2β + 1)}x + (2α + 1)(2β + 1) 4
i.e., x2 – 4x – 5. [Using (iii) and (iv)] 4. Using division algorithm, we have
8. Let us divide p(x) by 2x2 – 5. g(x) × (x – 2) – 2x + 4 = x3 – 3x2 + x + 2

3x2 + 4x + 5 x 3 – 3x 2 + 3x – 2
⇒ g(x) =
2x2 – 5 6x4 + 8x3 – 5x2 + ax + b x–2
6x4 – 15x2 Here, at x = 2, x3 – 3x2 + 3x – 2
– +
8x3 + 10x2 + ax + b = 8 – 12 + 6 – 2 = 0
8x3 – 20x ∴ = x3 – 3x2 + 3x – 2
– +
= (x – 2) (x2 – x + 1)
2
10x + (20 + a)x + b
10x2 – 25 ( x – 2)( x2 – x + 1)
=
– + ( x – 2)
(20 + a)x + b + 25 ⇒ g(x) = x2 – x + 1.

P O L Y N O M I A L S 25
3 Let us draw the graph of p(x) using this
5. Given s = 2 and p = – table.
2
The required polynomial is given by
Y
k [x2 – sx + p]
(0,6)
3
i.e., k  x 2 – 2x –  , where k is any real 6– (1,6)
 2
5–
number.
(–1, 4) 4– (2, 4)
6. Let f(x) = 4 3 x 2 + 5 x – 2 3
3–
2
= 4 3 x + 8 x – 3x – 2 3 2–
= 4x( 3x + 2) – 3 ( 3x + 2) 1–
(–2, 0) (3, 0)
= ( 3x + 2)(4x – 3) X X


–3 – 2 –1 0 1 2 3
To find zeroes of f (x), put f(x) = 0 –1 –

3x + 2 = 0 or 4 x – 3 = 0 –2 –

–3 – p(x) = – x2 + x + 6
–2 –2 3 3
x= = or x = Y
3 3 4
From the graph, it is clear that the zeroes
2 3 3
Thus, the zeroes are α = – and β = of p(x) are – 2 and 3.
3 4
(ii) Let y = p(x)
Sum of zeroes = α + β
∴ y = x3 – 4x
2 3 3 –5 3
=– + = The table for some values of x and their
3 4 12 corresponding values of y is given by
5 3 5 x –2 –1 0 1 2
= – =–
4× 3 4 3
y 0 3 0 –3 0
Coefficient of x
=– Let us draw the graph of p(x) by using this
Coefficient of x 2
table.
2 3 . 3 Y
Product of zeroes = αβ = – (–1, 3)
3 4 3–

2 3 Constant term 2–
=– = .
4 3 Coefficient of x 2 1–
(–2, 0) (2, 0)
Hence verified. X X

–3 – 2 –1 0 1 2 3
7. (i) Let y = p(x) –1 –
∴ y = – x2 + x + 6 –2 –
The table for some values of x and their p(x) = x3 – 4x – –
3 (1, –3)
corresponding values of y is given by Y
x –2 –1 0 1 2 3 From the graph, it is clear that the zeroes of
y 0 4 6 6 4 0 p(x) are – 2, 0 and 2 .

26 M A T H E M A T I C S – X
8. Let f (x) should be added to p(x) so that the 7
resulting polynomial is exactly divisible by ⇒ x = 0 or – .
2
g(x). Since the degree of f (x) is less than
So, option (B) is correct.
that of g(x).
So, f(x) may be ax + b i.e., f(x) = ax + b. p( x ) – (7 – 5 x )
4. Hint: g(x) =
Therefore, the new dividend would be 2x
x4 + 2x3 – 2x2 – 5x + 7 + ax + b,
i.e., x4 + 2x3 – 2x2 + (a – 5)x + b + 7 2x 3 + 4 x 2 + 5x + 7 – 7 + 5x
=
Let us divide this new dividend by g(x). 2x
= x2 + 2x + 5.
2
x +1
24 6
x2 + 2x – 3 x4 + 2x3 – 2x2 + (a – 5)x + b + 7 5. Hint: α + β = =
4 5 5
x4 + 2x3 – 3x2
– – +
x2 + (a – 5)x + b + 7 –9 5 –9
αβ = = .
x2 + 2x – 3 4 5 4
– – +
(a – 7)x + b + 10 6. Hint: α= –β
–b
Thus, the remainder obtained must be zero. α+β= 0 ⇒ =0
a
∴ (a – 7)x + b + 10 = 0
⇒ (a – 7)x + (b + 10) = 0 . x + 0 3k + 1 –1
⇒ =0 ⇒ k= .
⇒ a – 7 = 0 and b + 10 = 0 2 3
⇒ a = 7 and b = – 10 7. Answer may vary.
Hence, f (x) = 7x – 10. 8. If α, β and γ are the zeroes of a cubic
polynomial f(x), then
CHAPTER TEST
f(x) = x3 – (α + β + γ) x2
–5 1 + (αβ + βγ + γα) x – αβγ
1. (A) ∵ α + β = , αβ =
2 2 Here, α + β + γ = 4, αβ + βγ + γα = 1
∴ α + β + αβ = –2 and αβγ = – 6
So, option (A) is correct. ∴ f(x) = x3 – 4x2 + x + 6.
2. (B) 9. We know that
p(x) = x2 – (α + β)x + αβ Dividend = Quotient × Divisor
= x2 + x – 2 + Remainder
So, option (B) is correct. ⇒ Dividend – Remainder = Quotient
3. (B) × Divisor
Clearly, RHS of the above result is divisible
Hint: p(x) = x  x + 7  by the divisor. Thus, if we subtract remainder
 2 from the dividend, then it will be exactly
∴ zeroes are given by divisible by the divisor.
 7 Dividing x4 + 2x3 – 13x2 – 11x + 10 by
xx+  = 0
 2 x2 – 4x + 3, we get

P O L Y N O M I A L S 27
∴ Other zeroes are given by
x2 + 6x + 8
3x2 – 15x + 18 = 0
x2 – 4x + 3 x4 + 2x3 – 13x2 – 11x + 10
⇒ x2 – 5x + 6 = 0
x4 – 4x3 + 3x2
– + – ⇒ (x – 3) (x – 2) = 0
6x – 16x2 – 11x +10
3
∴ x = 3, 2.
6x3 – 24x2 + 18x 11. We have
– + –

( )( )
2
8x – 29x + 10
4 3 x 2 + 5x – 2 3 = 3 x + 2 4x – 3
8x2 –32x + 24
– + –
So, the value of 4 3 x 2 + 5 x – 2 3 is zero
3x – 14
Quotient = x2 – 6x + 8 and when,
remainder = 3x – 14. –2
3 x + 2 = 0 or 4 x – 3 = 0, i.e., when x =
Thus, if we subtract the remainder 3x – 14 3
from x4 + 2x3 – 13x2 – 11x + 10, it will be
divisible by x2 – 4x + 3. 3
or x = . Therefore, the zeroes of
4
–1 1
10. Hint: Since x = and x = are zeroes. –2 3
3 3 4 3x2 + 5x – 2 3 are and .
3 4
 1  1 
Therefore,  x –  x +  will be a Now,
 3  3
–2 3 –5
2 1 sum of zeroes + =
factor of p(x), i.e., x – is a factor of p(x). 3 4 4 3
3
3x2 – 15x + 18 – Coefficient of x
1 3x4 – 15x3 + 17x2 + 5x – 6 =
x2 – Coefficient of x 2
3
3x4 – x2
– +  –2  3  –2 3
Product of zeroes =  × =
– 15x3 + 18x2 + 5x – 6  3   4  4 3
– 15x3 + 5x
+ –
Constant term
2
18x – 6 = .
Coefficient of x 2
18x2 – 6
– +
0
❑❑

28 M A T H E M A T I C S – X
Chapter

3 PAIR OF LINEAR EQUATIONS IN TWO VARIABLES

WORKSHEET–18 1 –2 8
Here, = =
1. (B) Since (3, a) lies on the equation 5 –10 40
2x – 3y = 5, therefore, (3, a) must satisfy
⇒ The equations represent a pair of
this equation.
coincident lines.
∴ 2 (3) – 3 (a) = 5
⇒ The equations have infinitely many
1 solutions.
⇒ 3a = 1 ⇒ a= .
3
7. The given equations are
2 –3 4(2x + 3y) = 9 + 7y
2. (A) Hint: =
k –9 and 3x + 2y = 4
⇒ k = 6. or 8x + 5y –9 = 0
3. (C) Hint: The condition of inconsistency of 3x + 2y – 4 = 0
two equations a1x + b1y = c1 and a2x + b2y = c2 By cross-multiplication, we have
is given by a1 = b1 ≠ c1 . x −y 1
a2 b2 c2 = =
− 20 + 18 − 32 + 27 16 − 15
4. x = 1, y = 2 –y 1
x
⇒ = =
Hint: Joining the given equations, we get –2 –5 1
80x + 80y = 240 x = – 2 and y = 5
or x + y= 3 ...(i) Hence, x = – 2, y = 5 is the solution of the
Subtracting given first equation from other given system of equations.
one, we get
8. To draw a line, we need atleast two
6x – 6y = – 6
solutions of its corresponding equations.
or x – y = –1 ...(ii)
x + 3y = 6; at x = 0, y = 2 and x = 3, y = 1.
Solving equations (i) and (ii), we obtain
So, two solutions of x + 3y = 6 are:
x = 1, y = 2.
5. x = 3, y = 2 x 0 3
1 1 y 2 1
Hint: Let = u, = v.
x+y x−y
2x – 3y = 12; at x = 0, y = –4 and at x = 6, y = 0
∴ Given equations become So, two solutions of 2x – 3y = 12 are:
10u + 2v = 4 and 15u – 5v = – 2.
x 0 6
6. False
Let us substitute c = 40, The given y –4 0
equations become
x – 2y = 8 Now, we draw the graph of given system
of equations by using their corresponding
or 5x – 10y = 40
solutions given in the above tables.

P A I R O F L I N E A R E Q U A T I O N S ... 29
2. (A) Condition for parallel lines is
a1 b c
= 1 ≠ 1
a2 b2 c2

1 –2 –3
⇒ = ≠ ⇒ k = – 6.
3 k –1
3. (D) As y = 2 and y = 7,
both represent straight lines parallel to
x-axis
∴ y = 2 and y = 7 are parallel lines.
Hence, the given pair of equations has no
solution.
4. The given lines to be coincident, if
k 3 – ( k – 3)
= =
From the, graph the two lines intersect the 12 k –k
y-axis at (0, 2) and (0, – 4). I II III
9. Let the fixed charges and change per km Taking I and II, we have
be Rs. x and Rs. y respectively. k2 = 36 ⇒ k = ± 6. ...(i)
x + 10y = 105 ...(i) Taking II and III, we have
x + 25y = 255 ...(ii) k2 – 3k = 3k ⇒ k(k – 6) = 0
Subtracting equation (i) from equation (ii), ⇒ k = 0 or 6 ... (ii)
we get Using (i) and (ii), we obtain
15y = 150 k = 6.
y = 10 ...(iii)
5. x = 5, y = 2
From equations (i) and (iii), we get
Hint: Adding the given equations,
x= 5
we get 2x + y = 12 ...(i)
Now, the fare for travelling a distance of
Subtracting the given equations,
35 km
we get 3x + y = 17 ...(ii)
= x + 35y
Now, (i) – (ii) ⇒ – x = – 5
= 5 + 35 × 10
x=5
= Rs. 355.
∴ from (ii ) ⇒ 3(5) + y = 17
Fixed charge = Rs. 5
y = 2.
Charge per km = Rs. 10
Total charge for 35 km = Rs. 355. 6. Yes.
Applying the condition
WORKSHEET–19 a1 b1 c1
= = c
a2 b2 2
1. (C) x – 5y = 5. We have
(2, k) lies on it. 1 2 –3
= =
3 6 –9
∴ 2 – 5(k) = 5 ⇒ 5(k) = – 3
That is true.
3 Therefore, the pair of equations is consis-
⇒ k= – .
5 tent with infinitely many solutions.

30 M A T H E M A T I C S – X
255 615 Equations (i) and (ii) from the required
7. x = , y= pair of linear equations. On solving these
52 104
Hint: The system: two equations, we will find starting salary
of x = Rs. 13000 and fixed increment of
9x – 10y + 15 = 0
y = Rs. 500.
5x + 6y – 60 = 0
By cross-multiplication, we have WORKSHEET–20
x −y 1
= = . 1. (B) As the lines are intersecting each other,
600 − 90 − 540 − 75 54 + 50
3 2 –3
8. For equation 3x + y – 2 = 0, ≠ ⇒ a≠ .
a –1 2
x 0 1 2. (D) Line x = a is parallel to y-axis and the
y 2 –1 line y = b is parallel to x-axis. These lines
intersect each other at (a, b).
For equation 2x – 3y – 5 = 0, 3. 3x – y – 5 = 0 and 6x – 2y – k = 0 have no
solution.
x –2 1 ⇒ These equations represent a piar of
parallel lines.
y –3 –1
3 –1 –5
⇒ = ≠
6 –2 k
⇒ k ≠ 10.
4. No.
For infinitely many solutions, the following
condition must be satisfied.
λ 3 7
= =
2 6 –14
3 –7 1 1
But, here ≠ as ≠ –
6 14 2 2
Hence, no value of ‘λ’ provides the pair of
infinitely many solutions.
5. The given system of equations can be
written as
ax + by – (a – b) = 0
bx – ay – (a + b) = 0
By cross-multiplication,
x –y
As the lines corresponding to the given =
equations intersect each other at (1, –1), – b( a + b) – a ( a – b) – a( a + b) + b ( a – b)
the required solution is x = 1, y = –1. (i) (ii )
9. Let the man's starting salary and fixed 1
increment be x and y respectively. – a2 – b2
According to the question, (iii )
x + 4 y = 15000 ...(i) ⇒ Taking (i) and (iii) simultaneously,
x + 10 y = 18000 ...(ii) we get x = 1 and y = – 1

P A I R O F L I N E A R E Q U A T I O N S ... 31
Hence x = 1, y = – 1 is the solution of the WORKSHEET–21
given system of equations.
6. x = 6, y = – 4, m = 0 1. (B) Hint: According to the condition of
infinitely many solutions, we reaches at
1 1
Hint: Take = u and = v. a+b 2a – b 21
x y = = .
7. No; (6, 0) , (4, 0) 2 3 7
Hint: For x + 3y = 6 2. (C) Hint: Simplifying the given linear
equations, we have
x 0 3 7 2 8 7
– = 5, + = 15
y y x y x
2 1
1 1
Now take = u, = v; and solve.
For 3x + 9y = 12 x y

x 1 4 3. (D) Let unit's and ten's digit be x and y


respectively.
y 1 0 x + y= 9 ...(i)
Let us draw the graph of lines using the 10y + x + 27 = 10x + y ...(ii)
tables obtained above. Solving equations (i) and (ii), we have
x = 6, y = 3
Hence, the required number is 3 × 10 + 6,
that is 36.
4. False
... Equations are 5x – 5y = 3 and
10x – 10y – 3 = 0
a1 5 1 b1 –5 1
∴ = = ; = = –
a2 10 2 b2 10 2
c1 3 a1 b1 c1
= =1 ∴ = ≠
c2 3 a2 b2 c2
⇒ lines are parallel.
In the graph, lines are parallel. So, the pair 5. p ≠ 6
of equations is not consistent.
3 5
The lines intersect the x–axis at (4, 0) and Hint: ≠ ⇒ p≠6
(6,0). p 10

8. Let the initial length be x and breadth be y. ∴ p can take any value but not 6.
Then according to question, 4a–b − a + 4b
xy – 9 = (x – 5) (y + 3) ...(i) 6. x = , y=
5a 5b
and xy + 67 = (x + 3) (y + 2) ...(ii)
Simplifying equations (i) and (ii), we have x
Hint:
3x – 5y = 6 ...(iii) – 3b (2 a + b) + 2b ( a + 2b )
2x + 3y = 61 ...(iv) –y
=
On solving (iii) and (iv), – 2 a (2 a + b ) + 3 a (a + 2b )
x = 17, y = 9
1
Hence, length of rectangle is 17 units and =
2 a × 2b – 3 a × 3b
that of breadth is 9 units.

32 M A T H E M A T I C S – X
Take first and third terms as well as second 9. Let son's present age be x years and father's
and third terms and solve. present age be y years.
7. a = 7, b = 3 Since, the father's age is 3 years more than
Hint: For infinitely many solutions, 3 times the son's age.
... y = 3x + 3
2 − (a − 4) 2b + 1
= = ⇒ 3x – y = 0 ...(i)
4 − ( a − 1) 5b − 1
3 years hence, father's age = (y + 3) years
1 a−4 and son's age = (x + 3) years.
Take = ⇒ a=7
2 a−1 Since, 3 years hence, the father's age will
1 2b + 1 be 10 years more than twice the son's age.
and = ⇒ b = 3. ... y + 3 = 2 × (x + 3) + 10
2 5b − 1
⇒ 2x + 6 + 10 – y – 3 = 0
8. Table for values of x and y as regarding
equation 3x + y – 5 = 0 is ⇒ 2x – y + 13 = 0 ...(ii)
Subtracting equation (ii) from equation (i),
x 0 1 we have
y 5 2 x = 10
Substituting x = 10 in equation (i), we have
Similarly table for equation 2x – y – 5 = 0 is y = 3 × 10 + 3 = 33.
x 0 1 Son's age = 10 years; father's age = 33 years.
OR
y –5 –3
Let the speed of train and bus be x km and
Let us draw the graph of lines using the y km respectively.
tables obtained above. According to the given conditions,
60 240
4= + ...(i)
x y
 Distance 
 Using Time = 
 Speed 

10 100 200
And 4 + = + ...(ii)
60 x y
1 1
Putting = u and y = v in (i) and (ii)
x
and simplifying, we get
15u + 60v = 1 ...(iii)
24u + 48v = 1 ...(iv)
On solving equations (iii) and (iv), we
obtain
1 1
u= and v =
60 80
i.e., x = 60 and y = 80
The lines intersect y-axis at (0, 5) and Hence, the speed of train is 60 km/hr and
(0, – 5). that of bus 80 km/hr.

P A I R O F L I N E A R E Q U A T I O N S ... 33
WORKSHEET–22 2 –3
On solving = and
p+q – ( p + q – 3)
1. (D) The condition to be coincident for lines
ax + by + c = 0 and dx + ey + f = 0 is given –3 –7
by = ,
– ( p + q – 3) – (4p + q)
a b c
= = we obtain p = – 5, q = – 1.
d e f
5. x = 1, y = 2
⇒ ae = bd ; bf = ce. Hint: Adding and subtracting the given
Note: Two lines are coincident if both the two equations, we have
equations follow the condition of infinitely x+y=3 ...(i)
many solutions. and x–y=–1 ...(ii)
Now, solve equations (i) and (ii).
2. (C) Let the required equation be ax + by + c
= 0. 6. x = a2, y = b2
a c Hint: Given system of linear equations may
6
Then, = ≠ be written as
2 – 3 –5
bx + ay – ab (a + b) = 0
a 6 b 2x + a2y – 2 a2 b 2 = 0
⇒ = = k (say)
2 – 3 Solve these two equations by the method
of cross-multiplication.
⇒a= 2 k, b = – 3 k, c any real number 7. Let the two digits number be 10x + y.
Then, 2 k – 3 k + c = 0 Since ten's digit exceeds twice the unit's
Putting c = –1, we have digit by 2
∴ x = 2y + 2
⇒ 2 kx – 3 ky – k = 0 ⇒ x – 2y – 2 = 0 ...(i)
⇒ 2x – 3 y = 1. Since the number obtained by inter-
changing the digits, i.e., 10y + x is 5 more
3. (A) For no solutions, than three times the sum of the digits.
k
=
3

–(k – 2) ... 10y + x = 3 (x + y) + 5
12 k –k ⇒ 2x – 7y + 5 = 0 ...(ii)
⇒ k= ± 6 On solving equations (i) and (ii), we obtain
if k= 6 x = 8 and y = 3
... 10x + y = 83
6 3 6–2 4 2 Hence, the required two digits number
∴ = ≠ = = True
12 6 6 6 3 is 83.
if k= –6 8. Tables for equations 3x + y – 11 = 0 and
x – y – 1 = 0 are respectively.
–6 3 –8 4
= ≠ = True
12 –6 –6 3 x 3 4
and
∴ Required value of k, can be 6 or – 6. y 2 –1
4. For infinite number of solutions, we have Let us draw the graph.
2 –3 –7 x 0 1
= =
p+q – ( p + q – 3) – (4 p + q)
y –1 3

34 M A T H E M A T I C S – X
Put x – y = u, x + y = v and solve further
find x and y.
OR
Let each boy receive Rs. x and the number
of boys be y. Then sum of money which is
distributed is Rs. xy.
Had there been 10 boys more, each would
have received a rupee less,
... (y + 10) (x – 1) = xy
⇒ 10x – y = 10 ...(i)
Had there been 15 boys fewer, each would
have received Rs 3 more,
... (y – 15) (x + 3) = xy
⇒ 5x – y = –15 ...(ii)
Solving (i) and (ii), we get
x = 5 and y = 40
From the graph, it is clear that the lines ... xy = 200
intersect each other at a point A(3, 2). So
the solution is x = 3, y = 2. Hence, sum of money = Rs. 200
And number of boys = 40.
The line 3x + y – 11 = 0 intersects the
y-axis at B(0, 11) and the line x – y – 1 = 0
intersects the y-axis at C (0, –1). Draw the
WORKSHEET–23
perpendicular AM from A on the y-axis
intersect it at M. 2 –3 9
1. (A) Here, = ≠
Now, in ∆ABC, 6 –9 –5
base BC = 11 + 1 = 12 units, height ... Lines are parallel.
AM = 3 units. 2. (B) As the given equation are homogeneous
1 so only solution will be x = 0 and y = 0.
... ar(∆ABC) = × base × height
2 3. (C)
1 1 1
= × 12 × 3 = 18 sq. units Hint: Put =u, = v and solve.
2 x y
x = 3, y = 2 ; Area = 18 sq. units.
4. The given equations have a unique solution
9. Speed of boat = 6 km/hr,
a b
Speed of stream = 2 km/hr ⇒ ≠ ⇒ am ≠ bl.
l m
Hint: Let the speed of boat in still water =
x km/h and the speed of stream = y km/h 5. The given equation can be written as
6ax + 6by = 3a + 2ab ...(i)
12 40
+ = 8 ...(i) and 6bx – 6ay = 3b – 2a ...(ii)
x–y x+y
Multiplying equation (i) by a and (ii) by b
 Distance  and adding the results, we have
 Using Time = 
 Speed  6(a2 + b2) = 3(a2 + b2)
16 32 1
+ = 16 ...(ii) ⇒ x=
x–y x+y 2

P A I R O F L I N E A R E Q U A T I O N S ... 35
1 OR
Substituting x = in equation (i), we have
2 Let fare from A to B and from A to C be
6a Rs. x and Rs. y respectively.
+ 6by = 3a + 2b
2 According to the given conditions,
1 2x + 3y = 795 ...(i)
⇒ 6by = 2b ⇒ y=
3 3x + 5y = 1300 ...(ii)
1 1 Solving eqn. (i) and (ii), we obtain
Thus, the solution is x = , y = .
2 3 x = 75, y = 215
6. a = 5, b = 1 Hence, fares from A to B is Rs. 75 and
Hint: Two linear equations a1 x + b1 y + c1 = 0 from A to C is Rs. 215.
and a2 x + b2 y + c2 = 0 have infinite number 9. Let us make the table for the values of x
of solutions if
and corresponding to the equation
a1 b c
= 1 = 1. 2x + y – 8 = 0
a2 b2 c2
7. Given system of linear equations can be x 2 4
written as y 4 0
(a + 2b) x + (2a – b) y – 2 = 0
(a – 2b) x + (2a + b) y – 3 = 0 Similarly, for the equation x – y – 1 = 0
By cross-multiplication,
x x 4 3
– 3 (2 a – b) + 2 (2 a + b) y 3 2
(i )
–y Let us draw the graph.
=
– 3 ( a + 2b) + 2 ( a – 2b)
(ii)
1
=
( a + 2b) (2a + b) – (2a – b) ( a – 2b)
(iii )
5b – 2a
Taking (i) and (iii), x =
10 ab
a + 10 b
Again taking (ii) and (iii), y =
10 ab
5b – 2a a + 10 b
Thus, x = , y = is the
10 ab 10 ab
solution of the given system of equations.
8. Speed of rowing = 6 km/hr,
Speed of current = 4 km/hr
Hint:
2 × (x + y) = 20
[Time × Speed = Distance]
2 × (x – y) = 4
Where, x = speed of rowing and,
y = speed of current.

36 M A T H E M A T I C S – X
From the graph, the lines intersect each – 12p – 12q = – 15p + 3
other at the point A(3,2). Therefore, the ⇒ 2p – 10q = 9
solution is x = 3, y = 2.
and 3p – 12q = 3
The lines intersect the y-axis at B(0,8) and
⇒ p = 5, q = 1.
C (0, – 1).
6. x = 1, y = 1,
To find the area of the shaded portion,
that is, ∆ABC, draw perpendicular AM 1 1
Hint: Take = u, =v
from A on the y-axis to intersect it in M. 3x + y 3x − y
Now, AM = 3 units and BC = 8 + 1 = 9 units. ∴ given equation can be written as:
1 3
... ar(∆ABC) = × BC × AM u+v= ⇒ 4u + 4v = 3
2 4
1 1 1 1
= ×9×3 and u– v=– ⇒ 4u – 4v = – 1.
2 2 2 8
27 −1 1
= sq. units 7. x = ,y=
2 2 3
x = 3, y = 2; Area = 13.5 sq.units.
1 1
Hint: Put = u and = v.
x y
WORKSHEET–24
8. Table for values of x and y corresponding
1. (C) For unique solution, to equation 4x – 5y – 20 = 0 is
4 p x 5 0
≠ ⇒ p ≠ 4.
2 2
y 0 –4
2. (A) In the case of no solution,
Similarly for the equation 3x + 5y – 15 = 0
3 –1 –5
= ≠ ⇒ k ≠ 10.
6 –2 –k x 5 0
3. (D) x = 80, y = 30 y 0 3
Hint: x + 2y = 140, 3x + 4y = 360.
Let us draw the graphs for the two equations.
4. True
According to the conditions of consistency,
either
2 2
3 ≠ – 5 or 3 = – 5 = 1
3 –5 3 –5 3
2 2
Clearly, the first condition holds. Hence,
the system of equations is consistent with
a unique solution.
5. For infinitely many solutions,
p + q 2( p – q) – (5p – 1)
= =
3 4 – 12
⇒ 4p + 4q = 6p – 6q and

P A I R O F L I N E A R E Q U A T I O N S ... 37
As the graphs of the two lines intersect WORKSHEET–25
each other at the point A(5, 0), the required
solution is x = 5, y = 0. 1. (C) The given equations represent to be
The graphs intersect the y-axis at B (0, 3) parallel lines if
and C(0, – 4). Therefore, the coordinates of 2(k – 1) 1 –1
vertices of the triangle ABC are A(5, 0), = ≠
B(0, 3) and C(0, – 4).
3 –1 –1
x = 5, y = 0 and (5, 0), (0, 3), (0, – 4). 3
⇒ k –1= –
9. Let speeds of two cars that start from places 2
A and B be x km/hr and y km/hr respec-
1
tively. ⇒ k= – .
2
Case I: When two cars travel in same direction.
2. (D) For the point of intersection of any line
Let the cars meet At C
with x-axis, put y = 0
... – 3x + 7 (0) = 3
⇒ x =–1
So the required point is (– 1, 0).
Distance travelled by the car that starts 3. (B) x – y = 0 ...(i)
from A 2x – y = 2 ...(ii)
AC = 5 × x – + –
Similarly distance for other car – x = –2 (Subtracting)
.
.. x = 2.
BC = 5 × y
.
.. AC – BC = 5x – 5y Further y = x = 2.
⇒ 5x – 5y = 100 4. For inconsistency,
⇒ x – y = 20 ...(i)
k+2 6 –(3k + 2)2
Case II: When two cars travel in opposite = ≠
2 3 –4
directions.
Let the cars meet at D ⇒ k + 2 = 4 and (3k + 2)2 ≠ 8
1
⇒ k = 2 and k ≠
3
(± 2 2 –2 )
Hence, k = 2.
5. m ≠ 4
Distance travelled by the car that starts m –2
from A Hint: ≠ .
2 –1
AD = 1 × x
6. Given system of equations can be written
Similarly distance for other car
as
BD = 1 × y 2x + 3y – 18 = 0 ...(i)
... AD + BD = x + y x – 2y – 2 = 0 ...(ii)
⇒ x + y = 100 ...(ii)
2 3
Solving equations (i) and (ii), we get Now, ≠
1 –2
x = 60 and y = 40
Hence the system has unique solution.
Hence, speeds of two cars that start from
Now, by cross-multiplication on (i) and (ii),
place A and B are 60 km/h and 40 km/h
we get
respectively.

38 M A T H E M A T I C S – X
x –y 1 To draw the graphs of the equations (i)
= = and (ii), we need atleast two solutions of
– 6 – 36 – 4 + 18 – 4 – 3
each of the equations. These solutions are
⇒ x = 6, y = 2 given below:
Thus, the solution of given system is
x = 6, y = 2. x 0 2

7. x = 5, y = –1 y = 4x – 8 –8 0
1 1
Hint: Take = u, = v and solve.
x+y x–y x 0 –3
2x + 6
8. Let Meena received x notes of Rs. 50 and y y 2 0
notes of Rs. 100 3
Since total number of notes is 25 Plot the points A(0, – 8), B(2, 0), C(0, 2)
... x + y = 25 ...(i) and D(– 3, 0) on graph paper and join them
Since the value of both types of notes is to form the lines AB and CD as shown in
Rs. 2000. figure.

... 50x + 100y = 2000


and x + 2y = 40 ...(ii)
Solving equations (i) and (ii), we get
x = 10, y = 15
Hence, Meena received 10 notes of Rs. 50
and 15 notes of Rs. 100.
OR
Let the length and breadth of rectangle be
x units and y units respectively.
Then area of rectangle = xy sq. units
Case I: The length is increased and breadth
is reduced by 2 units.
... (x + 2) (y – 2) = xy – 28
⇒ xy – 4 – 2 x + 2 y = xy – 28
⇒ x – y = 12 ...(i)
Case II: The length is reduced by 1 unit
and breadth increased by 2 units.
∴ (x – 1) (y + 2) = xy + 33
⇒ xy – 2 – y + 2x = xy + 33
2x – y = 35 ...(ii) The graphs of these lines intersect each
Solving equations (i) and (ii), we get other at P(3, 4). So, unique solution:
x = 23 and y = 11 x = 3, y = 4.
Hence, the length of the rectangle is 23 Also, the graphs meet the x-axis at D(– 3, 0)
units and the breadth is 11 units. and B(2, 0).
9. The given linear equations are Hence, the triangle formed by the lines
4x – y – 8 = 0 ...(i) is PBD with vertices P(3, 4), B(2, 0) and
and 2x – 3y + 6 = 0 ...(ii) D(– 3, 0).

P A I R O F L I N E A R E Q U A T I O N S ... 39
WORKSHEET–26 7. x = – 2, y = 5 and m = – 1
11 – 2x
1. (C) For coincident lines, Hint: 2x + 3y = 11 ⇒ y =
3
1 2 7 Substitute this value of y in 2x – 4y = – 24
= =
2 k 14 and solve for x.
⇒ k = 4. 8. The given system of linear equations is
2. (A) The given system of equations can be 2x – y – 5 = 0 ...(i)
written as 3x + y – 5 = 0 ...(ii)
x + 2y = 140, 3x + 4y = 360 To draw the graph of equations (i) and (ii),
we need atleast two solutions of each of
Solving this system, we obtain
the equations, which are given below:
x = 80, y = 30.
x 0 4
3. Adding the given equations, we have
3x = 0 ⇒ x = 0 y = 2x – 5 –5 3
Substituting x = 0 in any of the given
equations, we get y = 0 x 0 –3
Hence, the required solution is x = 0, y = 0.
–3x+ 5
4. False y= 5 –4
3
a1 2 b1 5 c1 Using these points, we are drawing the
Hint: As = , = , =6
a2 4 b2 10 c2 graphs of lines as shown below:
1 1
... = ≠6
2 2
⇒ They are parallel.
5
5. a = –1, b =
2
2 – (2a + 5) 5
Hint: = = .
2b + 1 –9 15
1 1
6. Put = u and = v in given system of
x y
equations.
u + v – 7= 0 ...(i)
2u + 3v – 17 = 0 ...(ii)
By cross-multiplication,
u –v 1
= =
– 17 + 21 – 17 + 14 3–2
⇒ u = 4, v = 3 From the graph, the lines intersect each
1 1 other at the point P(2, –1). Therefore, the
⇒x= ,y= solutions is x = 2, y = –1.
4 3
The lines meet the y-axis at the points
1 1 Q(0, 5) and R(0, –5).
Hence, x = ,y= is the solution of the
4 3
9. Let the fixed charge and additional charge
given system of equations.
for each day be Rs. x and Rs. y respectively.

40 M A T H E M A T I C S – X
Since saritha paid Rs. 27 for a book kept To solve the system for x and y, using the
for 7 days method of cross-multiplication, we have
... x + 4y = 27 ...(i) x
=
–y
Also, Susy paid Rs. 21 for the book kept – q( p + q) – p( p – q ) – p( p + q) + q( p – q)
for 5 days 1
... x + 2y = 21 ...(ii) =
– p2 – q 2
Subtracting equation (ii) from (i), we get
2y = 6 ⇒ y = 3 x –y 1
⇒ 22
= =
Again substituting y = 3 in equation (ii), –p –q 2
–p – q 2 – p – q2
2

we get ⇒ x = 1, y = –1.
x = 15
8. The given system of equations can be
Hence, the fixed charge is Rs. 15 and
written as
additional charge for each day is Rs 3.
3x – 4y – 1 = 0 ...(i)
OR
Son's age = 10 years, father's age = 40 years. 6x – 15y = 0 ...(ii)
Hint: Let the present age of father and son To draw the graph, we need atleast two
be x and y years respectively. solutions for each of the equations (i) and
... x + 5 = 3 (y + 5) (ii), which are given below:
And x – 5 = 7 (y – 5).
x 3 7
WORKSHEET–27 – 3x – 1
y= 2 5
4
1. (A) Hint: In a cyclic quadrilateral,
x 3 11
∠A + ∠C = 180° and ∠B + ∠D = 180°. 2 2
2. (C) Hint: Both lines are passing through 6x + 15
y= 3 6
the origin. 8
3. (A) For infinite number of solutions, Let us draw the graph to use these points.
p+q 2 p – q –21
= =
2 3 –7
⇒ p + q = 6 and 2p – q = 9
p = 5 , q = 1.
4. False, as a + 5b = – 10.
5. False, x = 4, y = 1 does not satisfy the
second equation.
6. No solution
Hint: 2x + 3y = 7, 6x + 9y = 11
2 3 7
Here, = ≠ Parallel lines.
6 9 11
7. The given system of linear equations can
be written as
From the graph, it is clear that the lines are
px + qy – (p – q) = 0 parallel. Hence, the given system of
qx – py – (p + q) = 0 equations is inconsistent.

P A I R O F L I N E A R E Q U A T I O N S ... 41
x
9. Let the fraction be p2 + 1 p–2 5
y = ≠
3p + 1 3 2
On adding 1 to each of numerator and
⇒ 5p = – 5 ⇒ p = – 1.
6
denominator, the fraction becomes
5 4. (A) Multiplying first equation by 2 and
the other one by 3 and adding, we get
... x +1 6
= 1
y +1 5 21.8x = 10.9 ⇒ x =
2
⇒ 5x + 5 = 6y + 6
1
⇒ 5x – 6y = 1 ...(i) Substituting x = in any of the given
2
Further, on subtracting 5 from each of 1
numerator and denominator, the fraction equations, we have y = .
3
3 1 1
becomes ∴ x= , y = .
2 3
2
x–5 3
... = 5. True
y–5 2 The condition for parallel lines is
⇒ 2x – 10 = 3y – 15 2 –2 –3
⇒ 2x – 3y = – 5 ...(ii) = ≠
6 –6 5
Solving equations (i) and (ii), we get 1 1 –3
x = 11, y = 9 ⇒ = ≠
3 3 5
11 The condition holds. The lines are parallel.
Hence, the required fraction is .
9 6. x = a2 , y = b2
OR 1 1
Hint: Put = u and = v.
Rs. 6000, Rs. 5250 x y
Hint: Let incomes of x and y be 8x and 7x 7. Given system of linear equations can be
respectively; and expenditures of them be written as:
19y and 16y respectively. (a – b) x + (a + b) y – (a2 – 2ab – b2) = 0
∴ 8x – 19y = 1250 ...(i) (a + b) x + (a + b) y – (a2 + b2) = 0
7x – 16y = 1250 ...(ii) By cross - multiplication,
x
WORKSHEET–28 – ( a + b )(a 2 + b 2 ) + ( a + b) ( a2 – 2 ab – b 2 )
–y
1. (A) =
– ( a – b)( a + b ) + ( a + b) ( a2 – 2ab – b2 )
2 2
k–3 3 k
Hint: = = .
k k 12 1
=
( a – b ) ( a + b) – ( a + b ) ( a + b)
2. (C) The condition that the given system of
equations has unique solution is represen- x –y 1
⇒ 2
= 2
=
ted by – 2b ( a + b) – 4 ab – 2b ( a + b)
a1 b1 Hence, the solution of given system of
≠ .
a2 b2 equations is
3. (B) The condition that the given system of 2ab
x = a + b, y = – .
equations represents parallel lines is a+b

42 M A T H E M A T I C S – X
8. To draw graph of the equation, we need 1
atleast two solutions. area of ∆ ABC = × base × height
2
Two solutions of the equation
1
4x + 3y – 24 = 0 are mentioned in the = × 10 × 8 = 40 sq. unit.
following table: 2
9. Rs. 600, Rs. 700
x 0 6
Hint: Let cost price of trouser be Rs. x and
y 8 0 that of shirt Rs. y. Then

Similarly, two solutions of each of the 125 110 


x+ y = 1520 


equations 2x – y – 2 = 0 and y + 4 = 0 are 100 100 
respectively. 
110 125 
x+ y = 1535 

100 100 

x 0 1
and ⇒ 25 x + 22 y = 30400 
y –2 0 


22 x + 25 y = 30700 


x 0 4 OR
y –4 –4 6 l of 50% and 4 l of 25%.
Hint: Let x litres of 50% acid and y litres of
Using the tables obtained above, let us
25% acid should be mixed.
draw the graph.
50 25 40 
x+ y= ( x + y) 


100 100 100 


x + y = 10 

2x = 3y 


⇒ 
x + y = 10 

WORKSHEET–29
1. (C) x = 9, y = 6
Hint: x – y = 3 and 2x + 3y = 36.
3p 6 50
2. (D) = ≠
18 24 75
p 3 2
⇒ = ≠
2 2 3
∴ p= 3.
Observing the graph, the lines meet each 3. (A) Solving 3x – 2y = 4 and 2x + y = 5, we
of her pairwise in A(3, 4), B(– 1, – 4) and have x = 2, y = 1.
C(9, – 4).
Now, substituting these values of x and y
Hence, the vertices of the triangle ABC
in y = 2x + m, we have 1 = 2 × 2 + m
so obtained are A(3, 4), B(–1, – 4) and
C(9, – 4). ∴ m = – 3.

P A I R O F L I N E A R E Q U A T I O N S ... 43
4. x = b, y = – a ⇒ 5x + 6y = 6700 ...(ii)
a2 b2
Hint: x – y = ab (a + b), ax – by = 2ab Adding and subtracting (i) and (ii), we get
Solving the equation we get x = b1, y = – a. x + y = 1200 ...(iii)
5. For inconsistency, x – y = – 200 ...(iv)
α 3 α–3 9. Two solutions of 6y = 5x + 10 are:
= ≠ ⇒ α2 = 36 and
12 α α x –2 4
3α ≠ α2 – 3α
y 0 5
⇒ α = ± 6 and α ≠ 0 or α ≠ 6 ⇒ α = –6.
Two solutions of y = 5x – 15 are
22a −26b
6. x = ,y=
5 5 x 3 2
Hint: 4bx + 3ay – 2ab = 0 y 0 –5
3bx + ay – 8ab = 0.
Now, we draw the graph of the system on
7. 3x + 2y = 800. the same coordinate areas.
12x + 8y = 3000; Not possible
Hint: Let cost of 1 chair be Rs. x and that
of 1 table be Rs. y.
∴3x + 2y = 800, 12x + 8y = 3000.
8. Let the actual prices of tea-set and lemon-
set be Rs. x and Rs. y respectively
According to the question,
Case I: Selling price – cost price = Profit
⇒ 0.95x + 1.15y – (x + y) = 7
⇒ – 0.05x + 0.15y = 7 ...(i)
Case II: Selling price – Cost price = Profit
⇒ 1.05x + 1.10y – (x + y) = 13
⇒ 0.05x + 0.10y = 13 ...(ii)
Solving equations (i) and (ii), we get
x = 100 , y = 80
Hence, actual prices of tea-set and lemon- (i) From the graph, we look that the two
set are Rs. 100 and Rs. 80 respectively. lines intersect each other at A(4, 5).
OR (ii) The vertices of the triangle : A (4, 5);
The person invested Rs. 500 at the rate of B (–2, 0); C (3, 0).
12% per year and Rs. 700 at the rate of Height of ∆ABC corresponding to the
10% per year. base BC,
Hint: Let the person invested Rs. x at the h = 5 units
rate of 12% per year and Rs. y at the rate and base, b = BC = 5 units
of 10 % per year
1
12x 10 y Now, ar(∆ABC) = ×b×h
∴ + = 130 2
100 100
⇒ 6x + 5y = 6500 ...(i) 1
= ×5×5
2
12y 10x
and + = 134 = 12.5 square units.
100 100

44 M A T H E M A T I C S – X
ASSESSMENT SHEET–5 Solving equations (iii) and (v), we have
3 1
1. (C) Let us check option (C). x= ;y= .
2 2
2 5 2 5
x + y = (– 3) + (– 2) = – 2 – 5 = – 7 3
3 2 3 2 Therefore, xy =
4
1 1
3x – y = 3 (– 3) – (– 2) = – 9 + 1 = – 8. Solving equations (iv) and (vi), we have
2 2
1 3
2. (D) 4x – y = 42 ⇒ x– y=2 ...(i) x= – ;y=
4 4
x – 2y = 8 ...(ii)
– + – 3
Therefore, xy = – .
y= –6 16
∴ x + y = – 10 3 3
Hence, xy = or – .
∴ from (i) ⇒ x = – 4. 4 16

3. For coincident lines 6. Given equations can be written as


a1 b c x y
= 1 = 1 + – (a + b) 2 = 0
a2 b2 c2 a b
2 3 7 x y
+ 2 – 2= 0
⇒ = = a2 b
a+b a+b–3 4a + b
⇒ a – 5b = 0. Let us apply cross-multiplication method
to solve these equations.
4. False, because the given pair of equations
has infinitely many solutions at k = 40 only. x –y 1
= =
5. Given equations are 2 a 1 2 1 b 1 1
– + 2 + – + + 2 2
– 2
2y – x. (x + y) = 1 b b b a a a ab ba
1
⇒ x + y= ...(i) b2 x – a2 y a2 b2
2 y–x ⇒ = =
–b+a –a+b a–b
and (x + y)x – y = 2 ...(ii)
Substituting the value of x + y from b2 x a2 b2
equation (i) in equation (ii), we get Taking =
–b+a a– b
x–y
 1 
 y–x  =2 – a2 y a2 b2
2  and =
–a + b a–b
⇒ (2x – y)x – y = 21
⇒ (x – y)2 = 1 a 2 b 2 ( a – b) ( a – b) a2 b 2
⇒ x–y=±1 ⇒ x= and y =
b 2 ( a – b) a2 ( a – b)
⇒ x–y=1 ...(iii)
or x – y = –1 ...(iv) ⇒ x = a2 and y = b2.
Substituting x – y = 1 and x – y = –1 in 7. Given equations of lines are:
equation (ii), we get respectively 3x + y + 4 = 0 ...(i)
x+y=2 ...(v) and 6x – 2y + 4 = 0 ...(ii)
1 To draw the graphs of lines (i) and (ii), we
and x+y= ...(vi) need atleast two solutions of each equation.
2

P A I R O F L I N E A R E Q U A T I O N S ... 45
For equation (i), two solutions are: 10
Sale price of the sweater = y – y ×
100
x 0 –3
90
= y
y –4 5 100
108 90
For equation (ii), two solutions are: ∴ x+ y = 1008
100 100
x 0 2 ⇒ 108x + 90y = 100800 ...(i)
y 2 8 Case II:
10
Let us draw the graphs of the lines (i) Sale price of the saree = x + x ×
100
and (ii)
110 x
x y =
+ – 2= 0 100
a b 8
Sale price of the sweater = y – y ×
100
92
= y
100
110 92
∴ x+ y = 1028
100 100
⇒ 110x + 92y = 102800 ...(ii)
Adding equations (i) and (ii), we get
218x + 182y = 203600 ...(iii)
Subtracting equation (i) from (ii), we get
2x + 2y = 2000
or 218x + 218y = 218000 ...(iv)
(Multiplying by 109)
Solving equation (iii) and (iv), we have
x = 600 and y = 400
Hence, the cost price of the saree is
Rs. 600 and the list price of the sweater is
Rs. 400.

From the graph it is clear that the two lines ASSESSMENT SHEET–6
intersect each other at a point, P (–1, – 1),
1. (B) For no solution,
therefore, the pair of equations consistent.
3 7 k
The solution is x = –1, y = – 1. = ≠
12 2k 4k + 1
8. Let the cost price of the saree and the list 3 7
price of the sweater be Rs. x and Rs. y ∴ = ⇒ k = 14.
12 2k
respectively. Now two cases arise.
Case I: 2. (B) For infinitely many solutions,
8 13 k k
Sale price of the saree = x + x × = =
100 39 6 k+4
108 13 k
= x Taking = ⇒ k = 2.
100 39 6

46 M A T H E M A T I C S – X
3. x = – a and y = k must satisfy both the given and x – y= 2 ...(iv)
equations. Let us substitute these values of Adding equations (iii) and (iv); and
x and y in
subtracting equation (iv) from (iii), we get
bx – ay + 2ab = 0 respectively.
b (–a) – ak + 2ab = 0 x = 1 and y = – 1.
⇒ –ak + ab = 0
7. One of the given equation is
⇒ k = b.
2x + y = 14 ...(i)
4. Yes, because
Here, at x = 0, y = 4
a1 b c 1 2 –3
= 1 = 1 , i.e., = = . and at x = 2, y = 0
a2 b2 c 2 3 6 –9
Two solutions of equation(i) are given in
5. Given equations are: the following table:
4x + 5y = 2
(5p + 2q)x + (4p – 2q)y = p + q + 3 x 0 2
Here, a1 = 4, b1 = 5, c1 = – 2, y 4 0
a2 = 5p + 2q, b2 = 4p – 2q,
Another given equation is
c2 = –p –q –3
2x – y = 4 ...(ii)
For infinitely many solutions,
c Two solutions of equation (ii) are given by
a1 b
= 1 = 1 the following table:
a2 b2 c2
4 5 –2 x 0 2
⇒ = =
5 p + 2q 4 p – 2q –p – q – 3 y –4 0
4 5 Let us draw the graph of the two equations
Taking =
5 p + 2q 4 p + 2q (i) and (ii) by using their corresponding
⇒ 25p + 10q = 16p – 8q tables.
⇒ 9p = – 18q
⇒ p = – 2q ...(i)
Also, taking
4 –2
=
5 p + 2q –p – q – 3
⇒ –10p – 4q = – 4p – 4q –12
⇒ p= 2
Substitute p = 2 in equation p = – 2q to get
q = – 1.
Hence, p = 2, q = – 1.
6. Given system of equations is
43x + 67y = –24 ...(i)
67x + 43y = 24 ...(ii)
Adding (i) and (ii); and subtracting (i) from
(ii), we get respectively.
110x + 110y = 0
and 24x – 24y = 48
i.e. x + y= 0 ...(iii)

P A I R O F L I N E A R E Q U A T I O N S ... 47
From the graph, vertices of the triangle and y + 4 = 4(x + 4)
ABC are A(0,4), B(0, – 4) and C(2, 0). Solve yourself.
1 3. (C) The lines are coincident
ar(∆ABC) = × base × height
2
3 –1 8
1 ⇒ = = ⇒ k = 2.
= × AB × OC 6 –k 16
2
1 4. Yes,
= × (4 + 4) × 2 For consistency,
2
= 8 square units. 2a b 2a b –a
Either ≠ or = =
Thus, area of the triangle is 8 square units. 4a 2b 4a 2b –2b
8. Let Anand had a total of x oranges; and he
2a b –a
makes lot A of p oranges and lot B of Here the relation = = ,
remaining x – p oranges. There are two 4a 2b –2 a
cases now. 1 1 1
Case I: i.e., = = holds.
2 2 2
2 ⇒ The pair is consistent.
Selling price of lot A = Rs. p
3
5. 21x + 47y = 110
Selling price of lot B = Rs. (x – p)
47x + 21y = 162
∴ 2 p + x – p = 400
+ + +
3
68x + 68y = 272
⇒ 3x – p = 1200 ...(i)
⇒ x + y= 4 ...(i)
Case II:
Selling price of lot A = Rs. p Subtracting the given equation, we get
4 – 26x + 26y = – 52
Selling price of lot B = Rs. (x – p)
5 ⇒ x – y= 2 ...(ii)
4 Solve equation (i) and (ii) and we get
∴ p + (x – p) = 460
5 x = 3, y = 1.
4x + p = 2300 ...(ii)
6. We are given
Add equations (i) and (ii) to get
7x = 3500 2xy 3
⇒ x = 500 x+y = 2 ...(i)
Hence Anand had a total number of 500
xy –3
oranges. and 2 x – y = 10 ...(ii)

CHAPTER TEST Taking equation (i)


1. (C) A pair of linear equations is said to 2xy 3
consistent, if the lines intersect each other x+y = 2
at a point or coincide.
⇒ 3x + 3y = 4xy ...(iii)
2. (C) 6, 36
Now, Taking equation (ii)
Hint: Let the son's age = x,
xy –3
father's age = y
2 x – y = 10
∵ y = 6x

48 M A T H E M A T I C S – X
⇒ – 6x + 3y = 10xy ...(iv) 10
Multiplying equation (iii) by 2 and adding ∴ Selling price of a sweater = y – y
100
its with (iv), we get
9y = 18xy 90
= y.
100
1
∴ x= According to condition, we have
2
1 108 x 90 y
Putting x = in equation (iv), we get + = 1008
2 100 100

⇒ – 3 + 3y = 5xy 108x + 90y = 100800


6x + 5y = 5600 ...(i)
–3
∴ y= If profit on a saree is 10% and discount on
2
a table is 8%, then total selling price is
1 –3 Rs. 1028.
This, x= and y = .
2 2 10 x   8y 

7. The given system of equations will have ⇒ x +  + y –  = 1028
 100   100 
infinite number of solutions if
110 x 92 y
1 2 1 ⇒ + = 1028
= = 100 100
a–b a+b a+b–2
110x + 92y = 102800
1 1 55x + 46y = 51400 ...(ii)
⇒ = and
a–b a+b–2 Multiplying eqn. (i) by 55 and eqn. (ii) by
6, we get
2 1
= 330x + 275y = 308000 ...(iii)
a+b a+b–2
330x + 276y = 308400 ...(iv)
⇒ a + b – 2 = a – b and
Subtract eqn. (iv) from eqn. (iii), we get
2a + 2b – 4 = a + b
y = 400
⇒ a + b – a + b = 2 and a + b = 4
the value of y, put in eqn. (i), we have
⇒ b = 1 and a = 3
x = 600
Hence, the given system of equations will
Cost price of saree = Rs. 600
have infinite number of solutions, if
List price of sweater = Rs. 400.
a = 3, b = 1.
9. To draw the graph of a line, we are
8. Let the cost price of saree be Rs. x and list
required atleast two solutions of its
price of sweater be Rs. y. corresponding equation.
Profit on a saree = 8% At x = 0, 5x – y = 5 gives y = –5
8 At x = 1, 5x – y = 5 gives y = 0
∴ Selling price of a saree = x + x
100 Thus, two solutions of 5x – y = 5 are given
in the following table:
108 x
=
100 x 0 1
Discount on a sweater = 10% y –5 0

P A I R O F L I N E A R E Q U A T I O N S ... 49
x 9 1
y –4 0

x + 2y = 1

x 2 4
y 5 –7
6x + y = 17
Now, we will draw the graphs of the three
lines on the same coordinate axes.

From the graph, it is clear that the lines


form a triangle ABC with vertices A(1, 0),
Similarly, we can find the solutions of each B(3, –1) and C(2, 5).
remaining equation as given in the
following tables: ❑❑

50 M A T H E M A T I C S – X
Chapter

4 TRIANGLES

WORKSHEET–33 QR QT
⇒ = [Using (i)]
1. (A) ∵ ∆ABC ~ ∆DEF QS PQ
and ∠1 = ∠1 (Common)
AB2 BC2 AC2 ar (∆ABC)
∴ 2
= 2 = 2 = ∴ ∆PQS ~ ∆TQR (SAS criterion)
DE EF DF ar (∆DEF)
Hence proved.
BC2 ar (∆ABC) 6. 1 : 4
Taking 2
= A
EF ar (∆DEF) 1
Hint: ar(∆DEF) = ar ∆ABC
4
BC2 64
⇒ 2
= ar(∆ DEF) 1 D F
(15.4) 121 ⇒ = .
ar(∆ ABC) 4
64 × 15.4 × 15.4 B C
⇒ BC = 7. 13 m E
121
Hint: Distance between tops = AD
⇒ BC = 11.2 cm.
∴ AD = AE2 + DE2
2. (D) Observing the figure, we obtain
∠A = ∠R, ∠B = ∠Q, ∠C = ∠P = (5)2 + (12)2
∴ ∆ABC ~ ∆RQP.
= 13 m.
3. Yes.
Here, 262 = 242 + 102 = 676 8. Hint: Use Pythagoras Theorem.
⇒ AC2 = AB2 + BC2 9. Statement: If a line is drawn parallel to
∴ ∆ABC is a right triangle. one side of a triangle to intersect the other
4. In ∆ABC, LM || CB two sides at distinct points, the other two
AM AL sides are divided in the same ratio.
∴ = …(i) Proof: ABC is a given triangle in which
AB AC
DE || BC. DE intersects AB and AC at D
(Basic Proportionality Theorem)
and E respectively.
Similarly in ∆ADC,

∴ AN = AL …(ii)
AD AC
Comparing equations (i) and (ii), we have
AM AN
= . Hence proved.
AB AD
We have to prove
5. As ∠1 = ∠2
⇒ PQ = PR …(i) AD AE
=
BD CE
QR QT
= (Given) Let us draw EM ⊥ AB and DN ⊥ AC. Join
QS PR
BE and CD.

T R I A N G L E S 51
1 2
Now, ar(∆ADE) = × base × height  AB 
2 2. (D) DC2 = BC2 + BD2 = BC2 +  
 2 
1
= × AD × EM ...(i) 1
2 = BC2 + (AC2 – BC2)
4
1
Also, ar(∆ADE) = × AE × DN ...(ii) 1
2 =9+ (25 – 9) = 9 + 4 = 13
4
1
ar(∆BDE) = × BD × EM ...(iii) ⇒ DC =
2 13 cm.

1 3. (C)
ar(∆CDE) = × CE × DN ...(iv)
2 Hint: As DE || BC
Dividing equation (i) by equation (iii) and
AD AE
equation (ii) by equation (iv), we have ∴ =
DB EC
ar (∆ADE) AD
= ...(v) 2x − 1 2x + 5
ar(∆BDE) BD ⇒ = .
x − 3 x −1
ar(∆ADE) AE
and = ...(vi) 4. DE || BC and DB is transversal
ar(∆CDE) CE
⇒ ∠EDA = ∠ABC
But ar(∆BDE) = ar(∆CDE) ...(vii)
(Alternate interior angles)
(Triangles are on the same base and
Similarly, ∠AED = ∠ACB
between the same parallels BC and DE)
Comparing equations (v), (vi) and (vii), we Consequently, ∆ADE ~ ∠ACB
have (AA similarity)
AD AE AD 2 ar (∆ADE)
= . ∴ =
BD CE AB 2 ar (∆ABC)
2nd Part:
As ∠B = ∠C AD 2 ar (∆ADE)
⇒ AB = AC ⇒ 2
=
9 AD 153
⇒ AD + DB = AE + EC
⇒ AD = AE (... BD = EC) ⇒ ar(∆ADE) = 17cm2.
AD AE 5. No.
∴ =
DB EC DP 5 1
Here, = =
∴ By converse of Basic Proportionality PE 10 2
Theorem, DE || BC. DQ 6 1
Hence proved. And = =
QF 18 3

WORKSHEET– 34 DP DQ
∵ ≠
PE QF
1. (A) ∆BAC ~ ∆ADC Therefore, PQ is not parallel to EF.
BC AC
⇒ = ⇒ y2 = 16 × 4 ⇒ y = 8 cm. 6. Hint: Use Basic Proportionality Theorem.
AC DC

52 M A T H E M A T I C S – X
7. As AB = BC = AC A
Let us construct a ∆PQR with ∠Q = 90°
1 such that
∴ AD ⊥ BC ⇒ BD = BC PQ =A′B′ and QR + B′C′ ...(ii)
2
∴ Using Pythagoras Theorem
AB2 = AD2 + BD2 B C
D
2
1 
⇒ AD2 = AB2 −  BC 
2 
3AB2
=
4
⇒ 4AD2 = 3AB2. Hence proved In ∆PQR,
PR2 = PQ2 + QR2
OR
(Pythagoras Theorem)
Let ABCD be a rhombus =A′B′2 + B′C′2 ...(iii)
Since, diagonals of a rhombus bisect each
[From (ii)]
other at right angles, 2 2 2
But A′C′ =A′B′ + B′C′ ...(iv)
∴ AO = CO, BO = DO,
∠AOD = ∠DOC [From (i)]
∠COB = ∠BOA = 90° From equations (iii) and (iv), we have
Now, in ∆AOD PR2 = A′C′2
AD2 = AO2 + OD2 ...(i) ⇒ PR = A′C′ ...(v)
Similarly, DC2 = DO2 + OC2 ...(ii) Now, in ∆ABC and ∆PQR,
CB2 = CO2 + BO2 ...(iii) A′B′ = PQ [From (ii)]
and BA2 = BO2 + AO2 ...(iv) B′C′ = QR [From (ii)]
Adding equations (i), (ii), (iii) and (iv), A′C′ = PR [From (v)]
we have Therefore, ∆A′B′C′ ≅ ∆PQR
AD2 + DC2 + CB2 + BA2 (SSS congruence rule)
= 2(DO2 + CO2 + BO2 + AO2) ⇒ ∠B′ = ∠Q (CPCT)
 BD 2 AC2 BD 2 CA 2  But ∠Q = 90°
= 2 4 + 4 + 4 + 4  ... ∠B′ = 90°. Hence proved.
 
2nd Part
= BD2 + CA2. Hence proved
In ∆ADC, ∠D = 90°
1
8. Hint: BD = DE = EC = BC ... AC2 = AD2 + DC2 = 62 + 82
3
Using Pythagoras Theorem. = 36 + 64 = 100
In ∆ABC,
9. Statement: In a triangle, if square of the AB2 + AC2 = 242 + 100 = 676
largest side is equal to the sum of the
and BC2 = 262 = 676
squares of the other two sides, then the
Clearly, BC2 = AB2 + AC2
angle opposite to the largest side is a right
angle. Hence, by converse of Pythagoras Theorem,
Proof: We are given a triangle ABC with in ∆ABC,
A′C′2 =A′B′2 + B′C′2 ...(i) ∠BAC = 90°
We have to prove that ∠B = 90° ⇒ ∆ABC is a right triangle.

T R I A N G L E S 53
WORKSHEET– 35 7. In ∆ABC and ∆AMP
∠A = ∠A (Common)
ar(∆ ADE) DE2 ∠ABC = ∠AMP = 90°
1. (B) =
ar (∆ ABC) BC2 (i) ∴ ∆ABC ∼ ∆AMP, (AA criterion)
2 CA BC
 2 BC  (ii) ∴ = .
  PA MP
3 
⇒ ar(∆ADE) = × 81 = 36 cm2. (∴ Corresponding sides of
BC2
similar triangles are proportional.)
2. (A) ∆OAB ~ ∆OCD 8. Hint:
OA OB ar(∆ AXY) = ar(BXYC) + ar(BXYC)
⇒ = ⇒ 2. ar(∆AXY) = ar(BXYC) + ar(∆ AXY)
OC OD
= ar(∆ABC) A
3
⇒ OB = 4 × = 6 cm. ar (∆ABC) 2
2 ⇒ = X Y
ar (∆ AXY) 1
3. In ∆ABC, To make DE || AB, we have to As ∆ABC ~ ∆ AXY
take 2
B C
 AB  ar ( ∆ABC) 2
AD BE 3 x + 19 3x + 4 ∴   = ar( ∆AXY) =
⇒ = ⇒ =  AX  1
DC EC x+3 x
⇒ 3x2 + 19x = 3x2 + 4x + 9x + 12 AB 2 BX 2 −1
⇒ = ⇒ = .
⇒ 6x = 12 ⇒ x = 2. AX 1 AB 2
4. No, 9. Hint: Prove converse of Pythagoras
∵ ∆FED ~ ∆STU Theorem.
Corresponding sides of the similar triangles
are in equal ratio. WORKSHEET– 36
DE EF 1. (A)
∴ =
TU ST In triangle ABC, A

DE EF AD 6 2
∴ ≠ . = = D
ST TU DC 21 7
AP BQ BE 18 – 14 2
5. AB || PQ ⇒ = …(i) = = C
AO BO EC 14 7 B E

AP CR AD BE
AC || PR ⇒ = …(ii) ∴ = ⇒ DE || AB .
AO CO DC EC
BQ CR
From (i) and (ii) = ar (∆DEF) EF 2
BO CO 2. (A) =
⇒ BC || QR. (By converse of BPT) ar (∆ABC) BC2

6. 1 : 2. 16
⇒ ar(∆DEF) = 54 × = 96 cm2.
Hint: Let AB = BC = a 9

... AC = 2 a 3. (B) ∆ABD ~ ∆BCD


AB BD
ar( ∆ABE) AB2 ⇒ =
... = . BC CD
ar( ∆ACD) AC2

54 M A T H E M A T I C S – X
5.4 3.6 So, by AA rule of similarity,
⇒ = ∆AQO ~ ∆BPO
BC 5.2
AQ AO
5.4×5.2 ⇒ =
⇒ BC = BP BO
3.6
= 7.8 cm. AQ 10 10 × 9
⇒ = ⇒ AQ =
4. In ∆ABC and ∆ADE, 9 6 6
⇒ AQ = 15 cm.
∠BAC = ∠DAE (Common angle)
OR
∠ACB = ∠AED (Each 90°)
Let the height of the tower be h metres
∴ ∆ABC ~ ∆AED (AA criterion) P

AB = AC 2 + BC 2 = 25 + 144 = 13 cm A

x
AB BC AC 12m
Now, = =
AD DE AE
B C
8m Q 40m R
13 12 5
⇒ = = ∆ABC ~ ∆PQR.
3 DE AE
AB BC 12 8
⇒ = ⇒ =
36 15 PQ QR h 40
⇒ DE = cm and AE = cm.
13 13
12 × 40
⇒ h= = 60 metres.
5. No. 8
Ratio of areas of two similar triangles 8. Hint: As ∆AOB ~ ∆COD
= square of ratio of their corresponding A B
altitudes.
2
3 9 6 O
=  = ≠ .
5 25 5
Hence, it is not correct to say that ratio of D C

areas of the triangles is


6
.
ar(∆AOB)
=
AB
=
( 2 CD )
2 2

=
4
.
5 ar(∆COD) CD 2
CD 2 1
6. AE2 = AC2 + EC2 …(i) 9. Hint: Prove Pythagoras Theorem.
BD2 = DC2 + BC2 …(ii) For 2nd Part:
∴ AB2 = AD2 + BD2 …(i)
Adding (i) and (ii), we get Also AC2 = AD2 + CD2 …(ii)
AE2 + BD2 = AC2 + EC2 + DC2 + BC2 From (i) and (ii),
⇒ AB2 − AC2 = BD2 − CD2
= (AC2 + BC2) + (EC2 + DC2)
⇒ AB2 + CD2 = AC2 + BD2.
AE2 + BD2 = AB2 + DE2. Hence proved.
Hence proved.
WORKSHEET–37
7. In ∆AQO and ∆BPO,
∠QAO = ∠PBO (Each 90°) 1. (A) ∠M = 180° – (∠L + ∠N) (ASP)
∠AOQ = ∠BOP = 180° – (50° + 60°) = 70°
∵ ∆LMN ~ ∆PQR
(Vertical opposite angles)
∴ ∠ M = ∠Q ⇒ ∠Q = 70°.

T R I A N G L E S 55
2. (C) In ∆KMN, as PQ || MN,
KP KQ
=
PM QN
KP KQ
⇒ =
PM KN – KQ
KN PM
⇒ –1 = In right ∆AOE,
KQ KP
AO2 = AE2 + OE2 ...(ii)
20.4 13
⇒ –1 = From equations (i) and (ii), we have
KQ 4
AF2 + OF2 = AE2 + OE2 ...(iii)
20.4 13 17 Similarly, we can find out that
⇒ =1+ =
KQ 4 4 BD2 + OD2 = BF2 + OF2 ...(iv)
20.4× 4 and CE2 + OE2 = CD2 + OD2 ...(v)
⇒ KQ =
17 Adding equations (iii), (iv) and (v), we
⇒ KQ = 4.8 cm. arrive
3. (A) ∆ABC ~ ∆DEF. AF2 + BD2 + CE2 = AE2 + CD2 + CD2 + BF2.
4. ∵ ∆ABC ~ ∆PQR Hence the result.
2 2
ar (∆ PRQ) QR 3 9 8. ∆ABC ~ ∆PQR
∴ = 2 =   = = 9 : 1.
ar (∆ BCA) BC 1 1
AB BC
5. True ⇒ = and ∠B = ∠Q
PQ QR
Hint: Use Basic Proportionality Theorem
6. Hint:
Use: ∠1 = ∠2 A B
3
∠3 = ∠4. 2
7. Draw EOF || AD
A D 4
D C
F
1
E F BC
O AB
B 1 ⇒ = 2 and ∠B = ∠Q
C E PQ 1
QR
∴ OB2 = EO2 + EB2 2
OD2 = OF2 + DF2
AB BP
∴ OB2 + OD2 = EO2 + EB2 + OF2 + DF2 ⇒ = and ∠B = ∠Q
PQ QM
= EO2 + CF2 + OF2 + AE2
[∵ DF = AE, EB = CF] (∵ BD = DC and QM = MR)
= (EO2 + AE2) + (CF2 + OF2) ⇒ ∆ABD ~ ∆PQM
OB2 + OD2 = OA2 + OC2. AB AD
⇒ = . Hence proved.
OR PQ PM
Join OA, OB and OC OR
In right ∆AOF, Hint: Show that
AO2 = AF2 + OF2 ...(i) ∆ABD ~ ∆PQM

56 M A T H E M A T I C S – X
Similarly, we can prove that
2
ar (∆ABC)  AB 
=   ...(vi)
ar (∆PQR)  PQ 
2
ar(∆ABC)  AC 
9. Let the two given triangles be ABC and and =   ...(vii)
PQR such that ∆ABC ~ ∆PQR ar(∆PQR)  PR 

AB BC From equations (v), (vi) and (vii), we obtain


∴ = ...(i)
PQ QR  AB 
2 2
ar(∆ABC)  BC 
=   =  
ar(∆PQR)  PQ   QR 
2
 AC 
=   .
 PR 
Hence, the theorem.
Further, in the question,
Let us draw perpendiculars AD and PM
from A and P to BC and QR respectively. 2
ar(∆ABC)  BC 
∴ ∠ADB = ∠AMQ = 90° ...(ii) =  
ar(∆DEF)  EF 
Now, in ∆ABD and ∆PQM,
64 BC 2
∠B = ∠Q (∵∆ABC~∆PQR) ⇒ =
121 15.4 × 15.4
∠ADB = ∠AMQ [From (ii)]
So, by AA rule of similarity, we have 64 × 15.4 × 15.4
∆ABD ~ ∆PQM ⇒ BC =
121
AB AD 8
⇒ = ...(iii)
PQ PM = × 15.4 = 11.2 cm.
11
From equations (i) and (iii), we get
WORKSHEET–38
BC AD
= ...(iv)
QR PM 1. (B) Ratio of areas of two similar triangles
= Ratio of squares of their
1 corresponding sides.
ar (∆ABC) × BC × AD
Now, = 2 = 42 : 92 = 16 : 81.
ar (∆PQR) 1
× QR × PM 2. (A) In ∆ABC, DE || BC
2
AB AD 5
1 ⇒ = ⇒ AB = 21 × = 15 cm.
× BC × BC AC AE 7
= 2 3. (C) ∠M = ∠Q = 35° (Corresponding angles)
1
× QR × QR
2 PQ QR
=
[Using (iv)] ML MN
2 (Ratio of corresponding sides)
 BC 
=   ...(v) 12
 QR  ⇒ MN = 5 × = 10 cm.
6

T R I A N G L E S 57
4 . Yes. the sum of the squares of the other two
sides, then the angle opposite the first side
AP 5 2
= = is a right angle.
AQ 7.5 3

BP 4 2
= =
BR 6 3
AP BP
Here, =
AQ BR
Hence, due to the converse of Basic
Proportionality Theorem, AB || QR.

5. ∵ DB ⊥ BC and AC ⊥ BC Proof: We are given a ∆ABC in which

∴ DB || AC AC2 = AB2 + BC2 ...(i)


We need to prove ∠ABC = 90°.
Now, ∠DBA = ∠BAC (Alternate angles)
Let us construct a ∆PQR such that
And, ∠DEB = ∠ACB (Each 90°)
∠PQR = 90°
∴ ∆BDE ~ ∆ABC (AA similarity)
and PQ = AB ...(ii)
BE DE QR = BC ...(iii)
= (Corresponding sides)
AC BC Using Pythagoras Theorem in ∆PQR, we
BE AC have
⇒ = . Hence proved.
DE BC PR2 = PQ2 + QR2
∴ PR2 = AB2 + BC2 ...(iv)
AX 2 − 2 [Using equations (ii) and (iii)]
6. =
AB 2 From equations (i) and (iv), we have
Hint: See worksheet-35 Sol. 8. AC = PR ...(v)
1 Now, in ∆ABC and ∆PQR,
7. Hint: AM = AB;
2 AB = PQ (From (ii)]
1 BC = QR [From (iii)]
AL =AC
2 AC = PR [(From (v)]
Using Pythagoras Theorem. So, ∆ABC ≅ ∆PQR (SSS congruence)

8. Hint: Join AC and use Basic proportionality ∴ ∠ABC = ∠PQR (CPCT)


theorem. But ∠PQR = 90° (By construction)
A B
∴ ∠ABC = 90°. Hence proved.
2nd Part:
E
X
F As AB2 = 2AC2
A
= AC2 + AC2
D C ⇒ AB2 = BC2 + AC2
9. Converse of Pythagoras Theorem: In a [... BC = AC]
triangle, if square of one side is equal to ⇒ ∠C = 90°. B C

58 M A T H E M A T I C S – X
WORKSHEET–39 AD 5 AD 5
6. = ⇒ =
1. (A) Let the length of shadow is x metres. DB 4 AB – AD 4
BE = 1.2 × 4 = 4.8 m AD 5
∆ABC ~ ∆DEC ⇒ 5AB – 5AD = 4AD ⇒ = ...(i)
AB 9
AB BC As DE || BC,
=
DE EC ∆ADE ~ ∆ABC
3.6 4.8 + x
⇒ = DE AD 5
0.9 x ∴ = = ...(ii)
BC AB 9
3.6x = 4.32 + 0.9x.
[Using (i)]
4.32 ∵ DE || BC and DC is a transversal
x= = 1.6 m.
2.7 ∴ ∠EDC ~ ∠BCD

( )
2
2. (B) Here,(a)2 + 3a = a2 + 3a2 (Alternate interior angles)
= 4a2 = (2a)2 i.e., ∠EDF = ∠BCF ...(iii)
According to the converse of Pythagoras Similarly,
Theorem, the angle opposite to longest side ∠DEF = ∠CBF ... (iv)
is of measure 90°.
From equations (iii) and (iv), we have
AD 2 AB – AD 3 ∆DEF ~ ∆CBF (AA similarity)
3. (A) = ⇒ =
DB 3 AD 2 2
ar (∆DEF)  DE  25
AB 3 AB 5 ⇒ =   = .
⇒ –1 = ⇒ = ar (∆CFB)  BC  81
AD 2 AD 2
DE || BC ⇒ ∆ABC~∆ADE [Using equation (ii)]
BC AB 5
∴ = = . 7. A
DE AD 2 D
4. No.
In ∆PQD and ∆RPD,
∠PDQ = ∠PDR = 90°
B L C E M F

AB = AC; DE = DF

AB DE
∴ = =1
AC DF
AB AC
⇒ = also ∠A = ∠D
But neither ∠PQD = ∠RPD DE DF
nor ∠PQD = ∠PRD ⇒ ∆ABC ∼ ∆DEF
Therefore, ∆PQD is not similar to ∆RPD.
ar(∆ABC) AL2
A ∴ =
5. Hint: ∆BAC ~ ∆ADC ar( ∆DEF) DM 2
1
BA AC BC AL 4
⇒ = = ⇒ =
AD DC AC DM 5
⇒ CA2 = BC × CD. 2
B C ∴ Ratio of corresponding heights is 4 : 5.
D

T R I A N G L E S 59
OR AC(AD + CD) = AB2 + BC2
Proof: Draw a ray DZ parallel to the ray AC.AC = AB2 + BC2
XY. AC2 = AB2 + BC2.
In ∆ADZ, XY || DZ Hence proved.
2nd part:
AY AX 2
∴ = = AB2 = AD2 + BD2
YZ XD 3
⇒ 2YZ = 3AY ... (i) = AD2 + (3CD)2
= AD2 + 9CD2
In ∆YBC, BY || DZ
= AD2 + CD2 + 8CD2
YZ BD 1
∴ = = (... BD = DC) = AC2 + 8CD2
ZC DC 1 2
1
⇒ 2YZ = 2ZC ... (ii) = AC2 + 8  BC
From (i) and (ii), 4 
2ZC = 3AY ... (iii)  1 
∵ CD = 4 BC 
Now, AC = AY + YZ + ZC
3 3 8 2AB2 = 2AC2 + BC2. Hence proved.
= AY + AY + AY = AY
2 2 2
= 4AY WORKSHEET– 40
Therefore, AC : AY = 4 : 1. Hence Proved.
AB AB AC
1. (A) = =
8. 2 5 cm PQ QR PR
1 (∵ ∆ABC ~ ∆PQR)
Hint: BD = BC;
2
12 7 10
1 ⇒ = =
AE = EB = AB 9 x y
2
Using Pythagoras Theorem. 7×9 21
x= =
9. Statement : In a right triangle, the square 12 4
of the hypotenuse is equal to the sum of
the square of the other two sides. 9×10 15
and y= = .
Proof: We are given a right triangle ABC 12 2
right angled at B.
16 4
We need to prove that 2. (C) Required ratio = = = 4 : 5.
25 5
AC2 = AB2 + BC2
Let us draw BD ⊥ AC. 3. 17 m
Now, ∆ADB ~ ∆ABC Hint:
AD AB N
So, = (sides are proportional)
AB AC
or AD .AC = AB2
P
...(i)
8m
Also, ∆BDC ~ ∆ABC W E
15 m O
CD BC
So, =
BC AC
or CD .AC = BC2 ...(ii)
S
Adding equations (i) and (ii) we get
AD . AC + CD . AC = AB2 + BC2 Use Pythagoras Theorem and find OP.

60 M A T H E M A T I C S – X
4. Hint: 7. Hint:
Let AB = c A
AC = b D
BC = a
...
E
a2 = b 2 + c2
B C
3 2
Also, ar(∆ABE) = a Prove that ∆AEB ~ ∆DEC.
4
8. See worksheet–33, sol. 9 (Ist part).
3 2
ar(∆BCF) = c 2nd part
4
Join EF and join BD to intersect EF at O.
3 2
ar(∆ACD) = b .
4
5. See worksheet − 36 sol. 6.
6. Let ABCD be a quadrilateral of which
diagonals intersect each other at O.
It is given that ∵ AB || DC, and EF || AB,
AO BO ∴ AB || DC || EF
= In ∆ABD, EO || AB,
CO DO
AO CO DE DO
or = ...(i) = ...(viii)
BO DO AE BO
In ∆AOB and ∆COD, (Basic Proportionality Theorem)
∠AOB = ∠COD Similarly, in ∆BCD,
(Vertical opposite angles)
DO CF
AO CO = ...(ix)
= [From (i)] BO BF
BO DO
Using equations (viii) and (ix), we obtain
Hence, by SAS rule of similarity, we obtain
the required result.
∆AOB ~ ∆COD
⇒ ∠BAO = ∠DCO AE BF
= .
i.e. ∠BAC = ∠DCA ED FC
These are alternate angles.
WORKSHEET– 41
Therefore, AB || CD and AC is transversal
⇒ ABCD is a trapezium. Hence proved DE EF DF
1. (B) = =
OR AB BC AC
Hint: (i) (ii) (iii)
As ∠BAC = ∠EFG ; ∠ABC = ∠FEG
DE + EF + DF
And ∠ACB = ∠FGE =
AB + BC + CA
1 1 (iv)
∴ ∠ACB = ∠FGE
2 2
4 Perimeter of ∆DEF
∴ ∠ACD = ∠FGH ⇒ =
2 3 + 2 + 2.5
and ∠DCB = ∠HGE
∴ ∆DCA ~ ∆HGF [Taking (ii) and (iv)]
Similarly ∆DCB ~ ∆HGE. ⇒ Perimeter of ∆DEF = 15 cm.

T R I A N G L E S 61
2. (A) DE || BC
x x+2
⇒ =
x–2 x –1
⇒ x2 – 4 = x2 – x
⇒ x = 4.
3. ∆KNP ~ ∆KML We need to prove
x c ac 1
⇒ = ∴x= . ar (∆APB) = ar (∆AQC)
a b + c b +c 2
In right ∆ABC,
4. Hint:
AC = AB2 + BC2
A B L
= AB 2 ...(i)
P (∵ AB = BC)
3
D C Now, ar(∆APB) = AB2 ...(ii)
4
Prove that ∆ADL ~ ∆CPD.
3
1 And ar(∆AQC) = AC2
5. Hint: 2AP = PC ⇒ AP = AC 4
3
3
( )
2
1 = AB 2
Similarly, BQ = BC 4
3
Use Pythagoras Theorem. 3
= AB2 ...(iii)
2
AP AQ 1 Divide equation (ii) by equation (iii), we
6. PQ || BC ⇒ = =
PB QC 2 obtain
∴ Also ∆APQ ~ ∆ABC 3
ar (∆APB) AB 2 1
2
= 4 =
ar (∆ABC)  AB 
⇒ =   = ( 3 )2 = 9 ar (∆AQC) 3 2
ar (∆APQ)  AP  AB 2
2
 AB  1
∵ AP = 3 ⇒ ar(∆APB) = ar(AQC).
2
ar (∆ABC) Hence proved.
⇒ −1=8
ar (∆APQ) 7. Hint:
ar ( BPQC) 8 ar (∆APQ) 1
⇒ = ⇒ = .
ar (∆APQ) 1 ar ( BPQC) 8
∴ Ratio of area of ∆APQ and trapezium
BPQC is 1 : 8.
OR
Let the given square be ABCD.
Let us draw an equilateral triangle APB
and another equilateral triangle AQC on
the side AB and on the diagonal AC Extend AD till E such that AD = DE and
respectively. similarly, PM = MN

62 M A T H E M A T I C S – X
Prove that ∆ACE ~ ∆PRN ar (∆ ABC) BC × AM
∠ 1 = ∠2 ...(i) Therefore, =
ar (∆ PQR) QR × PN
But ∠ 3 = ∠5 (CPCT)
∠ 3 = ∠4 ∠ 4 = ∠6 BC 2
= ...(v)
... ∠
∠5 = ∠6 ...(ii) QR 2
Adding (i) and (ii), [Using (iv)]
∠1 + ∠5 = ∠2 + ∠6
From results (i) and (v), we arrive
⇒ ∠BAC = ∠QPR
... ∆ABC ~ ∆PQR. (By SAS) ar (∆ ABC) AB 2 BC 2 CA 2
= = = .
8. Let us take two similar triangles ABC and ar (∆ PQR) PQ 2 QR 2 RP 2
PQR such that ∆ABC ~ ∆PQR. Hence the result.
AB BC CA Further, consider the question in the
... = = ...(i) following figure.
PQ QR RP
We need to prove
ar (∆ABC) AB 2 BC 2 CA 2
ar(∆PQR) = 2 = 2 =
PQ QR RP 2

∠ABO = ∠CDO and ∠BAO = ∠DCO


(Alternate angles)
⇒ ∆AOB ~ ∆COD (AA rule)
ar (∆AOB) AB 2
⇒ ar (∆COD) =
CD 2
Let us draw AM ⊥ BC and PN ⊥ QR. 2
1  CD 1 
∵ ∆ABC and ∆PQR ⇒ ar(∆COD) = 84 ×   ∵ = 
2  AB 2 
∴ ∠B = ∠Q ...(ii)
= 21 cm2.
In ∆ABM and ∆PQN,
∠B = ∠Q [From (ii)] WORKSHEET– 42
and ∠M = ∠N (Each 90°) 1. (A) ∆OBC ~ ∆ODA (AA criterion)
∴ ∆ABM ~ ∆PQN (AA criterion)
AB AM
∴ = ...(iii)
PQ PN
From equations (i) and (iii), we have
AM BC OB OC BC
= ...(iv) ⇒ = = =2
PN QR OD OA DA
⇒ BC = 2DA = 2 × 4 = 8 cm.
1
Now, ar(∆ABC) = × base × height
2 2. (D) Let the given areas be 2x and 3x.
1 Required ratio = 2x : 3x = 2 : 3.
= × BC × AM
2
3. (C) x = 11 or 8
1 OD OC
And ar(∆PQR) = × QR × PN Hint: Use = .
2 OB OA

T R I A N G L E S 63
4. True ∴ Using Pythagoras Theorem
Geometrical figures which are equiangular AD2 = AE2 + DE2
i.e., if corresponding angles in two = AE2 + (BE − BD)2
geometrical figure are same, are similar.
⇒ AD2 = AE2 + BE2 + BD2 − 2.BE.BD
5. In right ∆ADC,
2
AD2 = AC2 – CD2 1
⇒ AD2 = AE2 + EC2 +  a  − 2
= (2CD)2 – CD2 3 
[∵ AC = BC = 2 CD]
1 1 
= 3 CD2.  a  a
2 3 
6. Hint: BMDN is a rectangle.
∆BMD ~ ∆DMC a2 a2
= AC2 + −
DN DM 9 3
⇒ = ⇒ DM2 = DN × MC
DM MC a2 a2 7 a2
Also, ∆
∆BND ~ ∆DNA. = a2 + − =
9 3 9
DM DN
⇒ = ⇒ DN2 = DM × AN. ⇒ 9AD2 = 7AB2.
DN AN
9. See worksheet-39 sol. 9 (I part).
7. Let BE = 3x and EC = 4x.
Hint: 2nd Part:
In ∆BCD, GE || DC
∴ ∆BGE ~ ∆BDC AC2 = AD2 + DC2
BE GE = AD2 + (3BD)2
∴ = = AD2 + 9BD2
BC DC
= AD2 + BD2 + 8BD2
3x GE
⇒ = (∴ DC = 2 AB) 2
3x + 4x 2AB 1 
= AB2 + 8  BC 
4 
6
⇒ GE = AB ...(i) ⇒ 2AC2 = 2AB2 + BC2 .
7
Similarly, ∆DGF ~ ∆DBA
WORKSHEET– 43
FG 4 4
⇒ = ⇒ FG = AB ...(ii)
AB 7 7 1 . (C) 294 cm2
Adding equations (i) and (ii), we get Hint: Prove that ∆OBP ~ ∆OAQ.
6 4 2. (A) 6 cm
GE + FG = AB + AB
7 7 Hint: Use AA-similarity to prove
10 ∆AOB ~ ∆COD.
A
⇒ EF = AB
7 3. Hint: Draw AM ⊥ BC C
N
⇒ 7 EF = 10AB. Hence proved. and DN ⊥ BC
O
As ∆AOM ~ ∆DON M
8. Hint: Let AB = BC = AC = a B D
1
∴ AE ⊥ BC A × BC × AM
ar (∆ ABC) 2
1 ⇒ =
⇒ BE = EC = a ar (∆ DBC) 1
× BC × ON
2 2
1 1
and BD = BC = a AM AO
3 3 B C = = .
D E DN OD

64 M A T H E M A T I C S – X
4. Hint: Use concept of similarity. ⇒ ∠ALE = ∠CLB
5. Draw AP ⊥ BC (Vertically opposite angles)
∴ ∆LAE ~ ∆LCB (AA criterion)
A
AE LE
⇒ = (Corresponding sides)
BC BL
2BC EL
⇒ = [Using equation (i)]
B
1 2
C
BC BL
D P
⇒ EL = 2BL. Hence proved.
∴ AB2 = AP2 + BP2 OR
= AP2 + (BD + DP)2 Hint:
⇒ AB2 = AP2 + BD2 + DP2 + 2BD. DP
= AD2 + BD (BD + 2DP)
⇒ AB2 − AD2 = BD × CD. [... BP = PC]
Hence proved.
6. Hint:

As AD is median
P
a
b so, AB2 + AC2 = 2(AD2 + BD2)
x
p  2 BC2 
⇒ AB2 + AC2 = 2  AD + 
 4 
ab
From figure, show x = . ⇒2(AB2 + AC2) = 4AD2 + BC2 ...(i)
a+b
⇒ Similarly,
7. ∆MDE and ∆MCB,
2(AB2 + BC2) = 4BE2 + AC2 ...(ii)
∠MDE = ∠MCB (Alternate angles) 2 2 2 2
2(AC + BC ) = 4CF + AB ...(iii)
MD = MC (M is mid-point of CD) Add (i), (ii) and (iii),
∠DME = ∠CMB 3 (AB2 + AC2 + BC2)
(Vertically opposite angles) = 4 (AD2 + BE2 + CF2).
8. See worksheet-33 sol. 9 (Ist part).
2nd Part: Draw EM || AB
M is a point on CB
∴ EM || AB

CE CM
⇒ = ...(i)
∴ ∆MDE ≅ ∆MCB, (ASA criterion) AE MB
⇒ DE = CB (CPCT) DE CM
⇒ AE – AD = BC Also = ...(ii)
EB MB
⇒ AE = 2BC ...(i) (∵ BC = AD) From (i) and (ii),
Now, in ∆LAE and ∆LCB,
CE DE
⇒ ∠LAE = ∠LCB (Alternate angles) = .
AE EB

T R I A N G L E S 65
WORKSHEET– 44 ∴ ∆BMC ≅ ∆AND (RHS criterion)
⇒ BM = AN ...(i) (CPCT)
3 4 Using result on obtuse triangle ABC
1. (D) BE = a ⇒ a= BE
4 3 (∠ABC > 90°),
∴ AB2 + BC2 + CA2 AC2 = AB2 + BC2 + 2AB.BM ...(ii)
= a2 + a2 + a2 Using result on acute triangle ABD
= 3a2 (∠BAD < 90°),
2 BD2 = AB2 + DA2 – 2AB .AN
 4 
=3 ×  BE ⇒ BD2 = CD2 + DA2 – 2AB . BM ...(iii)
 3 
[Using (i) and AB = CD]
= 16 BE2. Adding equations (ii) and (iii), we have
2. (B) AC2 + BD2 = AB2 + BC2 + CD2 + DA2.
Hint: Use pythagoras theorem Hence proved.
3. (B) 6. Hint: AP || QB || RC
Hint: Using basic proportionality theorem Use Basic proportionality theorem.
we get, x = 4. 7. See worksheet 38 sol. 9 (Ist part).
4. Hint: In ∆ACD and ∆ABC, 8. Let us produce AD to J and PM to K so
∠A = ∠A that DJ = AD and MK = PM.
∠ACD = ∠ACB = 90° Join CJ and QK.
⇒ ∆ACD ~ ∆ABC
⇒ AC2 = AB. AD …(i)
∆BCD ~ ∆BAC
⇒ BC2 = BA. BD …(ii)
∴ Applying (ii) ÷ (i) gives the result.
5. Let the given parallelogram be ABCD
We need to prove that
AC2 + BD2 = AB2 + BC2 + CD2 + DA2
Let us draw perpendiculars DN on AB In ∆ADB and ∆JDC,
and CM on AB produced as shown in AD = JD, ∠ADB = ∠JDC, BD = CD
figure.
⇒ ∆ADB ≅ ∆JDC
(SAS criterion of congruence)
⇒ AB = JC ...(i) (CPCT)
Similarly, we can prove that
PQ = KR ...(ii)
According to the given conditions, we have
AB AD AC
= =
PQ PM PR
In ∆BMC and ∆AND,
AJ
BC = AD (Opposite sides of a ||gm) JC AC
⇒ = 2 =
∠BMC = ∠AND (Each 90°) KR PK PR
CM = DN (Distance between 2
same parallels) [Using (i) and (ii)]

66 M A T H E M A T I C S – X
JC AJ AC 100 52 25 × 49
⇒ = = ∴ = 2 ⇒ h2 =
KR PK PR 49 h 100
⇒ ∆AJC ~ ∆PKR (SSS criterion of
similarity) 25 × 49 5×7
⇒ h= ⇒ h2 = = 3.5 cm.
⇒ ∠JAC = ∠KPR (Corresponding 100 10
angles) 3. (B) Altitude AM divides base BC in two
i.e., ∠DAC = ∠MPR ...(iii) equal parts. That is BM = MC = 7 cm using
Similarly, we can prove that Pythagoras Theorem in right ∆ABM,
∠DAB = ∠MPQ ...(iv)
Adding equations (iii) and (iv), we obtain
∠BAC = ∠QPR ...(v)
Thus, in ∆ABC and ∆PQR, we have
AB AC
= (Given)
PQ PR
AM = 252 – 7 2 = (25 + 7) (25 – 7)
and ∠BAC = ∠QPR [From (v)]
Therefore, ∆ABC ~ ∆PQR. = 32 × 18 = 24 cm.
(SAS criterion of similarity) 4. (i) We know that diagonal of a square
Hence proved. = 2 × side
OR
In square AEFG, AF = 2 AG ...(i)
Hint: As ∆ADC is obtuse angled triangle,
AC2 = AD2 + CD2 + 2 . CD . DE …(i) In square ABCD,AC = 2 AD ...(ii)
and ∆ADB is acute-angled triangle Using equations (i) and (ii), we obtain
So, AB2 = AD2 + BD2 − 2 . DB . DE …(ii) AF AC
= . ...(iii)
Add (i) and (ii), to get the result. AG AD
(ii) ∠GAF = ∠DAC (Each 45°)
WORKSHEET– 45 ⇒ ∠GAF – ∠GAC = ∠DAC – ∠GAC
⇒ ∠CAF = ∠DAG ...(iv)
1. (D) ∆ABC ~ ∆PQR
From equations (iii) and (iv), we have
20 10 ∆ACF ~ ∆ADG.
⇒ =
h 50 (SAS criterion)
5 . Hint: ∵ ∠1 = ∠2
∴ PQ = PR
QR QT
∴ = .
QS PQ
6. Hint: Draw AM ⊥ BC and DN ⊥ BC.
7. Hint: Fig. A

50 × 20
⇒ h= = 100 m. c
10 b
p
2. (A) The ratio of similar triangles is equal
to the ratio of squares of their correspond- B
C a
ing altitudes.

T R I A N G L E S 67
8. Hint for 1st part: Prove Pythagoras Theorem. ASSESSMENT SHEET–7
Hint for 2nd part: Let the given triangle is
ABC with 1. (C) In ∆ABC, PQ || BC
∠A = 90° AP AQ
∴ =
Draw AD ⊥ BC BP QC
∆ABC ~ ∆DAC 2.4 2
and ∆ABC ~ ∆DBA. ∴ = ⇒ BP = 3.6 cm
BP 3
9. Hint: Let the DC = AB = x ∴ AB = AP + BP = 2.4 + 3.6 = 6 cm.
4 3 2
Then QC = x and AP = x ar(∆ABC) BC2 9  BC 
5 5 2. (B) = ⇒ =  
∆QRC ~ ∆PRA. ar(∆DEF) EF 2
4  EF 
OR BC 3
⇒ = .
Let the given parallelogram be ABCD. EF 2
We need to prove 3. Let draw AM ⊥ BC and DN ⊥ BC
AC2 + BD2 = AB2 + BC2 + CD2 + DA2 ∵ ∠AMO = ∠DNO = 90°
Let us draw perpendiculars DN on AB and and ∠AOM = ∠DON
CM on AB produced as shown in figure.
In ∆BMC and ∆AND

BC = AD (opposite sides of a ∴ ∆AMO ~ ∆DNO


parallelogram) (AA similarity)
∠BMC = ∠AND (Each 90°) AM AO
CM = DN (Distance between ∴ = ...(i)
DN DO
same parallels) 1
∴ ∆BMC ≅ ∆AND (RHS criterion) × BC × AM
ar(∆ABC) 2
⇒ BM = AN ...(i) CPCT Now, ar(∆DBC) = 1
× BC × DN
In obtuse triangle ABC (∠ABC > 90°), 2
AC2 = AB2 + BC2 + 2AB . BM AO
...(ii) = . [Using (i)]
DO
In acute triangle ABD (∠BAD < 90°),
4. True, because ∆BCD ~ ∆CAD
BD2 = AB2 + DA2 – 2AB . AN
⇒ BD2 = CD2 + DA2 – 2AB . BM ⇒ CD2 = BD . AD.
...(iii) 5. PQ || BC and AB is transversal
[Using (i) and AB = CD] ∴ ∠APQ = ∠ABC ...(i)
Adding equations (ii) and (iii), we have (Corresponding angles)
AC2 + BD2 = AB2 + BC2 + CD2 + DA2. In ∆ABC and ∆APQ,
Hence proved. ∠BAC = ∠PAQ (Common)

68 M A T H E M A T I C S – X
∠ABC = ∠APQ [From (i)] We need to prove
so, by AA criterion of similarity, ar(∆ABC) AD 2
=
∆ABC ~ ∆APQ ar(∆PQR) PM 2

ar( ∆ABC) In ∆ABD and ∆PQM,


AB2
∴ ar( ∆APQ) = ∠ADB = ∠PMQ = 90°
AP2
∠ABD = ∠PQM (∴∆ABC ~ ∆PQR)
∴ ∆ABD ~ ∆PQM
Subtracting unity from both the sides, we
(AA criterion of similarity)
have.
AB AD
2
ar( ∆ ABC) – ar( ∆ APQ) = AB – AP
2 ⇒ = ...(i)
⇒ PQ PM
2
ar(∆ APQ) AP (Corresponding sides)
2
We know that the ratio of areas of two
ar(trapezium BPQC)  AB  similar triangles is equal to ratio of squares
⇒ =   –1
ar(∆APQ)  AP  of their corresponding sides
...(ii) ar(∆ABC) AB 2
It is given that ∴ = ...(ii)
ar(∆PQR) PQ 2
AP 1 PB From equations (i) and (ii), we have
= ∴ =2
AB 2 AP ar(∆ABC) AD 2
= . Hence proved.
PB ar(∆PQR) PM 2
∴ + 1= 2 + 1
AP
PB + AP AB ASSESSMENT SHEET–8
⇒ =3⇒ =3
AP AP
x 6 30 15
2 1. (B) = ⇒x= ,⇒x= cm.
 AB  5 6+2 8 4
∴   =9 ...(iii)
 AP 
2. (A) ∆PQR ~ ∆CAB,
From equations (ii) and (iii), we have
PQ PR QR
ar(∆APQ) 1 ⇒ = =
= CA BC AB
ar(trapezium BPQC) 8
AB BC CA
⇒ ar(∆APQ) : ar(trapezium BPQC) or = ⇒ .
QR PR PQ
= 1 : 8. 3. In ∆ABC, ∠M || AB.
6. See worksheet-36 sol. 6. Using Basic Proportionality Theorem, we
7. See worksheet-33 sol. 9 (Ist part). have
8. We are given two triangles ABC and PQR AC BC 2x 2x + 3
= ⇒ =
such that ∆ABC ~ ∆PQR. AL BM x – 3 x–2
Draw perpendiculars AD and PM on BC ⇒ 2x2 – 4x = 2x2 + 3x – 6x – 9
and QR respectively.
⇒ – x – 9 ⇒ x = 9.
4. False, because ∆PQD and ∆PRD don’t obey
any condition of similarity.
5. Let the given right angled triangle be ABC
with ∠C = 90° such that AC = b, BC = a
and AB = c.

T R I A N G L E S 69
∵ ∆ABC ~ ∆PQR
1
AB BC BC
∴ = = 2
PQ QR 1
QR
2
AB BD
⇒ = ...(i)
PQ QM
(a) In ∆ABD and ∆PQM,
AB BD
⇒ = [From (ii)]
Using Pythagoras Theorem, we have PQ QM
AB2 = AC2 + BC2 ∠ABD ~ ∠PQM (∵∆ABC ~ ∆PQR)
⇒ c2 = a2 + b2 ...(i) So, by SAS criterion of similarity, we have
Area of equilateral triangle drawn on side ∆ABD ~ ∆PQM
BC AB AD
⇒ =
3 2 PQ PM
= a ...(ii)
4 (b) ∵ ∆ABD ~ ∆PQM, [From part (a)]
Similarly, areas of equilateral triangles ⇒ ∠ADB = ∠PMQ
drawn on side BC and side AB are
⇒ 180° – ∠ADC = 180° – ∠PMR
respectively
[From figure]
3 2 ⇒ ∠ADC = ∠PMR. Hence proved.
= b ...(iii)
4 7. See worksheet-38 sol. 9 (Ist part).
3 2 8. BD = BE (Given)
And = c ...(iv)
4 In ∆OBD, AF ⊥ OB and BD ⊥ OB
Sum of areas of equilateral triangles drawn ∴ AF || BD
on the sides BC and AC ∴ ∆OAF ~ ∆OBD
3 2 3 2 OA AF
= a + b ⇒ =
4 4 OB BD
[Adding (ii) and (iii)] OA AF
⇒ = ...(i) [Using (i)]
3 2 OB BE
= (a + b2)
4 In ∆AFC and ∆BEC,
3 2 ∠FAC = ∠EBC (Each 90°)
= c [Using (i)]
4 ∠FCA = ∠ECB
= Area of equilateral triangle (Vertically opposite angles)
drawn on hypotenuse AB. So by, AA criterion of similarity,
Hence proved. ∆AFC ~ ∆BEC
6. AC AF
⇒ = ...(ii)
BC BE
Comparing equations (i) and (ii), we have
OA AC
=
OB BC

70 M A T H E M A T I C S – X
OA OC – OA PM PN
⇒ = Clearly, =
OB OB – OC MQ NR

⇒ OA × OB – OA × OC ⇒ MN || QR.
= OB × OC – OB × OA 5. ∆AOB ~ ∆COD (AAA criterion of
⇒ (OA + OB) × OC similarity)
= 2OA × OB AO BO
⇒ = (Corresponding sides)
Dividing both sides by OA × OB × OC, we CO DO
get 7x – 9 9x – 8
⇒ =
1 1 2 2x – 1 3x
+ = . Hence proved.
OA OB OC ⇒ 21x2 – 27x = 18x2 – 16x – 9x + 8
⇒ 3x2 – 2x – 8 = 0 ⇒ (x – 2) (3x + 4)
CHAPTER TEST 3
⇒ x = 2 or x = –
4
1. (B) BC = 52 +122 = 13 cm ⇒ x = 2. (Negative value rejected)
6. ∴ ∆ABE ≅ ∆ACD
∴ AB = AC and AE = AD (CPCT)
Consider AB = AC
⇒ AD + DB = AE = EC
⇒ DB = EC ...(i) (∴ AE = AD)
∆ABD ~ ∆CBA Also AD = AE ...(ii)
AB AD (Proved above)
⇒ =
BC AC Dividing equation (ii) by equation (i), we
have
5 × 12 60
⇒ AD = = cm.
13 13 AD AE
= ...(iii)
DB EC
∆ P2 40 2 16
2. (D) 1 = 12 = 2
= Hence, in ∆ABC
∆2 P2 50 25
AD AE
⇒ ∆1 : ∆2 = 16 : 25. =
DB EC
AD AE 1.5 1 ⇒ DE || BC (Converse of Basic
3. (C) = ⇒ =
DB EC 3 EC Proportionality Theorem)
3 ⇒ ∠ADE = ∠ABC and ∠AED = ∠ACB
⇒ EC = = 2 cm.
1.5 ⇒ ∆ADE ~ ∆ABC.
4. Yes. 7. Hint:
MQ = PQ – PM x AC
= 15.2 – 5.7 = 9.5 cm ∆PAC ~ ∆QBC ⇒ =
z BC
NR = PR – PN = 12.8 – 4.8 = 8 cm y AC
∆RCA ~ ∆QBA ⇒ = .
PM 5.7 z AB
Now, = = 0.6
MQ 9.5 8. Hint:
PN 4.8 Draw MN || AD, passing through O to
and = = 0.6
NR 8 intersect AB at M and DC at N.

T R I A N G L E S 71
AB BC
∆ABC ~ ∆PQR ⇒ = and ∠B = ∠Q
PQ QR
AB 2BD
⇒ = and ∠B = ∠Q
PQ 2QM
Use Pythagoras Theorem for ∆AOM,
∆BOM, ∆CON and ∆DON. AB BD
⇒ = and ∠B = ∠Q
9. Hint : PQ QM
⇒ ∆ABD ~ ∆PQM
AB AD
⇒ = .
PQ PM
‰‰

72 M A T H E M A T I C S – X
Chapter

5 INTRODUCTION TO TRIGONOMETRY
WORKSHEET–50
sin 10°
1. (B) sin 3A = cos (A – 26°) = + cos 59° sec 59°
sin 10°
⇒ sin 3A = sin {90°– (A – 26°)}
cos 59°
116° =1+ =1+1=2
⇒ 3A = 90° – A + 26° ⇒ A = cos 59°
4
⇒ A = 29°. Hence, the given equation is valid.

x x 1 4 1
2. (D) 2 sin = 1 ⇒ sin = 7. 2
+ 2 – cos2 45°
2 2 2 cot 30° sin 60°
x π x π 2
⇒ sin = sin ⇒ = 4 1  1 
2 6 2 6 = + – 
( 3)
2 2
 3  2
π  
⇒x= ⇒ x = 60°.  2 
3

24
2 4 4 1 8+8−3
 24  = + – =
3. (B) sin θ = ⇒ sin2 θ =   3 3 2 6
25  25 
13
242 2 = .
⇒1– sin2 θ=1– ⇒ cos2θ = 7 6
252 252
3
7 8. sin (x + y) = 1 and cos (x – y) =
⇒ cos θ = 2
25
⇒ sin (x + y) = sin 90° and cos (x – y) = cos 30°
sin θ 1 ⇒ x + y = 90° and x – y = 30°
Now, tan θ + sec θ = +
co s θ cos θ Adding and subtracting, we get respectively
24 2x = 120° and 2y = 60°
1 24 25 49 i.e., x = 60° and y = 30°.
= 25 + = + = = 7.
7 7 7 7 7
25 25 9. cosec A = 10

sin θ 1 1
4. tan θ = . sin A = =
cosec A 10
1 − sin 2 θ
5. cot 25° + tan 41°. 1 3
cos A = 1 – sin 2 A = 1– =
6. True, 10 10

cos 80° sin A 1


LHS = + cos 59° cosec 31° tan A = =
sin 10° cos A 3
cos(90° – 10°) 1
= + cos 59° cosec (90° – 59°) cot A = =3
sin 10° tan A

I N T R O D U C T I O N T O T R I G O N O M E T R Y 73
1 10 24 7
sec A = = . sin C = and cos C = .
cos A 3 25 25

sin 2 A 1 – cos2 A cos 60° + sin 30° − cot 30°


10. Hint: RHS = = 7.
1 – cos A 1 – cos A tan 60° + sec 45° − cosec 45°
(1 – cos A) (1 + cos A) 1 1
= + – 3
1 – cos A 2 2 1– 3 3
= = ×
OR 3+ 2– 2 3 3

sin θ (1 – 2 sin 2 θ) 3 –3
Hint: LHS = = .
(2cos 2 θ – 1 ) 3
sin θ × cos 2 θ sin θ 8. Given expression
= = .
cos θ × cos 2 θ cos θ cot θ tan (90° − θ) − sec (90° − θ) cosec θ
=
sin θ cos (90° − θ) + cos θ sin (90° − θ)
WORKSHEET–51
cot θ cot θ − cosec θ cosec θ
1. (A) sin (θ + 36°) = cos θ =
sin θ sin θ + cos θ cos θ
⇒ sin (θ + 36°) = sin (90°– θ)
⇒ θ + 36° = 90° – θ ⇒ 2θ = 54° cot 2 θ − cosec 2 θ
=
⇒ θ = 27°. sin 2 θ + cos 2 θ
2. (C) = cosec2 θ – 1 – cosec2 θ
Hint: Divide numerator and denominator [∵ sin2 θ + cos2 θ = 1]
by cos θ. = – 1.
5 25 1
3. (D) sec θ = ⇒ sec2 θ = 9.
4 16 2
25 Hint: Draw ∆ ABC with
⇒ sec2 θ – 1 = –1 AB = BC = AC = a (say)
16
Draw AD ⊥ BC
9 3
⇒ tan2 θ = ⇒ tan θ = . ∴ ∠BAD = ∠DAC = θ = 30°
16 4
and BD = DC = a/2
4. (A)
BD a/2 1
Hint: ∠A = 30º, ∠B = 90º, ∠C = 60º. ∴ sin θ = = =
AB a 2
1 sin θ (1 + sin θ)
5. sec θ + tan θ = + = 1
cos θ cos θ cos θ ⇒ sin 30° = .
2
a
1+ 1
1 + sin θ b = b+a
= = tan θ tan θ
1 − sin 2 θ a2 b2 − a2 10. LHS = +
1− 2 1 1 − tan θ
1−
b tan θ

=
b+a b+a
.
tan 2 θ 1
= = +
b+a b − a b−a tan θ − 1 tan θ (1 − tan θ)

7 24 1  2 1 
6. sin A = , cos A = , =  tan θ − tan θ 
25 25 ( tan θ − 1)

74 M A T H E M A T I C S – X
1  tan 3 θ − 1  ⇒ cos θ = sin 60° ⇒ θ = 30°
=   3 3 1
tan θ − 1  tan θ  ... 2cos2 30°– 1 = 2× − 1 = − 1 =
4 2 2
= tan θ + cot θ + 1 4. (D)
Hint: (1 + sin θ) (1 – sin θ) = cos2 θ
tan 2 θ + 1 sec2 θ
= +1 = +1 1
tan θ tan θ = .
sec 2 θ
= 1 + sec θ cosec θ = RHS
5. (A)
OR Hint: sec 4A = cosec (90º – 4A).

cos A 6. Hint: cos (90º – θ) = sin θ,


cot A – cos A − cos A sin (90º – θ) = cos θ.
sin A
LHS = =
cot A + cos A cos A (1 + sin θ)(1 – sin θ) 1 – sin 2 θ
+ cos A 7. =
sin A (1 + cos θ)(1 – cos θ) 1 – cos2 θ
 1  2
cos A  – 1 cos2 θ 7 49
 sin A  = = cot2 θ =   = .
= 2
sin θ 8
  64
 1 
cos A  + 1
 sin A  OR

1 cosec 2 θ + cot 2 θ
−1
=
sin A
=
cosec A – 1
= RHS cosec 2 θ – sec 2 θ
1
+ 1 cosec A + 1 1 + cot 2 θ + cot 2 θ
sin A =
(
1 + cot 2 θ − 1 + tan 2 θ )
WORKSHEET–52
1 + 2 cot 2 θ
=
1. (A) cot 2 θ – tan 2 θ
Hint: tan 5º = cot 85º; tan 25º = cot 65º.
1+ 2×3 7 21
2. (C) 8 tan x = 15 = = = .
1 8 8
3–
225 3 3
⇒ tan2 x =
64 8. sin2 30° + sin2 45° + sin2 60° + sin2 90°
225 289
⇒ sec2x – 1 = sec2 x = 1
2
 1 
2
 3
2
64 64 =   +   +   + ( 1)
2

2  2  2 
17 8
⇒ sec x = ⇒ cos x =
8 17 1 1 3
= + + +1
Now, sin x – cos x = 1 – cos x – cos x 2 4 2 4
1+ 2 + 3 + 4 10 5
64 8 15 – 8 = = = .
= 1– – = 4 4 2
289 17 17 9. Hint:
7 1 − sin θ 1 − sin θ
= . LHS = ×
17 1 + sin θ 1 − sin θ
3. (A)
Hint: sec θ = cosec 60° = (sec θ − tan θ)2

I N T R O D U C T I O N T O T R I G O N O M E T R Y 75
OR 7. Hint:
Hint:
1 − 2sin θ cos θ + 1 + 2sinθ cos θ
cos A sin 2 A LHS = .
LHS = + (sin θ + cos θ)(sin θ − cos θ)
1 − tan A sin A − cos A
cos A − sin A + 1
cos 2 A sin 2 A 8. LHS =
= – cos A + sin A − 1
cos A − sin A cos A – sin A
Dividing Numerator and Denominator by
cos 2 A − sin 2 A sin A, we get
=
cos A − sin A cot A − 1 + cosecA
=
= cos A + sin A. cot A + 1 − cosecA
10. Hint: 1 – 2sin2 θ = 2cos2 θ – 1
(cot A + cosec A ) − (cosec2A – cot2 A )
= cos2 θ – sin2 θ . =
cot A + 1 − cosec A
WORKSHEET–53 (cosec A + cot A) [1 − cosec A + cot A]
=
Perpendicular cot A − cosec A + 1
3
1. (A) tan θ = = C = cosec A + cot A = RHS.
4 Base
BC = 9. Given expression
32 + 42 3 5
 2 sin 68° 2 cot 15°
= 25 = 5 A 4 B
= –
cos 22° 5 tan 75°
Base 4
cos θ = = 3 tan 45° tan 20° tan 40° tan50° tan70°
Hypotenuse 5 –
5
4 2 cot (90° − 75°)
1− 2 sin (90° − 22°)
1 − cos θ 5 = 1. = –
∴ = cos 22° 5 tan 75°
1 + cos θ 4 9
1+
5
3 × 1 × tan ( 90° – 70°) tan (90° – 50°)
2. (C)
Hint: tan 50° tan 70°

1 + tan θ + sec θ 5
1 1 + cot θ + cosec θ 2 cos 22° 2 tan 75°
=1+ + sec θ = = –
cot θ cot θ cos 22° 5 tan 75°
3. (B)
Hint: A + B = 90º; A – B = 30º. 3 cot 70° cot 50° tan 50° tan 70°

4. (A) tan 2θ = cot (θ + 9°) 5
⇒ tan 2θ = tan [90° – (θ + 9°)] 2
=2 –
⇒ 2θ = 90° – θ – 9° ⇒ 3θ = 81° 5
⇒ θ = 27°.
1 1
cos θ 3× × tan 50° tan 70°
5. cot θ = . tan 70° tan 50°

1 − cos2 θ 5
6. True.
2 3 10 − 2 − 3 5
Hint: A6 + B6 =2– – = = = 1.
= (A2 + B2) [(A2 + B2)2 – 3A2 B2]. 5 5 5 5

76 M A T H E M A T I C S – X
10. Given expression
1 2
= 8 3 cosec2 30°. sin 60°. cos 60°. cos2 45°. 2×
2 tan 30° 3 3
2. (C) = = 3 = .
sin 45°. tan 30°. cosec3 45°. 1 + tan 2 30°  1 
2 4 2
1+   3
1  3
= 8 3 × . sin (90° – 30°).
2
sin 30°
3. (D)
cos (90° – 30°) cos2 (90° – 45°). sin 45°.
15
sin 30° 1 Hint: tan x =
3 8
cos 30° sin 45°
15 8
1 ⇒ sin x = , cos x =
= 8 3× × cos 30°. sin 30°. sin2 45°. 17 17
sin 2 30°
225 64 161
sin 30° 1 ∴ sin2 x – cos2 x = – = .
sin 45°. × 289 289 289
3
cos 30° sin 45°
sin 30° + tan 45° – cosec 60°
4. (A)
 sin 30°. sin 30°  cos 30° sec 30°+ cos 60° + cot 45°
= 8 3 ×   ×
 sin2 30°  cos 30° 1 2
+1–
sin 2 45° sin 45° =
2 3 = 3 +2 3 –4
× 2 1 4+ 3 +2 3
sin 3 45° + +1
3 2
=8 3 × 1 × 1 × 1 = 8 3 .
3 3 –4 3 3 –4
OR = ×
3 3+4 3 3 –4
1 1 27 + 16 – 24 3 43 – 24 3
Hint: sec2 θ = x 2 + 2
+ = = .
16 x 2 27 – 16 11
1 1 5. ∵ A + B + C = 180º
... sec2 θ – 1 = x 2 + 2

16 x 2 C+A 180º – B
∴ LHS = cot = cot
2 2
2
1 
... tan2 θ =  x −  B
= cot (90º – ) = tan
B
 4x  2 2
= RHS.
 1  6. Yes.
⇒ tan θ = ±  x −  .
 4 x
7
Hint: Both sides = .
25
WORKSHEET–54
7. LHS = (cosec A – sin A) (sec A – cos A)
3  1  1 
1. (B) As sin A = , = – sin A   – cos A 
4  sin A   cos A 
let BC = 3x and CA = 4x
1 – sin2 A 1 – cos2 A
= ×
∴ AB = ( 4 x )2 – (3 x )2 = 7 x sin A cos A
cos 2 A × sin 2 A
BC 3x 3 =
Now, tan A = = = . sin A cos A
AB 7x 7
= sin A cos A .... (i)

I N T R O D U C T I O N T O T R I G O N O M E T R Y 77
1 1 2. (C) sin θ =
RHS = = 1 – cos 2 θ = 1 − 0.36 = 0.8
tan A + cot A sin A cos A
+ sin θ 0.8 4
cos A sin A And tan θ = = =
cos θ 0.6 3
sin A cos A
= 4
sin 2 A + cos 2 A Now, 5 sin θ – 3 tan θ = 5 × 0.8 – 3 × =0
3
= sin A cos A ....(ii) 3. (D)
2 2
(∵ sin A + cos A = 1) Hint: Divide numerator and denominator
From equations (i) and (ii) we obtain by sin A.
LHS = RHS. 3
1 + cot A 1+
= 4 .
8. 7 sin2 θ + 3(1 – sin2 θ) = 4 3
1 – cot A
Let sin θ = x 1–
4
∴ 7x2 + 3 – 3x2 = 4
2
1 1 4. (C) sec A = ⇒ sec A = sec 30° ⇒ A = 30°
⇒ 4x2 = 1 ⇒ x2 = ⇒ x=± 3
4 2
⇒ A + B = 90° ⇒ B = 90°– 30° = 60°
1 −1
∴ sin θ = or sin θ = 2
2 2 Now, cosec B = cosec 60° =
1 3
sin θ = – is not possible as θ is acute. 5. (A) Given expression
2
2 2
3  1   2 
( )
2
⇒ cosec θ = 2 ∴ cos θ = 3 +4   + 3  + 5× 0
2  2  3
=
2
( 3)
2
∴ sec θ + cosec θ = + 2 . Hence proved 2+2–
3
9. 1 3+2+4
Hint: cos (40º + θ) = sin {90° – (40° + θ)} = = 9.
4–3
= sin (50° – θ) 6. False.
and cos 40° = sin 50°. Hint: ∠A = 30º, ∠B = 60º.
10. LHS = m2 – n2 = (tan θ + sin θ)2
– (tan θ – sin θ)2 = 4sin θ tan θ ...(i) sec θ – cosec θ
7.
RHS. sec θ + cosec θ

sin 2 θ 1 1
= 4 mn = 4 – 1 = 4 sin θ sec2 θ –1 –
cos2 θ = cos θ sin θ = sin θ − cos θ
1 1 sin θ + cos θ
= 4 sin θ tan θ .... (ii) +
cos θ sin θ
From (i) and (ii), LHS = RHS.
1
WORKSHEET–55 (sin θ − cos θ ) ×
sin θ 1 – cot θ
= =
1 1 + cot θ
 64
+ 2×
36 8 
– 
(sin θ + cos θ) ×
1. (A) Required value = 25  sin θ
 100 100 6 
1 3
= 25 × (192 + 216 – 400) 1–
300 4 = 1
= . Hence proved.
3 7
1 2 1+
= ×8= . 4
12 3
78 M A T H E M A T I C S – X
cos θ − sin θ cos θ − sin θ 1
8. = cos 38° ×
cos θ + sin θ cos θ + sin θ = 2– cos 38°
cot θ – 1 tan 18° tan 35° tan 60° ×
= . 1 1
cot θ + 1 ×
(Dividing numerator and denominator by tan 18° tan 35°
sin θ) 1
=2–
p tan 60°
–1
q p–q
= = . 1 2 3 –1 3 6– 3
p p+q =2– = × = .
+1 3 3 3 3
q
10. tan A = n tan B
9. Given expression
n
2 2 ⇒ cot B = and sin A = m sin B
sin 35°   cos 55° 
=   +  – 2cos 60°
tan A
 cos 55°   sin 35°  1 m
⇒ sin B = sin A ⇒ cosec B =
2 2 m sin A
 sin 35°   cos 55°  .. . cosec2 B – cot2 B = 1
=   +  
 cos ( 90° − 35° )   sin ( 90° − 55° ) 
– 2cos 60° m2 n2
⇒ − =1
2 2
sin 2 A tan 2 A
sin 35°   cos 55° 
=   +   – 2cos 60°
 sin 35°  cos 55°  m2 − n2 cos2 A
⇒ =1
sin 2 A
= 1 + 1 – 2 cos 60°
⇒ m2 – 1 = (n2 – 1) cos2 A
1
= 2−2 × = 2 – 1 = 1 m2 − 1
2 ⇒ = cos2 A.
OR n2 – 1
Given expression Hence proved
cos 58° sin 22° OR
= +
sin 32° cos 68° Consider an equilateral triangle PQR in
cos 38°cosec 52° which PS ⊥ QR. Since PS ⊥
– QR so PS bisects ∠P as well
tan 18° tan 35° tan 60° tan 72° tan 55°
as base QR.
cos 58° sin 22° We observe that ∆PQS is a
= + right triangle, right-angled at
sin (90° – 58° ) cos (90° – 22° )
S with ∠QPS = 30° and ∠PQS = 60°.
cos 38°cosec ( 90° – 38° ) For finding the trigonometric ratios, we need
– to know the length of the sides of the
tan 18° tan 35° tan 60° tan ( 90° – 18° ) triangle. So, let us suppose PQ = x
tan (90° – 35° ) 1 x
Then , QS = QR =
2 2
cos 58° sin 22°
= + x2 3x 2
cos 58° sin 22° and (PS)2 = (PQ)2 – (QS)2 = x2 – =
4 4
cos 38° sec 36°
– 3x
tan 18° tan 35° tan 60° cot 18° cot 35° ∴ PS =
4

I N T R O D U C T I O N T O T R I G O N O M E T R Y 79
x 2
QS 1 sin 29°  cos 27° 
(i) cos 60° = = 2 = =
cos (90° − 29°)
+   –4
PQ x 2  sin ( 90° − 29°) 
2
 1 
3x ×  
PS 2 = 3  2
(ii) sin 60° = =
PQ x 2 4
= 1 + 12 – = 0.
2
x
QS 5. (B) Given expression
(iii) tan 30° = = 2 = 1 .
PS 3x 3 2
 1   1    1  
4 2 2
2  3 
2 = 4   +    –3   − (1)  –  
 2   2    2    
2

WORKSHEET–56  1 1 1  3
=4  +  – 3  – 1 –
1. (B) b2x2 + a2y2 = b2a2 cos2 θ + a2b2 sin2 θ  16 4  2  4
= a2b2. 1 3 3
= +1– +3–
2. (A) A = 90° – 60° = 30° 4 2 4
∴ cosec A = cosec 30° = 2. 9
17 8
= – = = 2.
12 122 4 4 4
3. (C) tan θ = ⇒ 1 + tan2 θ = 1 + 2
5 5
1 sin A cos A sin A
13 6. + = +
⇒ sec θ = tan A 1 + cos A sin A 1 + cos A
5
cos A + cos 2 A + sin 2 A
1 + sin θ =
1 + sin θ sin A (1 + cos A )
cos θ sec θ + tan θ
Now, = =
1 – sin θ 1 − sin θ sec θ – tan θ 1 + cos A
cos θ = = cosec A
sin A (1 + cos A )
13 12 25 = 2.
+
5 5
= = 5 = 25 7. ∵ sin θ =
3
∴ cosec θ =
4
13 12 1 3
– 4
5 5 5
9 7
12 ∵ cos θ = 1 – sin2 θ = 1– =
tan θ = 16 4
5
1 1 4 7
sin θ = = ∴ sec θ = and cot θ =
cosec θ 7 3
1 + cos2 θ
Now, LHS
1 12
= = . 16 7 9
2 13 cosec 2 θ – cot 2 θ –
 5 9 9 = 9
1+  = =
 12  sec 2 θ – 1 16 9
–1
7 7
2
 sin 29°   cos 27°  2 7 7
4. (A)   +   – 4 cos 45° = = = RHS. Hence Proved.
 cos 61°   sin 63°  9 3

80 M A T H E M A T I C S – X
2
1 1 x−y
8. Hint: LHS = – ∴ 
2
 = sin A
1 cos A sin A
– x+y
sin A sin A
2
sin A 1 and  x − y  = cos2 A
= –  
1 – cos A sin A  2 
∴ sin2 A + cos2 A = 1.
sin 2 A – 1 + cos A
=
sin A (1 – cos A ) 2. (A)

1 – cos 2 A – 1 + cos A Hint: (x + 1)2 = x2 + 52


=
sin A (1 – cos A )
1
cos A (1 – cos A ) 3. (A) tan A = = tan 30°
= = cot A. 3
sin A (1 – cos A )
⇒ ∴ ∠A = 30°
OR ∴ ∠C = 180° – A – B = 180° – 120°
Hint: Using a3 + b3 = (a2 + b2 – ab) (a + b)
= 60°
we get
Now, sin A cos C + cos A sin C
sin 3 θ+ cos3 θ = sin 30° cos 60° + cos 30° sin 60°
+ sin θ . cos θ
sin θ+ cos θ
1 1 3 3
(sin θ + cos θ)(sin 2 θ + cos2 θ − sin θ . cos θ) = × + × = 1.
= 2 2 2 2
sin θ + cos θ
+ sin θ ⋅ cos θ 1
4. (C) cos α = ⇒ cos α = cos 60°
= 1 − sin θ . cos θ + sin θ . cos θ = 1 . 2
⇒ α = 60°
9.
( 2 + 2 sin θ )(1 – sin θ )
1
(1 + cos θ)( 2 – 2 cos θ) tan β = ⇒ tan β = tan 30°
3
2 (1 + sin θ )(1 – sin θ )
= ⇒ β = 30°.
2 (1 + cos θ )(1 – cos θ )
Now, sin (α + β) = sin (60° + 30°) = sin 90° = 1.
1 – sin 2 θ cos 2 θ
= = = cot2 θ 5. (B) tan 1° tan 2°.... tan 43° tan 44° tan 45°
1 – cos 2 θ sin 2 θ tan 46° tan 47°..... tan 88° tan 89°
2
225
=  15  = . = (tan 1° tan 89°)(tan 2° tan 88°)....(tan 43°
 8 64 tan 47°)(tan 44° tan 46°) tan 45°
10. Hint: p2 – 1 = sec2 θ + tan2 θ + 2 sec θ tan θ – 1 = (tan 1° cot 1°)(tan 2° cot 2°)....(tan 43°
= 1 + tan2 θ + tan2 θ + 2 sec θ tan θ – 1 cot 43°)(tan 44° cot 44°) tan 45°
= 2 tan θ (tan θ + sec θ) = (1) × (1) × .... × (1) × (1) × tan 45°
Similarly p2 + 1 = 2 sec θ (tan θ + sec θ).
= (1 × 1 ×....× 1 × 1) × tan 45°
WORKSHEET–57 = 1 × 1 = 1.
6. Given expression
1. (B)
Hint: x + y = 2 cot A tan 50° + sec 50°
= + cos 40° cosec 50°
x – y = 2 cos A cot 40° + cosec 40°

I N T R O D U C T I O N T O T R I G O N O M E T R Y 81
tan 50° + sec 50° ⇒ sin2 θ + cos2 θ – 2 cos θ sin θ = sin2 θ
= + + sin2 θ
cot (90° − 50°) + cosec (90° − 50°)
⇒ (cos θ – sin θ)2 = 2 sin2 θ
cos 40°cosec (90° – 40°)
⇒ cos θ – sin θ = 2 sin θ Hence proved.
tan 50° + sec 50° 1
= + cos 40°. 10. Hint: l tan θ + m sec θ = n ...(i) × l′
tan 50° + sec 50° cos 40°
l′ tan θ – m′ sec θ = n′ ...(ii) × l
= 1 + 1= 2.
⇒ ll′ tan θ + ml′ sec θ = nl′
7. LHS l′ l tan θ – m′ l sec θ = n′ l
= tan (A – B) = tan (60° – 30°) = tan 30° – + –
1 (m′ l + ml′) sec θ = nl′ – n′ l
= .
3
nl' − n'l
RHS ⇒ sec θ =
m'l + ml'
tan A − tan B tan 60° − tan 30°
= = nm' + mn'
1 + tan A ⋅ tan B 1 + tan 60° ⋅ tan 30° Similarly, tan θ = .
lm' + ml'
1 3–1 2
3– WORKSHEET–58
3 3 = 3
= =
1 1+1 2
1+ 3 . 1. (D) Given expression
3
cos2 (90 − 70°) + cos2 70°
1 =
= = LHS. Hence verified. sec 2 (90° − 40°) − cot 2 40°
3
+ 2 {cosec2 58° – cot 58° tan (90° – 58°)}
sin6 θ 3 sin 2 θ 1
8. RHS = + . +1
6
cos θ cos θ cos2
2 sin 2 70° + cos2 70°
=
cosec 2 40° – cot 2 40°
sin 6 θ + cos6 θ + 3 sin 2 θ cos 2 θ
= + 2 (cosec2 58° – cot2 58°)
cos6 θ
1
= + 2(1) = 1 + 2 = 3.
( sin 2
θ + cos θ )
2 3 1
= 2. (A) sec 5A = cosec (A – 36°)
cos 6 θ
⇒ sec 5A = sec {90° – (A – 36°)}
= sec6 θ = LHS. [∵ sin 2 θ +cos2 θ = 1] 5A = – A + 126° ⇒ A = 21°.
OR 3. (D) Given expression
Hint: Numerator of = sin2 5° + sin2 10° ... + sin2 40° + sin2 45°
LHS = tan θ + sec θ – (sec2 θ – tan2 θ) + sin2 50° + ... + sin2 80° + sin2 85° + sin2 90°
= (tan θ + sec θ) – (tan θ + sec θ) (sec θ – tan θ) 2
= (tan θ + sec θ) (1 – sec θ + tan θ).  1 
= cos2 85° + cos2 80° + .... + cos2 50° +  
 2
9. cos θ + sin θ = 2 cos θ
+ sin2 50° + .... + sin2 80° + sin2 85° + (1)2
Squaring both sides, we get = (cos2 85° + sin2 85°) + (cos2 80° + sin2 80°)
cos2 θ + 2 cos θ sin θ + sin2 θ = 2 cos2 θ 1
⇒ 2 cos2 θ – cos2 θ – 2 cos θ sin θ = sin2 θ + .... + (cos2 50° + sin2 50°) + +1
2
⇒ cos2 θ – 2 cos θ sin θ = sin2 θ 1
= (1 + 1 + .... 8 terms) + +1
Adding sin2 θ to both sides , we have 2

82 M A T H E M A T I C S – X
1 1 [(a2 − b2)2 − (a2 + b2)2] tan2 θ + 4 a2b2 + 4 ab
=8+ +1= 9 .
2 2 (a2 − b2) tan θ − (a2 + b2)2 = 0
1 1 1 1 1 ⇒ – 4a b tan θ + 4ab (a2 – b2) tan θ − a4 − b4
2 2 2
4. (A) tan 3x = . + = + =1 + 2a2b2 = 0
2 2 2 2 2
2 2 2 2 2
− 4a b tan θ + 4ab (a − b ) tan θ
45° − (a2 − b2)2 = 0
⇒ tan 3x = tan 45° ⇒ x = = 15°.
3
4a b tan θ − 4ab (a − b ) tan θ + (a2 − b2)2 = 0
2 2 2 2 2

1 ⇒ [2ab tan θ − (a2 − b2)]2 = 0


5. (B) cosec A = 2 ⇒ sin A =
2 ⇒ 2ab tan θ = a2 − b2
1 1 a2 − b2
cos A = 1 – sin 2 A = 1– = ⇒ tan θ = .
2 2 2 ab
tan A = 1, cot A = 1 8. Hint: Use (a2 + b2)3 = a6 + b6 + 3a2b2 (a2 + b2).
1 9. LHS
2× +3×1
2 sin 2 A + 3 cot 2 A 2
Now, = (1+ cot A + tan A )( sin A – cos A )
(
4 tan 2 A – cos 2 A )  1
41 − 
=
sec 3 A – cosec 3 A
 2
 cos A sin A 
=
4
= 2. 1 + +  (sin A – cos A )
 sin A cos A
2 =
6. True
1 1

Hint: cos A sin 3 A
3

a cos θ + b sin θ = 4 ...(i) × sin θ 1


a sin θ – b cos θ = 3 ...(ii) × cos θ (sin A cos A + cos 2 A + sin 2 A)
sin A cos A
⇒ a cos θ sin θ + b sin2 θ = 4 sin θ (sin A − cos A)
a sin θ cos θ – b cos2 θ = 3 cos θ =
1
– + – (sin A − cos A)
sin 3 A cos 3 A
b = 4 sin θ – 3 cos θ = (sin2 A + cos2 A + sin A cos A)
Similarly, a = 4 cos θ + 3 sin θ = sin2 A . cos2 A = RHS. Hence proved.
∴ a2 + b 2 = 16 sin2 θ + 9 cos2 θ – 12 sin θ 1
cos θ + 16 cos2 θ + 9 sin2 θ + 12 sin θ cos θ 10. m = cosec θ – sin θ = – sin θ
sin θ
= 16 + 9 = 25.
2
1 – sin2 θ cos θ
7. (a2 − b2) sin θ + 2ab . cos θ = a2 + b2 = =
sin θ sin θ
Divide by cos θ
1
n = sec θ – cos θ = – cos θ
a2 + b2 cosθ
(a2 − b2) tan θ + 2ab =
cos θ 1 – cos2 θ sin 2 θ
⇒ (a2 − b2) tan θ + 2ab = (a2 + b2) . sec θ = =
cos θ cos θ
= (a2 + b2 . 1 + tan 2 θ 2 2

Squaring both sides:


Now, LHS = m 2 n( ) + (mn )
3 2 3

(a2 − b2)2 tan2 θ + 4 a2b2 + 4 ab (a2 − b2) tan θ 2 2

= (a2 + b2)2 (1 + tan2 θ)  cos 4 θ sin 2 θ  3  cos 2 θ sin 4 θ  3


=  2
×  +  × 2  
= (a2 + b2)2 + (a2 + b2)2 tan2 θ  sin θ cos θ   sin θ cos θ 

I N T R O D U C T I O N T O T R I G O N O M E T R Y 83
2 4. True, because LHS = tan 60° =
2
3 and
(
= cos 3 θ ) 3
(
+ sin 3 θ ) 3
2
= cos2 θ + sin2 θ = 1 = RHS. 2 tan 30° 3 =
RHS = = 3.
OR 1 – tan 2 30° 1
1+
LHS 3
= (1 + cot A – cosec A)(1 + tan A + sec A) 5. 2 (sin A + cos A) – 3 (cos4 A + sin4 A) + 1
6 6

 cos A 1   sin A 1  = 2{(sin2 A + cos2 A)3 – 3sin2 A cos2A


= 1 + −  1+ +
 sin A sin A   
cos A cos A  (sin2 A + cos2 A)}
sin A + cos A –1 cos A + sin A + 1 – 3{(sin A + cos A) – 2sin2 A cos2 A} + 1
2 2 2
= ×
sin A cos A [∵ (a + b)3 = a3 + b3 + 3ab(a + b)
and (a + b)2 = a2 + b2 + 2ab]
=
(sin A + cos A )2 – 12
= 2(1 – 3sin2 A cos2 A) – 3(1 – 2sin2 A
sin A cos A
cos2 A) + 1
sin A + 2 sin A cos A + cos 2 A – 1
2
[∵ sin A + cos2 A = 1]
2
=
sin A cos A = 2 – 6 sin2 A cos2 A – 3 + 6 sin2 A cos2 A
2 sin A cos A +1=0
= =2
sin A cos A 6. 2sin x + cos y = 1 (Given)
= RHS. Hence proved. ⇒ 2sin x + cos y = 20
⇒ sin x + cos y = 0 ... (i)
ASSESSMENT SHEET–9 sin 2 x + cos 2 y =4 (Given)
16

(4 )
2 2
2 sin
x + cos y
BC 3 10 ⇒ =4
1. (B) cos 30° = ⇒ =
AC 2 x
⇒ 4
(
2 sin 2 x + cos 2 y
= 41
)
20 20 3
⇒ x= ⇒x= cm ⇒ 2(sin2 x + cos2 y) = 1 ... (ii)
3 3
Substituting cos y = – sin x from (i) in (ii),
AB 1 y×3
sin 30° = ⇒ = 1
AC 2 20 3 we get 2(sin2 x + sin2 x) = 1 ⇒ sin2 x =
4
10 3 1
⇒ y= cm. ⇒ sin x = ±
3 2
2. (D) tan 2θ = cot (θ + 15°) 1 1
When sin x = – , cos y =
⇒ tan 2θ = tan [90°– (θ + 15°)]. 2 2
3. We know: sin θ = cos (90° – θ) 1 1
When sin x = , cos y = –
so, given expression 2 2
1 1 1
{ }
2 2
cos(90° − 35°)  cos 55°  Hence, sin x = – , cos y = or sin x = ,
= +   2 2 2
cos 55°  cos(90° − 35°) 
1
– 2 cos 60° .
cos y = –
2
2 2
 cos 55°   cos 55°  1 7. We know that
=   +   –2×
 cos 55 °   cos 55 °  2 sin (90°– θ) = cos θ, tan (90°– θ) = cot θ,
= 1 + 1 – 1 = 1. sec(90°– θ) = cosec θ

84 M A T H E M A T I C S – X
sec 2 (90° – θ) – cot 2 θ ASSESSMENT SHEET– 10
Now,
2(sin 2 25° + sin 2 65°)
1. (A) Given expression
2 2 2 = sin 25° cos (90° – 25°) + cos 25°
2 cos 60° tan 28° tan 62°
+
3(sec 2 43° – cot 2 47°) sin (90°– 25°)
2 2
= sin 25° + cos 25° = 1.
sec 2 (90° − θ) − cot 2 θ 2 sin θ cos θ
=
{
2 sin 2 25° + cos 2 ( 90° − 25°) } 2. (B)
2 sin θ – cos θ
=
cos θ

cos θ
2 sin θ + cos θ 2 sin θ cos θ
+
2 cos2 60° + tan 2 28° tan 2 (90° − 28°) cos θ cos θ
+
{
3 sec 2 43° − cot 2 ( 90° − 43°) } 4
2× – 1
2 tan θ – 1 3 5
= = = .
cosec 2 θ − cot 2 θ 2 tan θ + 1 4 11
= 2× + 1
(
2 sin 2 25° + cos 2 25° ) 3
1
2cos2 60° + tan 2 28° . cot 2 28° 3.
cos 45°
= 2
+ 2
3(sec 2 43° − tan 2 43°) sec 30° + cosec 30°
+2
3
1 1
2× ×1
1 4
= + 2 1 3
2×1 3×1 = = ×
(
2 1+ 3 ) 2 2 1+ 3 ( )
1 1 2
= + = . 3
2 6 3
8. Given equation are:
=
3× ( 3 –1 ) =
3– 3
×
2
sin θ + cos θ = p ... (i) 2 2 ( 3 +1 )( 3 –1 ) 4 2 2
sec θ + cosec θ = q ... (ii)
3 2– 6
Squaring both the sides of equation (i), = .
we get 8
sin2 θ + cos2 θ + 2sin θ cos θ = p2 4. False, because cos2 23° – sin2 67° = 0, 0 is
Subtract unity from both the sides to get not a positive value.
p2 – 1 = 2sin θ cos θ ... (iii) cos A 1 + sin A
5. LHS = +
Equation (ii) can be written as 1+ sin A cos A
1 1 cos2 A + (1 + sinA)
2
q= +
cos θ sin θ =
(1 + sin A) cos A
sin θ + cos θ
⇒ q= ... (iv) cos2 A + 1 + sin 2 A + 2sin A
sin θ cos θ =
(1 + sin A) cos A
From equations (iii) and (iv), we get
2 + 2 sin A 2 (1 + sin A )
sin θ + cos θ = =
q (p2 – 1) = × 2sin θ cos θ (1 + sin A ) cos A (1 + sin A) cos A
sin θ cos θ
2
⇒ q (p2 – 1) = 2(sin θ + cos θ) = = 2 sec A
cos A
⇒ q (p2 – 1) = 2p. Hence proved. = RHS. Hence proved.

I N T R O D U C T I O N T O T R I G O N O M E T R Y 85
6. Let us construct a triangle ABC in which 8. Since ABC is a acute angled triangle
AB = BC = AC = a (say). Draw AD ⊥ BC. so, ∠A < 90°, ∠B < 90° and ∠C < 90°.
AD bisects BC Also ∠A + ∠B + ∠C = 180° ....(i )
a 1°
⇒ BD = DC = sin (A + B – C) = (Given)
2 2
AD bisects ∠BAC ⇒ sin (A + B – C) = sin 30°
⇒ θ = 30° ⇒ ∠A + ∠B – ∠C = 30° .... (ii )
In right angled ∆ABD. Similarly, ∠B + ∠C – ∠A = 45° .... (iii )
2
a
AD2 = AB2 – BD2 = a2 –   Add equations (ii ) and (iii ) to get
2 1°
a2 3a2 2∠B = 75° ⇒ ∠B = 37
2
=a – = 2
4 4 Subtract equation (ii) from equation (i) to get
3 2∠C = 150° ⇒ ∠C = 75°
⇒ AD = a
2 Subtract equation (iii) from equation (i) to get
Now, in ∆ ABD, 1°
a
2∠A = 135° ⇒ ∠A = 67
BD 2 2
tan θ = ⇒ tan 30° =
AD a
3 1° 1°
2 Thus, ∠A = 67 , ∠B = 37 and ∠C = 75°.
1 2 2
⇒ tan 30° = .
3
CHAPTER TEST
7. ( a2 – b2) sin θ + 2ab cos θ = a2 + b2 (Given)
Divide both sides by cosθ to get sec θ 1 tan θ
1. (A) x = and =
( a2 – b2) tan θ + 2ab = (a2 + b2) sec θ 2 x 2
Squaring both sides, we get
 1  sec 2 θ tan 2 θ 
(a2 – b2)2 tan2 θ + 4a2 b2 + 4ab(a2 – b2) tan θ ∴ 2  x2 – 2  = 2  – 
 x   4 4 
= (a2 + b2)2 sec2 θ
⇒ (a2 – b2)2 tan2 θ – (a2 + b2)2 tan2 θ + 4ab  sec 2 θ – tan 2 θ  1
= 2  = .
(a2 – b2) tan θ – (a2 + b2)2 + 4a2b2 = 0  4 2
 
(∵∵ sec2 θ = 1 + tan2 θ)
⇒ – 4a b tan θ + 4ab (a2 – b2) tan θ
2 2 2 cos 2 20° + cos 2 70° 2
2. (C) Hint. =
– (a2 – b2)2 = 0 ( 2 2
2 sin 59 + sin 31° ) k
⇒ – 4a b x + 4ab (a – b ) x – (a2 – b2)2 = 0
2 2 2 2 2 2 2
sin 70° + cos 70° 2
where x = tan θ =
This is a quadratic equation in x. ( 2
2 sin 59° + cos 59°2
) k
Here, discriminant, 1 2
= ⇒ k = 4.
2 k
( ) ( )
2 2
D = 16a2b 2 a2 − b 2 − 4 × 4 a 2b 2 a 2 − b 2
3. (C) sin4 θ + cos4 θ = 1 + 4k sin2 θ cos2 θ
=0 ⇒ (sin2 θ + cos2 θ)2 – 2 sin2 θ cos2 θ
− 4 ab( a2 − b2 ) − 0 a2 − b2 = 1 + 4k sin2 θ cos2 θ
∴x= =
2 × ( − 4 a2 b 2 ) 2 ab ⇒ 2 sin2 θ cos2 θ (– 1 – 2k) = 0

a2 − b2 1
⇒ – 1 – 2k = 0 ⇒ k = – .
⇒ tan θ = . Hence proved. 2
2 ab

86 M A T H E M A T I C S – X
4. tan θ = 4 [Using equation (i)]
⇒ tan2 θ + 1 = 42 + 1
3 2 4 – 3 (a 2 – 1)2
( )
2
⇒ sec2 θ= = 1– a –1 = .
17 4 4
1 1 Hence proved.
∴ (tan2 θ + 2 sec2 θ) = (16 + 2 × 17)
10 10
= 5. (sec θ + tan θ )2 – 1
9. LHS =
5. False. (sec θ + tan θ )2 + 1
Suppose A = 30° and B = 60°
Then, LHS = tan (A + B) = tan (30° + 60°) sec 2 θ + tan 2 θ + 2 sec θ tan θ – 1
=
= tan 90° sec 2 θ + tan 2 θ + 2 sec θ tan θ + 1
⇒ LHS = undefined .... (i)
and RHS = tan A + tan B = tan 30°
=
(sec 2
)
θ − 1 + tan 2 θ + 2 sec θ tan θ
+ tan 60° ( )
sec θ + tan 2 θ + 1 + 2 sec θ tan θ
2

1 1+ 3 4
= + 3 = =
3 3 3 tan 2 θ + tan 2 θ + 2 sec θ tan θ
=
⇒ RHS = a real number .... (ii) sec 2 θ + sec 2 θ + 2 sec θ tan θ
From results (i) and (ii), it is clear that the 2 tan θ (tan θ + sec θ)
given identity is false. =
2 sec θ (sec θ + tan θ)
–1
6. tan θ
7 = = tan θ cos θ
sec θ
Hint: cos 55° = cos (90° – 35°) = sin 35°
cos 70° = sin 20° sin θ
= . cos θ = sin θ = RHS.
and tan 5° = cot 85°. cos θ
Hence proved.
7. 13 . OR
4
sin A – sin B cos A – cos B
1 3 +
Hint: sin 30° = = cos 60°, sin 60° = , cos A + cos B sin A + sin B
2 2

cos 45° =
1
= sin 45°, sin 90° = 1. (sin A – sin B )(sin A + sin B )
2 + (cos A – cos B )( cos A + cos B )
=
8. sin θ + cos θ = a (cos A + cos B )(sin A + sin B )
Square both sides.
sin 2 A – sin 2 B + cos2 A – cos2 B
sin2 θ + cos2 θ + 2 sin θ cos θ = a2 =
⇒ 2 sin θ cos θ = a2 – 1
(cos A + cos B )(sin A + sin B)

⇒ sin θ cos θ =
a2 – 1
... (i)
=
(sin 2
A + cos 2 A) − (sin 2 B + cos 2 B )
2
(cos A + cos B)(sin A + sin B)
( )
3
Now, sin6 θ + cos6 θ = sin2 θ + cos2 θ
1–1
–3 sin2 θ cos2 θ (sin2 θ + cos2 θ) =
(cos A + cos B )(sin A + sin B )
2
 a2 – 1  = 0 which is an integer.
= 13 – 3   (1)
 2  ❑❑

I N T R O D U C T I O N T O T R I G O N O M E T R Y 87
Chapter

6 STATISTICS

WORKSHEET–62 Now, l = 30, f1 = 32, f0 = 12, f2 = 20, h = 10


Use the formula:
1. (B)  f1 – f0 
Hint: 3 Median = Mode + 2 Mean. Mode = l +   × h.
 2 f1 – f0 – f2 
2. (C) Since the mode is 7
∴ 2k – 1 = 7 ⇒ k = 4. 7. Hint:
Production No. of Production c.f.
3. In such case, mean will increase by 3.
yield farms yield
∴ New mean = 18 + 3 = 21.
50-55 2 more than 50 100
Σf i x i 55-60 8 more than 55 98
4. Hint: Mean = .
Σf i 60-65 12 more than 60 90
5. 65-70 24 more than 65 78
Class Cumulative 70-75 38 more than 70 54
Frequency ( f )
interval Frequency (cf ) 75-80 16 more than 75 16
Total 100 more than 80 0
0-8 8 8 ∴ For more than ogive plot following points.
8-16 10 18 (50, 100), (55, 98), (60, 90), (65, 78), (70, 54),
16-24 16 34 (75, 16), (80, 0).
24-32 24 58
32-40 15 73 WORKSHEET–63
40-48 7 80 1. (B) 2. (B) median.
N = 80 3. The given distribution can be represented as:
N 80 Marks obtained No. of students
For median class, = = 40
2 2 0-10 5
In the cumulative frequency column, 58 is 10-20 3
just greater than 40.
20-30 4
So, 24-32 is the median class. 30-40 3
N 40-50 6
Here, l = 24, cf = 34, f = 24, = 40, h = 8
2 More than 50 42
Using formula:
Clearly, the frequency of the class 30-40 is 3.
N 
 – cf 
Median = l +  2  × h 4. Let us rewrite the given table with cumu-
 f  lative frequencies.
  Class interval f cf
 40 – 34  0-5 10 10
= 24 +  × 8 = 26
 24  5-10 15 25
Hence, median of the given distribution is 26. 10-15 12 37
6. 36.25 15-20 20 57
Hint: Here maximum class frequency is 32. 20-25 9 66
So, the modal class is 30-40. N = 66

88 M A T H E M A T I C S – X
∵ N = 66
N
∴ = 33
2
∴ Median class = 10-15
Mode class = 15-20
Required sum = 10 + 15 = 25.
5. In the given distribution, maximum class
frequency is 20, so the modal class is 40-50.
Here, lower limit of modal class: l = 40
Frequency of the modal class: f1 = 20
Frequency of the class preceding the modal
class: f0 = 12
Frequency of the class succeeding the modal
class: f2 = 11
Size of class: h = 10
Using the formula:
 f1 – f0 
Mode = l +  × h
 2 f1 – f0 – f2 
To obtain median from the graph:
 20 – 12 
= 40 +  × 10 We first locate the point corresponding to
 2 × 20 – 12 – 11 
N 35
= 40 + 4.70 = 44.70. = = 17.5 students on the y-axis. From
2 2
Hence, mode of the given data is 45 cars. this point, draw a line parallel to the x-axis
6. Let us rewritten the table with class intervals. to cut the curve at P. From the point P,
draw a perpendicular PQ on the x-axis to
Class interval f cf
meet it at Q. The x-coordinate of Q is 46.5.
36-38 0 0 Hence, the median is 46.5 kg.
38-40 3 3
Let us verify this median using the formula.
40-42 2 5
42-44 4 9 N 
44-46 5 14 − cf
 2 
Median = l +  ×h
46-48 14 28  f 
48-50 4 32
50-52 3 35  17.5 − 14 
= 46 +  ×2
N = 35  14 
We mark the upper class limits on x-axis
7
and with a suitable scale cumulative = 46 + = 46 + 0.5
14
frequencies on y-axis with a suitable scale.
= 46.5 kg.
We plot the points (38, 0); (40, 3); (42, 5);
(44, 9); (46, 14); (48, 28); (50, 32) and (52, 35). Thus, the median is the same in both
These points are joined by a free hand methods.
smooth curve to obtain a less than type ogive 7. In the given data, the class intervals are
as shown in the figure. formed by exclusive method. But we can

S T A T I S T LI C S 89
convert the series into inclusive form as: Let us assumed mean be a = 52 and

h = 20

Using the formula:

Σfi ui
Mean = a + ×h
Σfi

– 2.8 – 1.1 f1 + 0.9 f 2


⇒ 50 = 52 + × 20
120
Let the assumed mean be a = 57 and h = 3
Now, using the formula: ⇒ 1.1 f1 – 0.9 f2 = 9.2 ...(i)

Σfi ui But
Mean = a + ×h
Σfi 68 + f1 + f2 = 120

25 ⇒ f1 + f2 = 52 ...(ii)
= 57 + ×3
400 Solving (i) and (ii), we obtain
= 57.19.
f1 = 28 and

WORKSHEET–64 f2 = 24.

1. (A) 6. Let us convert the given data into less than


type distribution.
N 5 + 8+ 3+ 2
Hint: = = 9.
2 2 Class Lifetimes
f cf
interval (in hrs.)
2. (D) We have
Mode = 3 Median – 2 Mean 0-20 10 less than 20 10
⇒ 45 = 3 Median – 2 × 27 20-40 35 less than 40 45
⇒ Median = 33. 40-60 52 less than 60 97
3. (C) mid-points of the classes. 60-80 61 less than 80 158
4. Required number of athletes 80-100 38 less than 100 196
= 2 + 4 + 5 + 71 = 82.
100-120 29 less than 120 225
5.
We mark the upper class, limits along the
x-axis with a suitable scale and the cumu-
lative frequencies along the y-axis with a
suitable scale. For this, we plot the points
A(20, 10), B(40, 45), C(60, 97), D(80, 158),
E(100, 196) and F(120, 225) on a graph paper.
These points are joined by a free hand
smooth curve to obtain a less than type ogive
as shown in the following figure.

90 M A T H E M A T I C S – X
7260
∴ Mean = = 145.20.
50
Hence, the mean is Rs. 145.20
Median:

N 
− cf
Median = l +  2  ×h
 f 

N
∵ N = 50, = 25, f = 14, cf = 12,
2
l = 120 and h = 20

 25 − 12 
∴ Median = 120 +   × 20
14 

= 120 + 18.57 = 138.57


Hence, the median is Rs. 138.57.
Mode:

 f1 − f0 
Mode = l +  ×h
 1 − f0 − f2 
2 f

∵ l = 120, f1 = 14, f0 = 12
f2 = 8 and h = 20
7. The given distribution can be again
represented with the cumulative frequencies 14 − 12
 
as given below: ∴ Mode = 120 +  × 20
 2 × 14 − 12 − 8 
Class
fi xi cf fi x i
interval 40
= 120 + = 125
100-120 12 110 12 1320 8
120-140 14 130 26 1820 Hence, the mode is Rs. 125.
140-160 8 150 34 1200 8. Given that median is 28.5. It lies in the class
160-180 6 170 40 1020 interval 20-30, so 20-30 is the median class.
180-200 10 190 50 1900 Further N = 60
⇒ x + y + 45 = 60
50 7260
⇒ x + y = 15 .... (i)
Mean:
Σfi xi N 
Mean =  – cf 
Σfi Median = l +  2 ×h
 f 
∵ Σ fi = 50 and Σfixi = 7260  

S T A T I S T LI C S 91
 30 – ( 5 + x )  4.
⇒ 28.5 = 20 +   × 10
 20  C.I. fi xi fixi
⇒ x=8 ...(ii) 1-3 9 2 18
From equations (i) and (ii), we have 3-5 22 4 88
5-7 27 6 162
x = 8 and y = 7.
7-10 17 8.5 144.5
WORKSHEET–65 Σfi = 75 Σfixi = 412.5
1. (B)
Hint: Σfi xi 412.5
Mean = = = 5.5 .
Σf i 75
Class interval Frequency Cumulative
(C.I.) (f) Frequency 5. In the given distribution, the classes are in
the inclusive form. Let us convert it into
0-10 4 4
10-20 4 8 163 − 162
exclusive form by subtracting , i.e.,
20-30 8 16 2
30-40 10 26 0.5 from lower limit and adding the same to
40-50 12 38 upper limit of each class.
50-60 8 46
Class interval f
60-70 4 50
159.5-162.5 15
2. (A)
Hint: 162.5-165.5 118
Draw a line parallel to the x-axis at the point 165.5-168.5 142
40 168.5-171.5 127
y= = 20. This line cuts the curve at a
2
point. From this point, draw a perpendi- 171.5-174.5 18
cular to the x-axis. The point of intersection
Here, the maximum frequency is 142.
of this perpendicular with the x-axis deter-
mines the median of the data. ∴ l = 165.5, fl = 142, f0 = 118, f2 = 127, h = 3
Now,
3. The given distribution can also be represented
as follows:  f1 − f0 
mode = l +  ×h
Class interval Frequency  2 f1 − f0 − f2 

0-10 3  142 − 118 


= 165.5 +  ×3
10-20 9  284 − 118 − 127 
20-30 15 = 165.5 + 1.85 = 167.35
30-40 30
Hence, the modal height of the students is
40-50 18 167.35 cm.
50-60 5 6. The given data may be re-tabulated by the
As the maximum frequency is 30, the modal following manner with corresponding
class is 30-40. cumulative frequencies.

92 M A T H E M A T I C S – X
Heights (cm.) No. of Girls Cumulative Let assumed mean be A = 60
C.I. ( f) frequency h = 20
(cf )
 Σfi ui 
Below 140 4 4 Mean = A + h × 
 Σfi 
140-145 7 11
145-150 18 29  14 
150-155 11 40 = 60 + 20 ×  
 50 
155-160 6 46
= 60 + 5.6 = 65.6
160-165 5 51
Hence, the required arithmetic mean is 65.6.
N = 51 8. 69.5.
N Hint: Change the given distribution into
Now, N = 51. So, = 25.5.
2 less than type and more than type
This observation lies in the class 145-150. distributions. For drawing the ‘less than
Then l = 145, cf = 11, f = 18, h = 5 type’ ogive, take upper class limits and
corresponding cumulative frequencies; and
N 
 – cf  drawing the ‘more than type’ ogive take
Now, median = l +  2  × h lower class limits and corresponding
 f 
  cumulative frequencies.
 25.5 – 11 
= 145 +   × 5 ASSESSMENT SHEET– 11
 18
= 149.03. 1. (A) Here, a = 25, h = 10.
Hence, the median height of the girls is
149.03 cm.  Σfi ui 
∴ x =a+h 
 Σfi 
7. Class Mid- Fre- ui =
interval values quency xi − A f iu i
(xi) (fi) h  20 
= 25 + 10   = 27.
100 
40
10-30 20 5 − = − 2 – 10 2. (B) Sum of 11 numbers = 11 × 35 = 385
20
20 Sum of first 6 numbers = 6 × 32 = 192
30-50 40 8 − = −1 – 8 Sum of last 6 numbers = 6 × 37 = 222
20
0 ∴ 6th number = 192 + 222 – 385 = 29.
50-70 60 = A 12 =0 0
20 3. The modal class is 30-40.
20 h = 10, f1 = 32, f0 = 12, f2 = 20, l = 30.
70-90 80 20 =1 20
20 f1 − f 0 

Mode = l +   ×h
40
90-110 100 3 =2 6  1 − f0 − f 2 
2 f
20
32 − 12 
60 = 30 +   × 10
110-130 120 2 =3 6  64 − 12 − 20 
20
Σfi = Σfiui = = 30 + 6.25
50 14 = 36.25.

S T A T I S T LI C S 93
4. False, the reason is given below: Marks Number Marks cf Point
N = 5 + 15 + 30 + 8 + 2 = 60 (C.I.) of stu- less
N dents than
∴ = 30
2
Since 30 lies in the interval 30-60, so the 0-10 5 10 5 (10, 5)
median will lie in 30-60. 10-20 8 20 13 (20, 13)
20-30 10 30 23 (30, 23)
5. xi fi f ix i
30-40 9 40 32 (40, 32)
3 5 15 40-50 6 50 38 (50, 38)
6 2 12 50-60 7 60 45 (60, 45)
7 3 21
4 2 8 On plotting these points on a graph paper
p+3 4 4p + 12 and joining them by a free hand smooth
8 6 48 curve, we get a curve called less than ogive.
Σfi = 22 Σfixi = 4p + 116
Σfi xi
Mean =
Σfi
4 p + 116
⇒ 6= ⇒ 132 = 4p + 116
22
⇒ 4p = 16 ⇒ p = 4.

6. Frequency
Class interval
(f)
0-20 4
20-40 6
40-60 18
60-80 8
8. f1 + f2 = 25 ... (i)
80-100 14
as median is 32 which lies in 30-40
The class corresponding to the maximum So median class is 30-40.
frequency is 40-60. So, 40-60 is the modal
class. ∴ l = 30; h = 10; f = 30; N = 100
c.f. = 10 + f1 + 25 = f1 + 35.
 fi − f0 
Mode = l +  ×h
 2 f1 − f0 − f2  ∴ Median = l +
(N/2 – cf )
×h
Here, l = 40, f1 = 18, f0 = 6, f2 = 8 and h = 20 f

 18 − 6 
⇒ f1 = 9 ∴ from (i) ⇒ f2 = 16.
∴ Mode = 40 +  × 20
 2 × 18 − 6 − 8 
ASSESSMENT SHEET–12
12 × 20
= 40 + = 50.91. 1. (D) x1 + x2 + ................ + xn = n × x
22
x x x n
7. We notice classes are continuous. We form ⇒ 1 + 2 + .......... + n = x
cumulative frequency table by less than k k k k
method. (Dividing through by k)

94 M A T H E M A T I C S – X
x1 x2 xn  16 − f 
+ + ......... + ⇒ 36 = 30 +  × 10
⇒ k k k = x  32 − f − 12 
n k
(Dividing throught by n) 6 16 − f
⇒ =
x 10 20 − f
⇒ Required mean = .
k ⇒ 120 – 6f = 160 – 10 f
2. (B) The first ten prime numbers are: ⇒ 4f = 40 ⇒ f = 10.
2, 3, 4, 7, 11, 13, 17, 19, 23, 29. 7. The cumulative frequency table for the given
11 + 13 24 data is given below:
Median = = = 12.
2 2 Marks No. of Cumulative
Σf i xi (C.I.) students frequency
3. Mean = (f) (cf )
Σf i
5 × 6 + 10 × k + 15 × 6 + 20 0-10 10 10
× 10 + 25 × 5 10-20 f1 10 + f1
⇒ 15 =
6 + k + 6 + 10 + 5 20-30 25 35 + f1
445 + 10k 30-40 30 65 + f1
⇒ = 15 ⇒ k = 8.
27 + k 40-50 f2 65 + f1 + f2
50-60 10 72 + f1 + f2
4. False, because the values of these three
measures depends upon the type of data, so N = 75 + f1 + f2
it can be the same.
Clearly, N = 75 + f1 + f2
5. Let us use the assumed mean method to
find the mean of the given data. But N = 100
∴ f1 + f2 = 25 .... (i)
Marks No. of Class di =
(C.I.) students mark xi – 35 fi d i N
∴ = 50.
( fi ) (xi) 2
0-10 4 5 –30 –120 The median is 32 which lies in the class
10-20 6 15 –20 –120 30-40.
20-30 8 25 –10 –80 So, l = 30, f = 30, cf = 35 + f1, h = 10.
Using the formula:
30-40 10 35 0 0
40-50 12 45 10 120  N − cf 
50-60 30 55 20 600  
Median = l +  2 ×h
Σfi = 70 Σfidi = 400  f 
Here, assumed mean, a = 35  50 − 35 − f1 
⇒ 32 = 30 +   × 10
Σf d  30
Now, required mean = a + i i
Σf i
2 15 − f1
400 ⇒ = ⇒ 75 – 5f1 = 30
= 35 + = 35 + 5.71 = 40.71. 10 30
70
6. Since mode = 36, which lies in the class 75 − 30
⇒ f1 = ⇒ f1 = 9
interval 30-40, so the modal class is 30-40. 5
∴ f1 = 16, f0 = f, f2 = 12, l = 30 and h = 10. Substitute f1 = 9 in equation (i) we get
f1 − f0  9 + f2 = 25 ⇒ f2 = 16

Now, mode = l +  ×h Hence, f1 = 9 and f2 = 16.
 1 − f0 − f 2 
2 f

S T A T I S T LI C S 95
8. We prepare the cumulative frequency table
by less than method as given below:
Scores Fre- Score Cumu- Point
quency less lative
than fre-
quency
(f) (f)
200-250 30 250 30 (250, 30)
250-300 15 300 45 (300, 45)
300-350 45 350 90 (350, 90)
350-400 20 400 110 (400, 110)
400-450 25 450 135 (450, 135)
450-500 40 500 175 (500, 175)
500-550 10 550 185 (550, 185)
550-600 15 600 200 (600, 200)
We plot the points given in above table on
a graph paper and then joint them by free
hand smooth curve to draw the cumulative
frequency curve by less than method.
Similarly can be drawn the cumulative
frequency curve by more than method. We
prepare the corresponding frequency table.
Scores Fre- Score Cumula- Point
quency more tive
(f) than fre-
quency CHAPTER TEST
(cf)
200-250 30 200 200 (200, 200) 1. (C) Let us rewrite the given distribution in
250-300 15 250 170 (250, 170) the other manner.
300-350 45 300 155 (300, 155) Marks No. of students
350-400 20 350 110 (350, 110)
0-10 3
400-450 25 400 90 (400, 90)
10-20 9
450-500 40 450 65 (450, 65)
500-550 10 500 25 (500, 25) 20-30 15
550-600 15 550 15 (550, 15) 30-40 30
40-50 18
We plot the points given in this last table on 50-60 5
the same graph and join them by free hand
smooth curve to draw the cumulative Clearly, the modal class is 30-40.
frequency curve by more than method.
2. (A) Let Σfi = N
Median: The two curves intersect each other
a point. From this point, we draw a Σ(fixi – x ) = Σfi xi – N x
perpendicular on the x-axis. The foot of this  Σfi xi 
perpendicular is P(375, 0). The abscissa of =N  − x
 N 
the point P, i.e., 375 is the required median.
Hence, the median is 375. = N ( x – x ) = 0.

96 M A T H E M A T I C S – X
3. Hint: First, find the cumulative frequency Let assumed mean be
table and N = 13 + 10 + 15 + 8 + 11 = 57 A = 850
N h = 20
∴ = 28.5.
2
 Σui fi 
4. Monthly income No. of Mean = A +  ×h
 Σfi 
(in Rs.) families
10000-13000 15 5
13000-16000 16 = 850 +   × 5
 64 
16000-19000 19
19000-22000 17 = 850 + 0.391
22000-25000 18 = 850.391.
Hence, required number of families is 19. Hence, the required mean is 850.391.
5. No, because an ogive is a graphical repre-
sentation of a cumulative frequency distri-  f1 − f0 
8. Mode = l +  ×h
bution.  2 f1 − f0 − f2 
6. Yes; as we know Here, l = 30, f1 = 45,
mode = 3 median – 2 mean
f0 = 30, f2 = 12, h = 10
⇒ 3 median = mode + 2 mean
1 2  45 − 30 
⇒ Median = mode + mean ∴ Mode = 30 +  × 10
3 3  90 − 30 − 12 
2 2
= mode – mode + mean = 30 + 3.125
3 3
= 33.125 marks.
2
= mode + (Mean – Mode). 9. 31.5 marks.
3
Hint:
7. C.I. xi fi ui = f iu i Classes No. of Cumulative
xi − A students frequency
h 0-10 5 5
10-20 8 13
40
800-820 810 7 − = −2 – 14 20-30 6 19
20 30-40 10 29
20 40-50 6 35
820-840 830 14 − = −1 – 14 50-60 6 41
20
Draw the ogive by plotting the points:
0
840-860 850 19 =0 0 (10, 15), (20, 13), (30, 19), (40, 29), (50, 35)
20
N
20 and (60, 41). Here = 20.5. Locate the
860-880 870 15 =1 15 2
20
point on the ogive whose ordinate is 20.5.
40 The x-coordinate of this point will be the
880-900 890 9 =2 18
20 median.

Σfi = 64 Σfiui = 5 ❑❑

S T A T I S T LI C S 97
Solutions to
P RACTICE PAPERS
(SUMMATIVE ASSESSMENTS)
[FIRST TERM]
PRACTICE PAPERS

Practice Paper-1 π 
⇒ cos 9α = cos  – α
2 
SECTION-A π
⇒ 9α = –α
14587 14587 8 116696 2
1. (D) ... = × = π
1250 1250 8 10000 ⇒ α =
20
= 11.6696.
π
x×y ∴ tan 5α = tan = 1.
2. (C) ... LCM (x, y) = 4
HCF( x , y) 8. (B) Median.
1800 9. (B) Consider,
= = 150.
12 3 sin θ = cos θ
3. (B) Let the zeroes of ax3 + bx2 + cx + d be sin θ 1 1
⇒ = ⇒ tan θ =
α, β and γ. We are given γ = 0. cos θ 3 3
c ⇒ θ = 30°
∴ αβ + βγ + γα = a
c 3
⇒ αβ + 0 + 0 = a ∴ cos θ = cos 30° = .
2
c 10. (B) Given expression
αβ = a .
sin 2 35° + sin 2 (90° – 35°)
a 6 b –3 c1 10 = + sin2 63°
4. (D) 1 = = 3, 1 = = 3, = cos 2 35°+ cos 2 (90° – 35°)
a2 2 b2 –1 c 2 9 + cos 63°sin (90° – 63°)
a1 b c (sin 2 35°+ cos 2 35°)
⇒ i.e., = 1 ≠ c1 = + (sin2 63° + cos2 63°)
b2 b2 2 (cos 2 35°+ sin 2 35°)
⇒ The given lines are parallel. 1
= + 1 (∵ sin2 θ + cos2 θ = 1)
5. (B) ∠D = ∠Q and ∠E = ∠R 1
⇒ ∆DEF ~ ∆QRP (AA rule of similarity) = 2.

DE EF SECTION-B
⇒ ≠ .
PQ RP 11. True, because out of any two consecutive
positive integers, one is even and the
6. (A) ( 3 +1 ) (3 – cot 30° ) other one is odd; and the product of an
even and an odd is even.
= ( 3 + 1) ( 3 – 3 )
12. No, if two zeroes are α and β of polynomial
= 3 ( 3 + 1)( 3 – 1) = 2 3. x2 + kx + k, then
α + β = – k and α . β = k
7. (C) 9α < 90° ⇒ α < 10° ⇒ 2α = – k and α2 = k (when α = β)
⇒ α is an acute angle. –k
cos 9α = sin α ⇒ α= and α2 = k.
2

P R A C T I C E P A P E R S 99
Hence, distance between tops of the poles
k2
⇒ =k (Comparing both) is 13 metres.
4
16. False, because the range of sin θ is [–1, 1]
⇒ k2 = 4k ⇒ k2 – 4k = 0
1
k(– k – 4) = 0 ∴ k = 4, 0. but a + ≥ 2.
a
13. For infinitely many solutions, OR
13 k k See worksheet-50, sol. 6.
= =
39 6 k+4 17. xi fi f i xi
13 k 3 6 18
⇒ = and
39 6 5 8 40
13 k 7 15 105
=
39 k + 4 9 p 9p
⇒ k = 2 and 3k = k + 4 11 8 88
⇒ k = 2 and k = 2, i.e., k = 2. 13 4 52
14. Yes, because converse of Basic propor- Σfi = p + 41 Σfixi = 9p + 303
tionality theorem is applicable here as
∑ fi xi 9p + 303
PA PB 2 Mean = ⇒ 7.5 =
= = . ∑ fi xi p + 41
AQ BR 3
⇒ 7.5p + 307.5 = 9p + 303
⇒ 1.5p = 4.5
∴ p = 3.
18. The maximum class frequency is 20.
⇒ The modal class is 15-20
Class f cf
15. Let the given pole be AB and CD with
0-5 10 10
their feet B and D respectively (see figure).
5-10 15 25
10-15 12 37
15-20 20 57
20-25 9 66
N = 66

N 66
= = 33
2 2
cf just more than 33 is 37.
⇒ Median class is 10-15
PD = AD = 6 m
Now, required sum = 15 + 10 = 25.
∴ CP = 11 m – 6 m = 5 m
And AP = BD = 12 m SECTION-C
Now, in right-angled triangle ACP,
19. On the contrary let us assume that 2 3 –
AC2 = CP2 + AP2
3 2 is a rational number. Then, we can
⇒ AC = 2
5 + 12 2
= 169 = 13 take coprime a and b such that

100 M A T H E M A T I C S – X
a
= 2 3 – 3 2 21. Since x – 5 is a factor of
b
a2 f (x) = x3 – 3 5 x2 + 13x – 3 5 ,
⇒ = 12 + 18 – 12 6
b2 so as f (x) may be rewritten.
(Squaring both the sides)
f(x) = x3 – 3 5 x2 + 13x – 3 5
2
⇒ 12 6 = 30 – a 2 = x3 – 5 x2 – 2 5 x2 + 10x
b
+ 3x – 3 5
30 b 2 – a 2
⇒ 6 =
12 b 2
. ( )
= x2 x – 5 – 2 5 x x – 5 ( )
+ 3 (x – 5 )
Since, a and b are integers, therefore, RHS
of this last equation is rational and so LHS
must be rational. (
= x– 5 ) (x 2
–2 5x + 3)

But this contradicts the fact that 6 is To find zeroes of f(x), put f(x) = 0.
irrational. (x – 5 ) (x 2
– 2 5x + 3 = 0 )
This contradiction has arisen due to
incorrect assumption that 2 3 – 3 2 is ⇒ x – 5 = 0 or x2 – 2 5 + 3 = 0
a rational number. ⇒ x= 5 or
So, we conclude that 2 3 – 3 2 is an
2 5 ± 20 – 4 × 1× 3
irrational number. x=
2
20. Let a be any odd positive integer. Then it
is of the form 6m + 1, 6m + 3 or 6m + 5, 2 5 ±2 2
⇒ x= 5 or x =
where m is an integer. 2
Here, 3 cases arise. ⇒ x= 5 or
Case I: When a = 6m + 1,
x= 5 + 2 or 5 – 2
a2 = (6m + 1)2 = 36m2 + 12m + 1
= 12m (3m + 1) + 1 Hence all the zeroes of f (x) are 5,
= 6q + 1, where q = 2m (3m + 1).
5 + 2 and 5 – 2.
Case II: When a = 6m + 3,
a2 = (6m + 3)2 = 36m2 + 36m + 9 22. The given pair of equations may be re-
= 36m2 + 36m + 6 + 3 written as
= 6(6m2 + 6m + 1) + 3 = 6q + 3, x+y 2 2x – y 10
where q = 6m2 + 6m + 1. = ; = –
2 xy 3 xy 3
Case III: When a = 6m + 5,
a2 = (6m + 5)2 = 36m2 + 60m + 25 1 1 4 1 2 10
i.e, + = ;– + = –
= 36m2 + 60m + 24 + 1 x y 3 x y 3
= 12(3m2 + 5m + 2) + 1 = 6q + 1, Adding this last pair, we get
where q = 2(3m2 + 5m + 2). 3
= –2
Hence, a2 is of the form 6q + 1 or 6q + 3. y
OR 3
⇒ y= –
See worksheet-3, sol. 9. 2

P R A C T I C E P A P E R S 101
3 24. ∠BAD = 90° – ∠CAD
Substituting y = – in the first equation
2 (∵ ∠BAC = 90°)
of last pair, we get = 90° – (90° – ∠ACD)
1 1 4 1 2 4 (∵ ∠ADC = 90°)
+ = ⇒ – =
x –3 3 x 3 3 ⇒ ∠BAD = ∠ACD ...(i)
2 ∠BDA = ∠ADC = 90° ...(ii)
1 1 Using equations (i) and (ii) in ∆ABD and
⇒ =2 ∴ x =
x 2 ∆CAD, we have
1 3 ∆ABD ~ ∆CAD
Hence, x = , y = – is the required
2 2 (AA rule of similarity)
solution. BD AD
⇒ =
23. Let the length of each side of the given AD CD
equilateral triangle be a, then (Corresponding parts)
AB = BC = CA = a ...(i) ⇒ BD . CD = AD2.
a OR
∴ BD = ...(ii)
3 See worksheet-36, sol. 8.

cos θ – sin θ 1– 3
25. =
cos θ + sin θ 1+ 3
cos θ sin θ

Draw AP⊥BC to meet BC at P. P will be cos θ cos θ 1– 3
⇒ =
the mid-point of BC, that is cos θ sin θ 1+ 3
+
a cos θ cos θ
BP = ...(iii)
2 1 – tan θ 1– 3
a a a ⇒ =
∴ DP = BP – BD = – = ...(iv) 1 + tan θ 1+ 3
2 3 6
⇒ tan θ = 3 ∴ θ = 60°.
[Using (ii) and (iii)]
Now, in right-angled triangle APB, 26. In ∆ABC,
AP2 = AB2 – BP2 1 BC
tan A = =
a2 2 2 AB
⇒ AP2 = a2 –
4 ∴ AC = (1)2 + (2 2)2
2
3a
⇒ AP2 = ...(v) = 1+ 8 = 3
4
Also, in right-angled triangle APD, 1 2 2
sin A = , cos A = ,
AD2 = AP2 + DP2 3 3
3a 2
a2 27 a2 + a2 2 2 1
⇒ AD2 = + = sin C = , cos C =
4 36 36 3 3
[From (iv) and (v)] Now, sin A. cos C + cos A. sin C
⇒ 36AD2 = 28a2 ⇒ 9AD2 = 7a2 1 1 2 2 2 2 1 8
= × + × = + = 1.
⇒ 9AD2 = 7AB2. Hence proved. 3 3 3 3 9 9

102 M A T H E M A T I C S – X
27. We will use the step-deviation method. SECTION-D
ui =
Marks No. of Class- d = 29. Let f (x) = 3 2 x2 + 13x + 6 2
(C.I.) students mark i d fiui
( fi ) xi xi – 45 i = 3 2 x2 + 9x + 4x + 6 2
10
(Spliting middle term)
0-10 5 5 – 40 – 4 – 20
10-20 4 15 – 30 – 3 – 12 = 3x( 2 x + 3) + 2 2 ( 2 x + 3)
20-30 8 25 – 20 – 2 – 16 = (3x + 2 2 ) ( 2 x + 3)
30-40 12 35 – 10 – 1 – 12 To find the zeroes of f (x), we have
40-50 16 45 0 0 0 3x + 2 2 = 0 or 2x + 3 = 0
50-60 15 55 10 1 15
60-70 10 65 20 2 20 –2 2 –3
⇒ x= or
70-80 8 75 30 3 24 3 2
80-90 5 85 40 4 20 ∴ Zeroes of the given polynomial are
90-100 2 95 50 5 10 –2 2 –3
Σfi = 85 Σfiui = 29 and .
3 2

Let a = 45. Here h = 10 –2 2 –3


Now, sum of zeroes = +
3 2
 ∑ fi ui 
Mean =a+  ×h – 13
=
 ∑ fi  3 2
29 Coefficient of x
= 45 + × 10 =–
85 Coefficient of x 2
= 45 + 3.41
–2 2 –3
= 48.41. Product of zeroes = ×
3 2
28. The given data is
6 2
=
C.I. 0-5 5-10 10-15 15-20 20-25 25-30 30-35 3 2
f 10 15 30 80 40 20 5 Constant term
= .
From the table, maximum occuring Coefficient of x 2
frequency is 80. So, modal class is 15-20. Hence proved.
30. Let speed of the train be x km/hr and that
 f1 − f 0 
∵ Mode = l +   ×h of the bus be y km/hr.
 2 f1 − f 0 − f 2  Distance
= Time
Here, l = 15, f1 = 80, f0 = 30, f2 = 40, h = 5 Speed
 80 − 30  Case I: According to question, we get
∴ Mode = 15 +   ×5
 160 − 30 − 40  60 300 – 60
⇒ + =4
x y
250
= 15 + = 17.78 60 240
90 ⇒ + =4
x y
Hence, modal size is 17.78 hectares.
OR 15 60 1 4 1
⇒ + =1 ⇒ + = ...(i)
x y x y 15
See worksheet-62, sol. 5.

P R A C T I C E P A P E R S 103
Case II: According to the given conditions, Now,area of a triangle
we get 1
× base × height
=
 100 300 – 100  10 2
 + =4+
 x y  60 1
∴ ar(∆APQ) = × AP × MQ ...(i)
100 2
200 25
⇒ + = 1
x y 6 Also ar(∆APQ) = × AQ × NP ...(ii)
2
4 8 1
⇒ + = Comparing equations (i) and (ii), we get
x y 6
Dividing by 4, we get 1 1
× AP × MQ = × AQ × NP
1 2 1 2 2
+ = ... (ii) AP NP
x y 24 ⇒ = ...(iii)
Use equation (i) – equation (ii), AQ MQ
4 2 1 1 1
– = – Further, ar(∆BPQ) = × PB × MQ ...(iv)
y y 15 24 2
1
2 8–5 3 And ar(∆CQP) = × QC × NP ...(v)
= = 2
y 120 120
But triangles BPQ and CQP are on the
120 ×2 same base PQ and between the same
∴ y= = 80 km/hr
3 parallels PQ and BC, so their areas must
Put y = 80 in equation (i), we get be equal.
1 4 1 i.e., ar(∆BPQ) = ar(∆CQP)
+ =
x 80 15
1 1
1 1 1 4–3 1 ⇒ × PB × MQ = × QC × NP
⇒ = – = = 2 2
x 15 20 60 60 [Using equations (iv) and (v)]
∴ x = 60 km/hr. PB NP
Hence, speed of the train = 60 km/hr ⇒ = ...(vi)
QC MQ
and speed of the bus = 80 km/hr. From, equations (iii) and (vi), we get
OR AP PB
=
See worksheet-22, sol. 9. AQ QC
31. We are given a triangle ABC in which a AP AQ
line PQ parallel to BC is drawn to intersect ⇒ = . Hence proved.
PB QC
the sides AB and AC at P and Q respectively.
We need to prove 32. sin θ + cos θ =3 (Given)
AP AQ ⇒ sin2 θ
+ cos2 θ
+ 2 sin θ cos θ = 3
=
PB QC (Squaring both the sides)
Join PC and QB. Draw QM ⊥ AB and ⇒ 1 + 2 sin θ cos θ = 3
PN ⊥ AC. ⇒ sin θ cos θ = 1 ...(i)
sin θ cos θ
Now, tan θ + cot θ = +
cos θ sin θ
sin 2 θ + cos2 θ 1
= = [Using (i)]
sin θ. cos θ 1
= 1. Hence proved.

104 M A T H E M A T I C S – X
33. Let us take LHS of the given identity.
tan 2 θ cos ec 2 θ
+
tan 2 θ – 1 sec 2 θ – cos ec 2 θ
sin 2 θ 1
cos 2 θ sin 2 θ
= +
sin 2 θ 1 1
–1 –
cos 2 θ cos2 θ sin 2 θ

sin 2 θ 1
cos 2 θ sin 2 θ
= +
sin 2 θ – cos 2 θ sin 2 θ – cos 2 θ
cos 2 θ sin 2 θ.cos 2 θ
sin 2 θ cos2 θ
= +
sin 2 θ – cos 2 θ sin 2 θ – cos2 θ
To obtain median from graph:
sin 2 θ + cos 2 θ Draw a line parallel to x-axis passing
=
sin 2 θ – cos2 θ 50
through y = = 25. This line meets the
1 2
= [... sin2 θ + cos2 θ = 1]
sin θ – cos 2 θ
2 ogive at (68.2, 25).
= RHS. Hence proved. ∴ Median = 68.2.
OR
Practice Paper–2
See worksheet-55, sol. 10.
34. Let us prepare the cumulative frequency SECTION-A
table by more than method as given below:
343 343
Production 1. (C) ... =
3
2 × 5 ×73 3
2 × 53 × 343
3
Production yield
(in kg/ha) cf Point 1
yield fi = = 0.001.
(in kg/ha) more than 1000
or equal to
2. (B) ... p(x) = 3x2 – (2x + 1) x + 3
50-55 2 50 50 (50, 50) = x2 – x + 3
55-60 6 55 48 (55, 48)
–1 3
60-65 8 60 42 (60, 42) ∴ (α + β) – αβ = – –
1 1
65-70 14 65 34 (65, 34)
= 1 – 3 = – 2.
70-75 15 70 20 (70, 20)
75-80 5 75 5 (75, 5) ar(∆ABC) BC 2
3. (A) As =
Total 50 ar(∆DEF) EF 2
(Result on areas of similar triangles)
We plot the points mentioning in the table
D
such that lower class limits are on the A
x-axis and the cumulative frequencies are
on the y-axis. By joining these points by
free hand smooth curve, We obtain more B C E F
than type ogive as shown in the adjoining 54 32
⇒ = 2 ⇒ ar(∆DEF) = 96 cm2.
graph. ar(∆DEF) 4

P R A C T I C E P A P E R S 105
4. (C) As cosec2 θ – cot2 θ = 1 10. (B) As sin (45° + θ) – cos (45° – θ)
⇒ 16 – 3k2 = 1 = sin (45° + θ) – sin {90° – (45° – θ)
15 [∵ sin (90° – α) = cos α]
⇒ k2 =
3 = sin (45° + θ) – sin (45°+ θ) = 0.
⇒ k= 5.
SECTION-B
cos (90o – 70o ) 2 cos θ k
5. (C) + =
sin 70o cos θ 2 11. False, because 3 or 2 3 is an irrational
sin 70 o
k and sum or difference of a rational
⇒ + 2= number and an irrational number is an
sin 70o 2
irrational number.
k
⇒ 3= ⇒ k = 6. 12. No, since the discriminant is zero for
2
1
6. (D) ∵ tan 45° = 1 k=± .
2
1 – tan 2 45° 1–1
∴ 2
= =0 13. The condition for infinitely many solutions
1 + tan 45° 1+1
a1 b c
= tan 0°. is a = 1 = 1 .
2 b2 c2
7. (B) Let us prepare the cumulative fre-
quency table from the given data. 3 – ( a + 1) 2b – 1
⇒ = =
Class Frequency Cumulative 5 1 – 2a 3b
interval frequency
3 – ( a + 1) 3 2b – 1
⇒ = and =
10-15 4 4 5 1 – 2 a 5 3b
15-20 7 11
⇒ 3 – 6a = – 5a – 5 and 9b = 10b – 5
20-25 20 31
⇒ a = 8 and b = 5.
25-30 8 39
14. Yes, because ∆PBC
30-35 1 40
and ∆PDE are
N = 40 similar by SAS rule.
The cumulative frequency just greater BP PC 1
as = = and
N DP PE 2
than = 20 is 31 and its corresponding ∠BPC = ∠DPE.
2
class is 20-25. Hence 20-25 is the median 15. In the given ∆ABC, DE || AB
class.
So by the Thales Theorem,
8. (D) 0.
CD CE
9. (D) The condition for the line parallel is: ⇒ =
AD BE
a1 b c
∵ = 1 =≠ 1 x+3 x
a2 b2 c2 ⇒ =
3x + 19 3x + 4
3 2c 2
⇒ = ≠
2 5 1 ⇒ 3x2 + 19x = 3x2 + 4x + 9x + 12
15 ⇒ 6x = 12
⇒ c= .
4 ⇒ x = 2.

106 M A T H E M A T I C S – X
16. We have the identity: ∴ 3 =2 × 1 + 1
sin2 θ + cos2 θ = 1 Since the new remainder 1 ≠ 0.
∴ 2 =1 × 2 + 0
a2 a
⇒ + cos2 θ = 1 (∵ sin θ = ) Since, the remainder has now become zero,
2
b b the divisor at this stage is 1, the HCF of
847 and 2160 is 1.
a2 b 2 – a2
⇒ cos θ = 1– = . OR
b2 b
See worksheet-1, sol. 9.
17. The empirical relationship among the three
distance
measures of central tendency is: 20. We have time =
Mode = 3 Median – 2 Mean speed
= 3 × 55 – 2 × 50 Time taken by Abhay to cover one
complete round
= 165 – 100 = 65.
OR 360
= = 30 hours
See Assessment sheet-11, sol. 4. 12
Time taken by Ravi to cover one complete
18. Let us convert the given less than type round
distribution to normal distribution.
360
Marks No. of students = = 24 hours
15
0-10 3 Abhay and Ravi reach the starting point
10-20 12 – 3 = 9 respectively after 30 hours and 24 hours,
20-30 27 – 12 =15 and their respective multiples. Therefore,
30-40 57 – 27 =30 they will meet again at the starting point
40-50 75 – 57 =18 after the time given by least common
multiple of 30 hours and 24 hours Let us
50-60 80 – 75 = 5
determine the LCM of 30 hours and 24
From the table, the maximum frequency hours.
is 30 and its corresponding class is 30-40. 30 = 2 × 3 × 5,
Therefore, the modal class is 30-40. 24 = 23 × 3
⇒ LCM = 23 × 3 × 5 = 120
SECTION-C
Hence, the required time is 120 hours.
19. To prove that the pair of numbers
(847, 2160) is coprime by using Euclid’s 21. Let α and β be the zeroes of 6x2 + x + k.
algorithm, we have to prove that the ∴ α2 + β2 =(α + β)2 – 2αβ
highest common factor of the pair is 1. 2
Since 2160 > 847  1 k
= –  – 2 
∴ 2160 = 847 × 2 + 466  6 6
Since the remainder 466 ≠ 0. 1 k
∴ 847 = 466 × 1 + 381 = –
36 3
Since the new remainder 381 ≠ 0.
∴ 466 = 381 × 1 + 85 25
But it is given that α2 + β2 =
Since the new remainder 85 ≠ 0. 36
∴ 381 = 85 × 4 + 41 25 1 k k – 24
Since the new remainder 41 ≠ 0. ∴ = – ⇒ =
36 36 3 3 36
∴ 85 = 41 × 2 + 3
Since the new remainder 3 ≠ 0. ⇒ k = – 2.
∴ 41 = 3 × 13 + 2 OR
Since the new remainder 2 ≠ 0. See worksheet-13, sol. 9.

P R A C T I C E P A P E R S 107
22. To solve a system of equations graphically 24. Draw RM ⊥ PQ.
we need atleast two solutions of each In ∆PRS, ∠PSR > 90°
equation.
Two solutions of the equation 2x – y = 2
are given in the following table:
x 0 3
y –2 4
⇒ PR2 = PS2 + RS2 + 2PS.SM ...(i)
Two solutions of the equation 4x – y = 8
are given in the following table: [Using result on obtuse-angled triangle]
In ∆QRS, ∠QSR < 90°
x 2 1
⇒ QR2 = RS2 + SQ2 – 2SQ.SM ...(ii)
y 0 –4
[Using result on acute-angled triangle]
Let us draw the graph of the two given Add equations (i) and (ii) to get
equations.
PR2 + QR2 = 4PS2 – 2SM(PS – SQ)
2
 PQ 
= 4  (∵ PS = SQ)
 2 
⇒ PR2 + QR2 = PQ2. Hence proved.
OR
See worksheet-41, sol. 7.
3
25. sin θ = ... (i) Given
4
∵ sin2 θ + cos2 θ = 1
∴ cos2 θ = 1 – sin2 θ
2
3
⇒ cos2 θ = 1 –   [Using (i)]
4
7
⇒ cos2 θ =
From the graph, it is clear that the two 16
lines intersect each other at the point (3, 4). 16
Hence, the solution is x = 3, y = 4. ⇒ sec2 θ = ... (ii)
7
23. In right-angled triangle PQS,
Let us take LHS of the given equation.
PS2 = PQ2 + QS2
⇒ QS2 = PS2 – PQ2 cosec 2 θ – cot 2 θ 1
LHS = = 2
2 sec θ – 1
QR 2 sec θ – 1
⇒ = PS2 – PQ2
4 [... cosec2 θ – cot2 θ = 1]
⇒ QR2 = 4PS2 – 4PQ2 ... (i)
In right-angled triangle PQR, 1 7 7
= = = [Using (ii)]
PR2 = PQ2 + QR2 16 9 3
–1
= PQ2 + 4PS2 – 4PQ2 [Using (i)] 7
⇒ PR2 = 4PS2 – 3PQ2. Hence proved. = RHS. Hence proved.

108 M A T H E M A T I C S – X
3 1  f1 – f0 
26. We have sin 60° = , cos 60° = , Now, mode = l +   ×h
2 2 2
 i f – f 0 – f 2 
2 Here,
sec 30° = , cosec 30° = 2
3 l = 40, fi = 20, f0 = 7, f2 = 15, h = 20
3 + sin 2 60° + cosec 2 30°  20 – 7 
∴ ∴ Mode = 40 +   × 20
5 + cos2 60° + sec 2 30°  40 – 7 – 15 
2
 3 2 260
3 +   + ( 2 ) 3
3++4 = 40 + = 40 + 14.44
 2  4 18
= =
2 2 1 4 = 54.44 marks.
1  2  5+ +
5+  +  4 3
 2  3  SECTION-D
Multiply Num. and Deno. by 12, 29. Let f (x) = x4 + x3 – 9x2 – 3x + 18
36 + 9 + 48 93 It is given that – 3 and 3 are two of
= = .
60 + 3 + 16 79 zeroes of f (x).
27. Let us prepare the cumulative frequency ⇒ x + 3 and x – 3 are the factors of f(x)
table from the given data.
⇒ (x + 3 ) (x – 3 ) is a factor of f(x)
C.I. fi xi di = x i – A fi d i
⇒ x2 – 3 is a factor of f (x)
0-10 7 5 – 20 – 140
10-20 10 15 – 10 – 100 ) x4 – 3x2
(
x2 – 3 x4 + x3 – 9x2 – 3x + 18 x2 + x – 6
20-30 15 25 0 0 – +
30-40 8 35 10 80 x3 – 6x2 – 3x + 18
x3 – 3x
40-50 10 45 20 200 – +
Σfi = 50 Σfi di = 40 – 6x2 + 18
– 6x2 + 18
∑ fi di 40 + –
Mean = A + = 25 + 0
∑ fi 50
= 25 + 0.8 = 25.8. ∴ p(x) = (x2 – 3) (x2 + x – 6)
To find other zeroes of p(x)
28. Let us convert the more than type
distribution to the normal distribution. Let x2 + x – 6 = 0
⇒ (x+ 3) (x – 2) = 0
Marks No. of students ⇒ x = – 3 and 2
0-20 3 So all zeroes are ± 3 , 2, – 3.
20-40 7 Hence, the zeroes of f(x) are ± 3 , 2, – 3.
40-60 20 30. Let the speed of rowing in still water
60-80 15 and the speed of the current be u km/hr
80-100 5 and v km/hr respectively.
The speed of rowing in downstream
We observe from the table that the value = (u + v) km/hr.
20 is the maximum frequency. So, the The speed of rowing upstream
modal class is 40-60. = (u – v) km/hr.

P R A C T I C E P A P E R S 109
Using the formula:
Distance
= Time
Speed
According to first condition of the question,
18 12
+ =3 ... (i) Draw AM ⊥ BC and PN ⊥ QR
u+v u – v
In ∆ABM and ∆PQN,
According to second condition of the
∠B = ∠Q (∵ ∆ABC ~ ∆PQR)
question,
∠M = ∠N (Each 90°)
36 40 So, ∆ABM ~ ∆PQN
+ =8 ... (ii)
u+v u–v (AA similarity criterion)
Let us put,
AB AM BC AM
1 1 ∴ = ⇒ = ...(ii) [Using (i)]
= x and = y such that PQ PN QR PN
u+v u–v
equations (i) and (ii) reduce to Since area of a
1
18x + 12y = 3 ... (iii) triangle = × base × height
2
And 36x + 40y =8 ... (iv)
Equations (iii) and (iv) form a pair of 1
ar ( ∆ABC )× BC × AM
linear equations. ∴ = 2
Multiply equation (iii) by 2 and subtract ar ( ∆PQR ) 1
× QR × PN
the result from equation (iv) to get 2
1 BC AM
16y = 2 ⇒ y = =×
8 QR PN
1
Substitute y = in equation (iv) to get Using equation (ii), we get
8
1 ar ( ∆ABC ) BC2
36x = 8 – 5 ⇒ x = = ... (iii)
12 ar ( ∆PQR ) QR 2
1
∵x= ∴ u + v = 12 From equations (i) and (iii), we obtain
u+v
1 ar ( ∆ABC ) AB 2 BC2 CA2
∵y= ∴ u–v=8 = = = .
u–v ar ( ∆PQR ) PQ 2 QR 2 RP2
This last system gives u = 10 and v = 2 Hence proved.
Hence, the speed of the rowing in still
water = 10 km/hr and the speed of the 32. Consider, left hand side of the given equation,
current = 2 km/hr. 1
31. We are given two triangles ABC and PQR tan θ cot θ tan θ tan θ
+ = +
such that ∆ABC ~ ∆PQR. 1 – cot θ 1 – tan θ 1 1 – tan θ
1–
AB BC CA tan θ
∴ = = ...(i)
PQ QR RP tan 2 θ 1
= +
We have to prove tan θ – 1 tan θ (1 – tan θ )

ar ( ∆ABC ) AB2 BC2 CA2 tan 3 – 1


= = = =
ar ( ∆PQR ) PQ 2 QR 2 RP2 tan θ ( tan θ – 1)

110 M A T H E M A T I C S – X
Adding equations (v) and (vi), we get
( tan θ – 1) ( tan 2 θ + 1 + tan θ )
= (m2n)2/3 + (n2m)2/3 = cos2 θ + sin2 θ
tan θ ( tan θ – 1)
i.e., (m2n)2/3 + (n2m)2/3 = 1.
[∵ a3 – b3 = (a – b) (a2 + b2 + ab] Hence proved.
1 sin θ cos θ 34. Let us prepare the cumulative frequency
= tan θ + +1 = + +1
tan θ cos θ sin θ distribution from the given distribution.

sin 2 θ + cos2 θ Class Frequency Cumulative


= + 1 = sec θ. cosec θ + 1 interval frequency
sin θ cos θ
(C.I.) ( f) (c f )
= RHS. Hence proved.
0-10 5 5
OR
10-20 x 5+x
See Assessment sheet-9, sol. 8.
20-30 20 25 + x
33. m = cosec θ – sin θ (Given) 30-40 14 39 + x
1 1 – sin θ 2 40-50 y 39 + x + y
⇒ m= – sin θ = 50-60 8 47 + x + y
sin θ sin θ
N = 47 + x + y
cos2 θ
⇒ m= ... (i) Since 27 is the median, therefore, 20-30
sin θ
is the median class.
(∵ 1 – sin2 θ = cos2 θ)
∴ l = 20, cf = 5 + x, f = 20, h = 10
cos 4 θ Given: N = 68
⇒ m2 = ... (ii) (Squaring)
sin 2 θ Using the formula:
Further, n = sec θ – cos θ (Given)
N 
1 – cf
⇒ n= – cos θ  2 
cos θ
Median = l +   ×h
 f 
 
1 – cos2 θ
⇒ n=
cos θ  68 
–5– x
 2 
sin 2 θ ⇒ 27 = 20 + 
20  ×10
⇒ n= ... (iii)  
cos θ  
sin 4 θ 29 – x
⇒ n2 = ... (iv) ⇒ 27 = 20 +
cos2 θ 2
Multiplying equations (ii) and (iii), we ⇒ 29 – x = (27 – 20) × 2 ⇒ x = 15
get
From the table,
cos 4 θ sin 2 θ N = 47 + x + y But N = 68 (Given)
m2n = × = cos3 θ
sin 2 θ cos θ ∴ x + y + 47 = 68
∴ (m2n)2/3 = cos2 θ ... (v) ⇒ 15 + y + 47 = 68 (Substituting x = 15)
Multiplying equations (i) and (iv), we get ⇒ y=6
sin 4 θcos2 θ Thus, x = 15, y = 6.
n2m = × = sin3 θ
cos2 θ sin θ OR
∴ (n2m)2/3 = sin2 θ ... (vi) See worksheet-65, sol. 7.

P R A C T I C E P A P E R S 111
Practice Paper –3 B + C  180° – A 
Now, tan   = tan  
 2   2 
SECTION-A
1. (D) There are infinitely many real  A A
= tan  90° –  = cot .
numbers of both types rational and  2  2
irrational between 3 and 5 . 10. (B) Given expression
2. (D) Decimal representation of an irrational = cos (40° + θ) – cos {90°– (50°– θ)}
number is always non-terminating, non-
repeating. cos2 40° + cos2 (90° – 40°)
+
6 sin 2 40° + sin 2 (90° – 40°)
3. (B) Product of zeroes = =4
a
= cos (40°+ θ) – cos (40°+ θ)
6 3
⇒ a= = . cos2 40° + sin 2 40°
4 2 1
+ 2 2 =0+ = 1.
4. (B) For infinite number of solutions: sin 40° + cos 40° 1
3 5 SECTION-B
= , i.e., k = 6.
k 10
11. 8n can be rewritten as 23n. Clearly, the
5. (A) ∆ABC ~ ∆DEF
prime factor of 8n is only 2. To end with
the digit 0, one of the prime factors of 8n
must be 5. Hence, 8n cannot end with the
digit zero for any n∈N.
12. True, because we find the remainder zero
when 3x4 + 5x3 – 7x2 + 2x + 2 is divided
by x2 + 3x + 1.
∠B = ∠E =180° – (40° + 65°) = 75°.
13. Infinite number of solutions because the
6. (C) AC2 = BC2 – AB2 system obeys the following condition:
⇒ AC2 = 2 – 1 = 1 a1 b c 1 –3 –3
AC = 1 = 1 = 1 , i.e., = = .
a2 b2 c2 3 – 9 –9
BC 2
cosec C = = 14. Yes, because converse of Basic Propor-
AB 1 tionality Theorem holds as
= 2.
sin θ sin θ 1 – cos θ PM PN 3
7. (C) = × = = .
1 + cos θ 1 + cos θ 1 – cos θ MQ NR 2

sin θ (1 – cos θ ) 1 – cos θ


= 2
= . 15. In ∆AOD and ∆COB,
sin θ sin θ
AO DO 1
8. (D) Using empirical formula, = =
OC OB 2
2 Mean = 3 Median – Mode
and∠AOD = ∠COB
⇒ 2 Mean = 3 × 500 – 400 ⇒ ∆AOD ~ ∆COB
1100
⇒ Mean = = 550. AD 1 4 1
2 ∴ = ⇒ =
BC 2 BC 2
9. (A) A + B + C = 180°
(Angle sum property of a triangle) ⇒ BC = 8 cm.

112 M A T H E M A T I C S – X
16. No. ⇒ a2 is divisible by 2 ...(ii)
1 ⇒ a is divisible by 2 ...(iii)
For θ = 30°, tan θ = and cot θ = 3
3 [If a prime (here 2) divides d2, then the
1 10 same prime divides d, where d is a positive
tan2 θ + cot2 θ = + 3 = ≠ 2.
3 3 integer.]
17. Since the maximum frequency is 8, so ⇒ a = 2c
modal class is 4-8. ⇒ a2 = 4c2 ... (iv)
∴ l = 4, f1 = 8, f0 = 4, f2 = 5, h = 4 From (i) and (iv), we get
4c2 = 2b2 ⇒ b2 = 2c2
 f1 – f0  ⇒ b2 is divisible by 2
Now, mode = l +  ×h
 2 f1 – f0 – f2  ⇒ b is divisible by 2 .... (v)

 8–4  From results (ii) and (v), we have a and b


=4 +  ×4 both are divisible by 2.
 16 – 4 – 5  But this contradict the fact that a and b are
= 4 + 2.29 = 6.29. coprime. This contradiction has arisen
OR because of our incorrect assumption that
See worksheet-65, sol. 4. 2 is a rational number. Thus, we conclude
18. Let us prepare cumulative frequency table: that 2 is an irrational number.
C.I. f cf OR
60-70 2 2 See worksheet-4, sol. 9.
70-80 5 7 20. Let a be a positive integer. Then it is of
80-90 12 19 the form 3q, 3q + 1 or 3q + 2.
Now, three cases arise.
90-100 31 50
Case I: a = 3q
100-110 39 89
∴ a3 = (3q)3 = 27q3 = 9 (3q3) = 9m,
110-120 10 99
120-130 4 103 where m = 3q3.
Case II: a = 3q + 1
N = 103
∴ a3 = (3q + 1)3
N 103 = 27q3 + 1 + 27q2 + 9q
= = 51.5
2 2
= 9 (3q3 + 3q2 + q) + 1
Cumulative frequency just greater than
51.5 is 89. So, median class is 100-110. = 9m + 1,
where m = 3q3 + 3q2 + q.
SECTION-B
Case III: a = 3q + 2
19. Let us assume on the contrary that 2 is ∴ a3 = (3q + 2)3
a rational number. Then 2 can be written
a = 27q3 + 8 + 54q2 + 36q
as 2 = , where a and b are coprime = 9 (3q3 + 6q2 + 4) + 8
b
and b ≠ 0. = 9m + 8,
2
a where m = 3q3 + 6q2 + 4.
2= (Squaring)
b2 Hence, a3 is of the form 9m, 9m + 1 or
⇒ a2 = 2b2 ... (i) 9m + 8.

P R A C T I C E P A P E R S 113
21. p(t) = t2 – 15 3 3 2
To obtain zeroes of p(t), put p(t) = 0 Adiagonal = (a 2 )2 = a
4 2
i.e., t2 – 15 = 0
3  4 
⇒ Adiagonal = ×  Aside  [Using (i)]
( ) =0
2
t2 – 15 2  3 
⇒ (t + 15 )(t – 15 ) = 0
⇒ Aside =
1
× Adiagonal . Hence proved.
⇒ t = – 15 , 15 2
24. In the figure drawn,
So, zeroes of p(t) are – 15 and 15 AB || DC and ∆AED
Sum of zeroes = – 15 + 15 = 0 ~ ∆BEC.
∆ADC and ∆BDC
–0 – Coefficient of t both are on the same base DC and lie
= =
1 Coefficient of t2 between same parallels AB and DC.
Product of zeroes So,
–15 ar(∆ADC) = ar(∆BDC)
= – 15 × 15 = – 15 =
1 ⇒ ar(∆AED) + ar(∆DEC)
Constant term = ar(∆BEC) + ar(∆DEC)
= .
Coefficient of t ⇒ ar(∆AED) = ar(∆BEC) (i)
Hence verified. Now,
22. The given system of equations can be re- ar(∆AED) (AD)2
written as =
ar(∆BEC) (BC)2
4x + 3y – 48 = 0
40x – 6y – 192 = 0 (∵ ∆AED ~ ∆BEC)
Applying the method of cross multipli-
( AD )2
cation to solve the system. ⇒ 1= [From (i)]
x –y 1 (BC )2
⇒ = = ⇒ AD = BC. Hence proved.
– 576 – 288 – 768 + 1920 – 24 – 120
x y 1 25. Given expression
⇒ = =
– 864 – 1152 – 144
{( }
2
 1 4  1  4   2 
)
2 2
= 2   +    – 3 + (1) + 3 
864 1152
⇒ x= and y =  2   2    3
144 144
 1 1  4
⇒ x = 6 and y = 8. = 2  +  – ( 3 + 1) + 3 ×
 16 16  3
23. We are given a square of side length a.
1 1
Then length of its diagonal will be a 2 . = –4+4= .
4 4
We know that area of an equilateral
26. Consider left hand side of the given
3 2 equation.
triangle of side length x is x .
4
∴ Area of the equilateral triangle described LHS = (cosec A – sin A) (sec A – cos A)
of the side of the square.  1   1 
= – sin A   – cos A 
3 2 4  sin A   cos A 
Aside = a ... (i) ⇒ a2 = A
4 3 side
1 – sin 2 A 1 – cos2 A
And area of the equilateral triangle = .
described on the diagonal of the square sin A cos A

114 M A T H E M A T I C S – X
cos2 A sin 2 A N
= . = sin A cos A Here, N = 80 ∴ = 40
sin A cos A 2
Also, taking right hand side, Cumulative frequency just more than 40
is 58. So 24-32 is the median class.
1 1
RHS = = ∴ l = 24, cf = 34, f = 24, h = 8
tan A + cot A sin A cos A
+
cos A sin A  N – cf 
 
=
sin A cos A
= sin A cos A ∴ Median = l +  2 ×h
sin 2 A + cos2 A f 
 
 
Hence, LHS = RHS.  40 – 34  48
= 24 +   × 8 = 24 + = 26.
OR  24  24
See worksheet-51, solution-9.
OR
27. Let us use step-deviation method to
See Assessment sheet-12, sol. 6.
obtain the mean.
di SECTION-D
C.I. fi xi di = xi – a u i = di u i
h 29. Let f (x) = 3x4 + 6x3 – 2x2 – 10x – 5
0-20 17 10 – 40 –2 – 34
20-40 p 30 – 20 –1 –p 5 5
x= and x = – are the zeroes of f (x)
40-60 32 50 0 0 0 3 3
60-80 24 70 20 1 24
80-100 19 90 40 2 38  5  5
⇒ x–  and  x +  are factors of f (x)
 3  3
Σfi = 92+p Σfi ui = 28 – p

Here a = 50, h = 20  5 5 5


⇒ x–  x+  = x2 – is a factor of f (x)
 ∑ fi ui   3  3  3
Mean = a +   ×h
 ∑ fi  5
Let us divide f (x) by x2 –
28 – p 3
⇒ 50 = 50 +
92 + p
⇒ 28 – p = 0 ∴ p = 28.
28. We prepare cumulative frequency table
from the given data.
C.I. Frequency Cumulative
( f) Frequency
(cf )
0-8 8 8
8-16 10 18
16-24 16 34
24-32 24 58
32-40 15 73 To obtain other two zeroes,
40-48 7 80 put the quotient = 0
N = 80 i.e., 3x2 + 6x + 3 = 0

P R A C T I C E P A P E R S 115
⇒ 3 (x2 + 2x + 1) = 0 So, ∆ABC ~ ∆DBA
⇒ 3 (x + 1)2 = 0 (A A criterion of similarity)
⇒ x = – 1 or – 1 AB BC
Therefore, =
BD AB
Hence, all the zeroes of f(x) are ± 5 , or AB2 = BD . BC ...(i)
3
– 1, –1. Similarly, ∆ABC ~ ∆DAC
x AC BC
30. Let the original fraction be . Therefore, =
y DC AC
On adding 1 to both the numerator and
AB2 = DC . BC ...(ii)
x 4 Adding equations (i) and (ii), we get
the denominator of , it becomes
y 5
BD . BC + DC . BC = AB2 + AC2
x +1 4
i.e., = , i.e., 5x + 5 = 4y + 4 or (BD + DC) . BC = AB2 + AC2
y +1 5
or BC . BC = AB2 + AC2
i.e., 5x – 4y = –1 ...(i)
(∵ BC = BD + DC)
On subtracting 5 from both the numerator
or BC2 = AB2 + AC2.
x 1
and the denominator of , it becomes Hence proved.
y 2
OR
x–5 1
i.e., = , i.e., 2x – 10 = y – 5 See worksheet-38, sol. 9.
y–5 2
32. We have
i.e., 8x – 4y = 20 ...(ii)
x3 = sec A – cos A
Subtracting equation (i) from equation
(ii), we get
1 1 – cos2 A
= – cos A =
cos A cos A
3x = 21 ⇒ x = 7
Substituting x = 7 in equation (i), we get 2
1

x =  sin A 
3
5 × 7 – 4y = –1 ⇒ y = 9 ∴
 cos A 
7
Hence, the required fraction is .
9
OR Similarly, y3 = cosec A – sin A
See worksheet-27, sol. 9 (OR). 1 1 – sin 2 A
= – sin A =
31. Pythagoras Theorem: sin A sin A
In a right triangle, the square of the
1
hypotenuse is equal to the sum of the 2
y =  cos A 
3
squares of the other two sides.
 sin A 
Proof: We are given, a ∆ABC in which  
∠A = 90°
We need to prove LHS = x2 y2 (x2 + y2)
BC2 = AB2 + AC2. = x4y2 + x2y4
Draw AD ⊥ BC
1 4 1 2
(see figure)  
  sin 2 A  3  2
 cos A  3 
In ∆ABC and ∆DBA, = 
∠ABC = ∠DBA (Common)  cos A    sin A  
     
∠BAC = ∠BDA (Each 90°)    

116 M A T H E M A T I C S – X
1 2 1 4 1 1 1
    or x+ –x+ = 2x or .
  sin 2 A  3  2
 cos A  3  4x 4x 2x
+ 
 cos A    sin A   Hence proved.
     
   
34. We prepare the cumulative frequency
4 2 table by less than method as given below:

=  sin 2 A  3  cos2 A  3
 cos A   sin A  Marks Frequ- Marks cf Point
   
ency less
2 4 than
 sin 2 A  3  cos2 A  3
+ 
 cos A   sin A  0-10 4 10 04 (10, 4)
   
10-20 10 20 14 (20, 14)
1
20-30 16 30 30 (30, 30)
 sin 8 A cos4 A  3
=  4
× 2  30-40 22 40 52 (40, 52)
 cos A sin A 
40-50 20 50 72 (50, 72)
1 50-60 18 60 90 (60, 90)
 sin 4 A cos8 A  3
+  ×  60-70 8 70 98 (70, 98)
2 4
 cos A sin A  70-80 2 80 100 (80, 100)
1 1
We take marks on the x-axis and
= (sin6 A )3 + (cos6 A )3 cumulative frequency on the y-axis and
= sin2A + cos2 A = 1 = RHS. then plot the points mentioned in the
table. On joining these points by free
Hence proved.
hand smooth curve, we get less than
1 ogive.
33. sec θ = x + ... (i) (Given)
4x Further, we prepare the cumulative
2
frequency table by more than method as
1 
or sec2 θ =  x +  (Squaring) given below:
 4x 
Marks Frequ- Marks cf Point
1 1 ency more
or 1 + tan2 θ = x2 + + than
16 x 2 2
or
1 1 equal
or tan2 θ = x2 + 2
– to
16x 2
2
0-10 4 0 100 (0, 100)
 1 
or tan2 θ = x –  10-20 10 10 96 (10, 96)
 4 x
20-30 16 20 86 (20, 86)
1  30-40 22 30 70 (30, 70)
or tan θ = ±  x –  ... (ii)
 4x  40-50 20 40 48 (40, 48)
Add equations (i) and (ii) to get 50-60 18 50 28 (50, 28)
60-70 8 60 10 (60, 10)
1 1
sec θ + tan θ = x + +x– 70-80 2 70 2 (70, 2)
4x 4x

P R A C T I C E P A P E R S 117
We will plot the points mentioned in this Practice Paper–4
table on the same graph. On joining these
points by free hand smooth curve, we SECTION-A
get more than ogive.
2− 3 2− 3 2− 3
1. (D) = ×
2+ 3 2+ 3 2− 3

Y 2+3−2 6
=
105
2−3
100 (0, 100) (80, 100) = 2 6 −5
(70, 98)
(10, 96)
95 As 2 6 − 5 is irrational, it has non-termi-
90 (60, 90) nating, non-repeating decimal form.
(20, 86)
85
less than ogive
2. (D) As p2 and p3 are common factors.
80 n l
∴ HCF (x, y) is p2 p3 .
75
(50, 72) 3. (B) p(x) = (x – 1)2 + 2x + 1 = x2 + 2
70 (30, 70)
0
65 Sum of zeroes = = 0;
1
60
2
55 product of zeroes = = 2.
(40, 52) 1
50 (40, 48)
4. (C) 2 + k = 1 and p + 3 = 2, i.e., k = – 1 and
45
p = – 1. So, p + k = – 2.
40
AD AE 1
35 5. (C) = =
BD CE 3
30 (30, 30) (50, 28)
25
⇒ DE || BC

20 more than ogive


15
(20, 14)
10 (60, 10)

5 (10, 4)
(39, 0) (70, 2)
0 X
10 20 30 40 50 60 70 80 90 100

Median: The abscissa of the point of


intersection of both the ogives
⇒ ∆ADE ∼ ∆ABC
determines the median of the given
distribution. To find such abscissa, we DE AD
∴ =
draw a perpendicular from the point BC AB
of intersection of both the ogives to x 3.5
⇒ =
the x-axis, which meet the axis at (39, 0). 7.5 14
Hence the required median is 39
marks. 15
⇒ x= cm.
8

118 M A T H E M A T I C S – X
6. (D) If x = 30°, 2k + 3 2k + 1 4(k − 1)
3 cos 30° – 4 cos3 30° i.e., = = ,
4 3 3
3 3 3 5
=3× –4× = 0. i.e., k= .
2 8 2
7. (A) The value of cosec θ is minimum at an 14. Yes.
angle where sin θ is maximum as sin θ is
In ∆ ABC,
maximum if θ = 90°.
AD AE 1
∴ sin 90° = 1 = =
DB EC 3
So minimum value of cosec θ = 1.
So, by converse of
8. (B) 52 is the median as the abscissa of the
point of intersection of the two ogives Thales Theorem, DE || BC.
determines the median of the data. 15. No.
9. (A) We know that sin (90° – θ) = cos θ, In ∆AOB and ∆DOC,
cos (90° – θ) = sin θ, tan (90° – θ) = cot θ ∠AOB = ∠DOC
∴ Given expression (Vertically opposite
sin θ cos θ tan θ cot θ angles)
= +
sin θ cos θ tan θ cot θ
AO OB
= 1 + 1 = 2. ≠
DO OC
10. (B) tan 2θ = cot (θ + 12°)
5 6 5 3
⇒ tan 2θ = tan {90° – (θ + 12°)} as ≠ , i.e., ≠
3 10 3 5
[∵ tan (90° – α) = cot α]
Therefore, ∆AOB is not similar to ∆DOC.
⇒ tan 2θ = tan (78° – θ)
16. Yes.
⇒ 2θ = 78° – θ
Let us take left hand side of the given
⇒ θ = 26°.
equation.
SECTION-B 1 + sin 2 θ 1 sin 2 θ
LHS = = +
11. The required highest number will be the cos 2 θ cos 2 θ cos 2 θ
HCF of 120, 224 and 256.
= sec2 θ + tan2 θ
120 = 23 × 3 × 5; 224 = 25 × 7; 256 = 28
= (1 + tan θ)2 + tan2 θ = 1 + 2 tan2 θ
Therefore, HCF = 23 = 8.
= RHS.
12. Let the required polynomial be f(x).
OR
Then f (x) = k[x2 – (sum of zeroes) x +
See worksheet-54, sol. 5.
product of zeroes]
17. Let us convert the given more than type
= k[x2 – (–5 + 2)x + (– 5)(2)]
distribution to the normal distribution.
= k(x2 + 3x – 10),
k being a real number. Marks No. of students
f(x) is not a unique polynomial as k is 0-20 3
any real number. 20-40 7
13. Condition for infinite number of solutions: 40-60 20
a1 b c 60-80 15
= 1 = 1
a2 b2 c2 80-100 5

P R A C T I C E P A P E R S 119
The maximum valuable frequency is 20. Case I: When n = 3q, n + 1 = 3q + 1 and
So, modal class is 40-60. n + 2 = 3q + 2.
∴ l = 40, f1 = 20, f0 = 7, f2 = 15, h = 20 Here, only n is divisible by 3.
Case II: When n = 3q + 1, n + 1 = 3q + 2
 f1 − f0 
Now, mode = l +  ×h and n + 2 = 3q + 3 = 3(q + 1)
 2 f1 − f0 − f2 
Here, only n + 2 is divisible by 3.
 20 − 7  Case III: When n = 3q + 2,
= 40 +  × 20.
 40 − 7 − 15  n + 1 = 3q + 3 = 3(q + 1)
= 40 + 14.44 and n + 2 = 3q + 5 = 3(q + 1) + 2
= 54.44 marks. Hence only n + 1 is divisible by 3.
18. As N = 50 OR
See worksheet-4, sol. 11.
N
∴ = 25 21 By the division algorithm,
2
Dividend = Divisor × Quotient +
∴ Draw a line parallel to x-axis from y = 25
Remainder
which meet to ogive at points P(40, 25).
3 2
∴ 6x + 8x – 3x + 8 = g(x) × (3x + 4) +
∴ Abscissa of P is 40. 6x + 20
⇒ Median = 40.
6 x 3 + 8 x 2 − 9x − 12
⇒ g(x) =
SECTION-C 3x + 4

19. Let us assume to the contrary that 3 − 4 5 2x 2 (3x + 4) − 3(3x + 4)


=
is rational. Then we can take integers a 3x + 4
and b ≠ 0 such that
a (3 x + 4)(2 x 2 − 3)
⇒ g(x) =
= 3– 4 5, 3x + 4
b
a ⇒ g(x) = 2x2 – 3.
i.e., 4 5 = 3–
b 22. To draw a line, we need atleast two
3b − a solutions of its corresponding equation.
i.e., 5=
4b x 0 2 x 1 0
RHS of this last equation is rational as a y y
–4 0 0 –1
and b are integers, but LHS of it is
irrational. This is an incorrect statement Two solutions of Two solutions of
due to our wrong assumption that 3 − 4 5 2x – y = 4 x–y=1
is rational.
From the graph, the two lines intersect
So, we conclude that 3 − 4 5 is each other at the point A(3, 2).
irrational. ∴ x = 3 and y = 2
20. Any positive integer is either of the form Shaded region is ∆ABC.
3q, 3q + 1 or 3q + 2. ∴ Height of ∆ABC = 3 units
There are 3 cases now: And its base = BC = 3 units.

120 M A T H E M A T I C S – X
24. Let median AD passes
through the point O on PQ.
To prove: PO = QO
Proof: In ∆APO and ∆ABD,
∠PAO = ∠BAD (Common angle)
∠APO = ∠ABD  Corresponding

and ∠AOD = ∠ADB  angles
∴ ∆APO ~ ∠ABD (By AAA similarity)
PO AO
⇒ = ...(i)
BD AD
Similarly, In ∆AQO and ∆ACD,
QO AO
⇒ = ...(ii)
CD AD
From equations (i) and (ii), we have
1
∴ ar(∆ABC) = ×3×3
2 PO QO
=
9 BD CD
= square units.
2 ⇒ PO = QO (... BD = CD)
Hence, median AD also bisects PQ.
OR
Proved.
See worksheet-29, sol. 9.
OR
23. Let us draw MN parallel to AB, which
See worksheet-44, sol. 5.
passes through P. So, AM = BN and
DM = CN. cos θ − sin θ 1 − 3
25. =
cos θ + sin θ 1 + 3
Using componendo and dividendo, we get
cos θ − sin θ + cos θ + sin θ
cos θ − sin θ − cos θ − sin θ
From right-angled triangles APM, BPN, 1− 3 + 1+ 3
CPN, DPM; we have respectively =
1− 3 −1− 3
PA2 = PM2 + AM2 .... (i)
2 cos θ 2
PB2 = PN2 + BN2 .... (ii) ⇒ =
− 2 sin θ −2 3
PC2 = PN2 + CN2 .... (iii)
1
PD2 = PM2 + DM2 .... (iv) ⇒ cot θ =
3
From equations (i) and (ii),
⇒ cot θ = cot 60°
PA2 – PB2 = PM2 – PN2 .... (v)
⇒ θ = 60°.
From equations (iii) and (iv),
PC2 – PD2 = PN2 – PM2 .... (vi) 2 1 2 1
26. LHS = 2
− 4
− 2
+
Add equations (v) and (vi) to get cos θ cos θ sin θ sin 4 θ
PA2 + PC2 = PB2 + PD2. = 2 sec2 θ – sec4 θ – 2 cosec2 θ
Hence proved. + cosec4 θ

P R A C T I C E P A P E R S 121
= 2(1 + tan2 θ) – (1 + tan2 θ)2
N
– 2(1 + cot2 θ) + (1 + cot2 θ)2 ∵ N = 500, ∴ = 250.
2
= (1 + tan2 θ) × (2 – 1 – tan2 θ) So, cf = 200, f = 115, l = 3000.
– (1 + cot2 θ) × (2 – 1 – cot2 θ)
 N − cf 
= 1 – tan4 θ – (1 – cot4 θ)  2 
= cot4 θ – tan4 θ = RHS. Median = l+   ×h
 f 
Hence proved.
27. We convert the given data of less than  250 − 200 
= 3000 +  × 500
type to the normal distribution.  115 

Marks fi xi di = f i di = 3000 + 217.39


xi – 25 = 3217.39 hours.

0-10 5 5 – 20 – 100 SECTION-D


10-20 11 15 – 10 – 110
29. Let f (x) = 6x4 + 8x3 – 5x2 + ax + b
20-30 19 25 0 0
f (x) is divisible by 2x2 – 5.
30-40 30 35 10 300
⇒ We obtain the remainder as zero when
40-50 15 45 20 300
f (x) is divided by 2x2 – 5.
Σfi = 80 Σfidi = 390 Now we divide f (x) by 2x2 – 5.
Let us use the assumed mean method with 3x 2 + 4 x + 5
assumed mean a = 25.
2x 2 − 5 6 x 4 + 8 x 3 − 5x 2 + ax + b
Σfi di 390 6 x 4 − 15 x2
Now, mean = a + = 25 + − +
Σfi 80 _____________________________

= 25 + 4.875 = 29.88 marks. 8 x 3 + 10 x 2 + ax + b


28. We prepare the cumulative frequency table 8x 3 − 20 x
− +
for the given data. _________________________________
2
Lifetimes Frequency Cumulative 10 x + ( a + 20) x + b
(in hrs.) (f) frequency 10 x 2 − 25
(cf ) − +
_______________________________

1500-2000 24 24 ( a + 20) x + b + 25
_____________________
2000-2500 86 110
As remainder = 0,
2500-3000 90 200
(a + 20)x + b + 25 = 0
3000-3500 115 315
i.e., (a + 20)x + (b + 25) = 0x + 0
3500-4000 95 410
i.e., a + 20 = 0 and b + 25 = 0
4000-4500 72 482
i.e., a = – 20 and b = – 25.
4500-5000 18 500
30. Let the present ages of father and his son
N = 500
are x years and y years respectively.
According to the given conditions:
Here, h = 500
x + y = 65

122 M A T H E M A T I C S – X
After 5 years, ⇒ AP × QM = AQ × PN
the father’s age = (x + 5) years QM AQ
⇒ = ... (i)
After 5 years, PN AP
the son’s age = (y + 5) years Since ∆BPQ and ∆CQP are on the same
Therefore, x + 5 = 2 (y + 5) base PQ and between the same parallels
i.e., x – 2y = 5 PQ and BC, therefore, their areas should
Thus, the required pair of linear equations is be equal.
x + y = 65 .... (i) i.e., ar(∆BPQ) = ar(∆CQP)
x – 2y = 5 .... (ii) 1 1
⇒ × PB × QM = × QC × PN
Subtracting equation (ii) from equation (i), 2 2
we get
QM QC
3y = 60 ⇒ = ... (ii)
PN PB
⇒ y = 20
Substituting y = 20 in equation (i), we get Form equations (i) and (ii), it is clear that
x + 20 = 65 AQ QC
= ,
⇒ x = 45 AP PB
Thus, present age of father = 45 years and AP AQ
i.e., = .
present age of his son = 20 years. PB QC
31. We are given a ∆ABC in which a line Hence proved.
PQ || BC intersects the sides AB at P and cot x tan x
AC at Q. 32. + = 1 + sec x . cosec x
1 – tan x 1 – cot x
AP AQ
We need to prove = . LHS
PB QC
cos x sin x
Draw QM ⊥ AB and PN ⊥ AC. cot x tan x sin x cos x
= + = +
1 – tan x 1 – cot x sin x cos x
1– 1–
cos x sin x

cos x cos x sin x sin x


= × + ×
sin x cos x – sin x cos x sin x – cos x

cos 2 x sin 2 x
= +
sin x (cos x – sin x) cos x (sin x – cos x)

Join PC and BQ. cos3 x – sin 3 x


=
Proof: Area of a triangle sin x . cos x (cos x – sin x)
1 (cos x – sin x)(cos2 x + cos x . sin x + sin 2 x)
= × Base × Height =
2 sin x . cos x (cos x – sin x)
1
∴ ar(∆APQ) = × AP × QM 1 + cos x . sin x 1
2 = = +1
sin x . cos x sin x . cos x
1 = 1 + sec x . cosec x = RHS.
= × AQ × PN
2 Hence proved.

P R A C T I C E P A P E R S 123
33. It is given that 34. We prepare a table for less than series with
sin θ + cos θ = a corresponding cumulative frequencies and
Squaring both the sides, we get points.
sin2 θ + cos2 θ + 2 sin θ cos θ = a2 Marks Fre- Marks Cumu- Point
But sin2 θ + cos2 θ = 1 .... (i) quency less lative
∴ 1 + 2 sin θ cos θ = a2 than frequency
⇒ 2 sin θ cos θ = a2 – 1
0-10 5 10 5 (10, 5)
a2 − 1 10-20 8 20 13 (20, 13)
⇒ sin θ cos θ = .... (ii)
2
20-30 6 30 19 (30, 19)
Cubing both the sides of equation (i), we
30-40 10 40 29 (40, 29)
get
sin6 θ + cos6 θ 40-50 6 50 35 (50, 35)
+ 3 sin2 θ cos2 θ (sin2 θ + cos2 θ) = 1 50-60 5 60 40 (60, 40)
⇒ sin θ + cos6 θ + 3 (sin θ cos θ)2 = 1
6

[Using (i)]
2
 a 2 − 1
⇒ sin6 θ+ cos6 θ+3  =1
 2 
( a2 − 1)2
⇒ sin6 θ + cos6 θ = 1 – 3 ×
4
4 − 3( a 2 − 1)2
⇒ sin6 θ + cos6 θ = .
4
Hence proved.
OR
Consider
a sin3 α + b cos3 α = sin α . cos α
⇒ a sin α . sin2 α + b cos α . cos2 α
= sin α . cos α
⇒ b cos α . sin α + b cos α . cos2 α
2

= sin α . cos α
. .
( . a sin α = b cos α)
⇒ b sin2 α + b cos2 α = sin α
⇒ b (sin2 α + cos2 α) = sin α
∴ b = sin α ...(i)
We take upper limits on the x-axis and
Again, a sin α = b cos α cumulative frequencies on the y-axis. Then
⇒ a . b = b cos α [From (i)] we plot the points on the graph paper. By
∴ a = cos α ...(ii) joining these points by free hand smooth
Now, squaring and adding equations (i) curve, we obtain less than ogive as shown
and (ii), we get in the above figure.
b2 + a2 = sin2 α + cos2 α OR
∴ a2 + b2 = 1. Hence proved. See worksheet-62, sol. 7.

124 M A T H E M A T I C S – X
Practice Paper–5 DE + EF + FD EF
6. (B) =
AB + BC + CA BC
SECTION-A
1. (D) 2 and 5.
2. (B) Let us assume that x + y is rational
number and let x + y = z; when z is
rational.
⇒ x + y + 2 xy = z2
4
⇒ 2 xy = z2 – x – y ⇒ Perimeter of ∆DEF = × (3 + 2 + 2.5)
2
z2 – x – y ⇒ Perimeter of ∆DEF = 15 cm.
xy =
2
7. (C) cos A + cos2 A = 1
which given a contradiction as LHS is
⇒ cos A = 1 – cos2 A
irrational but RHS is rational.
⇒ cos A = sin2 A
⇒ x+ y is an irrational number. ⇒ cos2 A = sin4 A (Squaring)
3. (C) Coincident lines is given by ⇒ 1 – sin2 A = sin4 A
b sin4 A + sin2 A = 1.
a1 c
= 1 = 1
a2 b2 c2 8. (C) Relation among mean, median and
mode is given by the following impirical
Here, a1 = p, b1 = q, c1 = – 4 formula:
a2 = 4, b2 = 3, c2 = – 5 Mode = 3 Median – 2 Mean.
p q –4 sin θ p
Now, = = 9. (C) tan θ = = .
4 3 –5 cos θ q
p q 1
⇒ = = 0.8 (... sin θ = p, sec θ = or cos θ = q)
4 3 q
⇒ p = 3.2, q = 2.4
10. (B) In right triangle ADC,
Therefore, 3p + q = 12.
AC = 32 + 42 = 5
3p 6 50
4. (A) = ≠ CD 4
18 24 75 sin A = = .
AC 5
6 18 CD 4
⇒ p= cos C = =
3 24 AC 5
⇒ p = 3. Now,
4 4 16
5. (B) Consider 3A = 90° sin A cos C = × = .
5 5 25
∴ A = 30°
So, sin 30° – cos 2 × 30° SECTION-B
= sin 30° – cos 60° 11. True, because product of an even
1 1 number and an odd number is an even
= – = 0. number.
2 2

P R A C T I C E P A P E R S 125
12. Let p(x) = ax2 + bx + c ⇒ ∆ABC ~ ∆DFE
So, y = ax2 + bx + c should be satisfied by ⇒ ∠B = ∠F
(–1, 0), (0, –3) and (4, 0) But ∠B = 60°, so, ∠F = 60°.
Therefore, 0 = a – b + c; – 3 = c; 16. True,
0 = 16a + 4b + c
3 9
⇒ c = – 3, a = ,b=– .
4 4
3 2 9
Hence, p(x) =x – x –3
4 4
3 2
⇒ p(x) = (x – 3x – 4)
4
This is the required expression.
AC
sin B =
13. For infinite number of solutions, AB
a1 b c
= 1= 1 PR
a2 b2 c2 sin Q =
PQ
2a a + b 18 sin B = sin Q
i.e., = = ,
2 5 9
AC PR
i.e., a = 2 and a + b = 10 ⇒ =
AB PQ
i.e., a = 2, b = 8.
AC AB
14. In ∆ADP, and ∆EBQ, ⇒ =
PR PQ
AD = EB (Given)
∠ADP = ∠EBQ, (Corresponding angles AC 2 AB2 AB2 – AC2
⇒ = =
as BQ || DP and AB is transversal) PR 2 PQ 2 PQ 2 – PR 2
∠DAP = ∠BEQ (Corresponding angles)
AC 2 AB 2 BC 2
Therefore, ∆ADP ≅ ∆EBQ (ASA criterion) ⇒ 2
= 2
=
PR PQ QR 2
∴ DP = BQ.
AC AB BC
⇒ = =
PR PQ QR
∴ ∆ACB ~ ∆PRQ
⇒ ∠B = ∠Q.
OR
p2 – q2 = (a cot θ + b cosec θ)2
– (b cot θ + a cosec θ)2
Also, DP || BQ as DP || BC.
= a2 cot2 θ + 2ab cot θ cosec θ + b2 cosec2 θ
So, DPQB is a parallelogram.
– b2 cot2 θ – 2ab cot θ cosec θ – a2 cosec2 θ
Therefore, PQ || DB. Hence,
= – a2 (cosec2 θ – cot2 θ)
PQ || AB.
+ b2 (cosec2 θ – cot2 θ)
AB BC CA 1 = – a2 (1) + b2 (1)
15. = = =
DF EF DE 2 = b 2 – a2 .

126 M A T H E M A T I C S – X
17. xi 15 17 19 20 + p 23 ∴ a (a + 1) = 2q × 2q + 1
2q (2q +1) = 2r, where r = q (2q + 1)
fi 2 3 4 5p 6 Σfi =15+5p
So, a (a + 1) is divisible by 2.
fixi 30 51 76 100p+5p2 138 Σfixi = 295 Case II: When a = 2q + 1,
+100p+ 5p2
a + 1 = 2q + 2 = 2 (q + 1)
∑ fi xi ∴ a (a + 1) = 2 (2q + 1) (q + 1)
Mean =
∑ fi where r = (2q + 1) (q + 1)
So, a (a + 1) is divisible by 2.
295 + 100p + 5p2
⇒ 20 = Hence, multiplication of any two conse-
15 + 5p
cutive positive integers is divisible by 2.
⇒ 300 + 100p = 295 + 100p + 5p2
20. Ram, Ravi and Nitin will meet next after
⇒ 5p2 = 5 the time given by the LCM of 5 days, 24
⇒ p = ± 1. days and 9 days.
But frequency cannot take negative value. Now, we find out the LCM of 5, 24 and 9
So, p ≠ –1. 5 = 5; 24 = 23 × 3; 9 = 3 × 3
Hence, p = 1. ∴ LCM = 23 × 3 × 3 × 5 = 360
18. Table for cumulative frequency: They met last on Sunday. So, it will be
Sunday after 7n days, where n is a natural
Cumulative number.
Class Frequency frequency
So, it will be Sunday after 357 days.
0-10 5 5 Therefore, it will be Wednesday after 360
10-30 15 20 days. Hence, they will meet on next
30-60 30 50 Wednesday.
60-80 8 58 OR
80-90 2 60 See worksheet-2, Sol. 9.

N = 60 21. Let zeroes are α, β, γ


Let αβ = 8 ... (i)
Modal class is 30-60. Also we know α + β + γ = 9 ... (ii)
30 + 60 αβ + αγ + βγ = 26
Class mark = = 45
2 αβγ = 24
N ∴ 8 (γ) = 24
Since = 30 so, median class is 30-60.
2 ⇒ γ=3
⇒ Class mark = 45 (ii) ⇒ α+β=6 ... (iii)
Required sum = 45 + 45 = 90. 8
(i) ⇒ β=
α
SECTION-C ∴ Use if in (iii) we get
19. Let a be any positive integer. We know 8
α+ =6
that any positive integer is either of the α
form 2q or 2q + 1 for some integers q. ⇒ α2 – 6α + 8 = 0
∴ a = 2q or 2q + 1 (α – 4) (α – 2) = 0
Case I: When a = 2q, a + 1 = 2q + 1. ⇒ α=4 ⇒ α = 2.

P R A C T I C E P A P E R S 127
If α = 4 ⇒ β = 2 and if α = 2
AB 2
⇒ β=4 =
AD 1
∴ Zeroes are 2, 4 and 3.
Let AB = 2 x and AD = x,
1 1 then from the figure,
22. Put = u and = v in the given
x+y x–y
system of equation. we get
BD = 2x – x = ( )
2 –1 x

10u + 2v = 4
Now,
BD
=
( 2 –1 x )
15u – 5v =– 2 AB 2x
i.e., 5u + v = 2 ... (i)
2 –1 2 2– 2
15u – 5v =– 2 ... (ii) = × = .
2 2 2
Multiply equation (i) by 5 and add the
result to (ii) 24. In right-angled ∆ABC,
1 AB2 + BC2 = AC2 ...(i)
40u = 8 or u = (By Pythagoras Theorem)
5
1 In ∆ABN,
Substitute u= in equation (i)
5 AN2 = AB2 + BN2
v= 1 2
 BC 
1 = AB2 +  
u= and v = 1 give x + y = 5 2 
5
and x – y= 1 (... N is the mid-point of BC)
On solving, we get 1
= AB2 + BC2
x = 3 and y = 2 4
A
Hence, x = 3, y = 2 is the required solution.
23. In ∆ABC, DE || BC
∵∆ABC ~ ∆ADE M

ar ( ∆ABC ) AB2
∴ = ...(i)
ar ( ∆ADE ) AD2
B N C
Again, DE || BC
⇒ 4 AN2 = 4AB2 + BC2 ...(ii)
A
Similarly, in ∆CBM,
4 CM2 = AB2 + 4 BC2 ...(iii)
D E Adding equations (ii) and (iii), we get
4AN2 + 4 CM2
B C
= (4AB2 + AB2) + (BC2 + 4BC2)
∵ ar (∆ADE) = ar ( BCED) = 5 AB2 + 5BC2
∴ ar (∆ABC) = 2 ar (∆ADE)
⇒ 4(AN2 + CM2)
ar ( ∆ABC ) = 5 (AB2 + BC2)
⇒ =2 ... (ii)
ar ( ∆ADE )
= 5 AC2. [From (i)]
From equations (i) and (ii), we get Hence proved.

128 M A T H E M A T I C S – X
tan θ + sec θ – 1 2
25. LHS = 2 1
tan θ – sec θ + 1 = 3 = =
4 4 2

=
(
tan θ + sec θ – sec 2 θ – tan 2 θ ) 3
Thus, LHS = RHS. Hence proved.
tan θ – sec θ + 1
27. First, we prepare the cumulative frequency
(tan θ + sec θ) – (sec θ + tan θ) (sec θ – tan θ) table as given below:
=
tan θ – sec θ + 1
Class Frequency Cumulative
interval ( f) frequency (cf )
( tan θ + sec θ ) (1 – sec θ + tan θ )
= 85-100 11 11
tan θ – sec θ + 1
100-115 9 20
sin θ 1
= tan θ + sec θ = + 115-130 8 28
cos θ cos θ
130-145 5 33
1 + sin θ
= N = 33
cos θ
= RHS. Hence proved. N
∵ N = 33 ∴ = 16.5
2
 cos 58°   cos 18° cosec 52°  Cumulative frequency just greater than
26. 2  – 3  
sin 32° tan 15 ° tan 60° tan 75 ° 16.5 is 20. So, median class is 100-115.
   
∴ cf = 11, f = 9, l = 100, h = 15
 cos ( 90° – 32° ) 
= 2   N – cf 
 sin 32°   
Now, median = l +  2 ×h
 cos ( 90° – 52°) cosec 52°  f
– 3   
  
 tan ( 90° – 75°) × 3 × tan 75°   16.5 – 11
= 100 +   × 15 = 100 + 9.17
 1   9
 sin 32°   sin 52° × sin 52°  = 109.17
= 2  – 2 
Hence, the median speed is 109.17 km/hr.
 sin 32°   cot 75° × 3 × 1 
 cot 75°  28. Since, the maximum frequency is 41, so

the modal class is 10000-15000.
1
=2×1– 3 × = 2 – 1 = 1. ∴ l = 10000, f1 = 41, f0 = 26, f2 = 16, h = 5000
3
OR  f1 – f0 
Now, mode = l +  ×h
1  2 f1 – f0 – f2 
LHS = cos 2θ = cos (2 × 30°) = cos 60° =
2  41 – 26 
= 10000 +  × 5000
RHS =
1 – tan 2 θ
=
1 – tan 2 30°  82 – 26 – 16 
2
1 + tan θ 1 + tan 2 30°
75000
2 = 10000 +
 1  1 40
1–  1–
=  3 = 3 = 10000 + 1875 = 11875.
 1 
2 1
1+ 1+ Thus, the monthly modal income is
 3
 3 Rs. 11875.

P R A C T I C E P A P E R S 129
OR We know that
Let a = 50; h = 20 Time × Speed = Distance
xi – a When the cars run in the same direction.
C.I. fi xi ui = fi ui
h x km/hr y km/hr

0-20 4 10 –2 –8
20-40 10 30 –1 – 10 A 80 km B
40-60 28 50 0 0 Difference of the distances covered by
60-80 36 70 1 36 the two cars = 80 km
80-100 50 90 2 100 8x – 8y = 80
128 Σfi ui = 118 i.e., x – y = 10 ... (i)
When the cars run in the opposite
 ∑ fi ui   118  directions:
x = a + h  ∑ f  = 50 + 20 ×  
128  y km/hr
i x km/hr

295
= 50 + = 18.4 ≅ 68.4. A 80 km B
16
1 × x + 1 × y= 80 or x + y = 80 ... (ii)
SECTION-D To solve equations (i) and (ii), adding
29. Given that p(x) = a (x2 + 1) – x (a2 + 1) them and subtracting them respectively,
i.e., p(x) = ax2 – (a2 + 1)x + a we get 2x = 90 and 2y = 70
To find zeroes of p (x), put p (x) = 0. i.e., x = 45 and y = 35
i.e., ax2 + a – a2x – x = 0 Hence, the speed of the cars are 45 km/
hr and 35 km/hr.
i.e., (ax2 – a2x) – (x – a) = 0
31. See worksheet-34, Sol. 9 (1st part).
i.e., ax (x – a) – 1 (x – a) = 0
OR
i.e., (x – a) (ax – 1) = 0
See Assessment sheet-7, Sol. 8.
1 32. We have
i.e., x = a,
a q sin θ = p and p cos θ = q
1 p q
Thus, a and are the zeroes of p (x).
a ⇒ sin θ = and cos θ =
q p

Sum of zeroes = a + =
2
1 a +1
=–
– a2 + 1 ( )  p
6
 q
6
a a a (i) +   = sin6 θ + cos6 θ
 q   p
Coefficient of x
= –
Coefficient of x2 = (sin2 θ)3 + (cos2 θ)3
1 a = (sin2 θ + cos2 θ)3 – 3 sin2 θ . cos2 θ
Product of zeroes = a × =
a a (sin2 θ + cos2 θ)
Constant term . .
[ . a + b = (a + b)3 – 3ab (a + b)]
3 3
= .
Coefficient of x 2 2
p2 . q .
Hence proved. = (1)3 – 3 . 2 1
q2 p
OR
= 1 – 3 = – 2. Proved.
See worksheet-12, Sol. 10.
6
30. Let the speeds of the cars be x km/hr p q6
(ii) 6
+ = –2
and y km/hr. q p6

130 M A T H E M A T I C S – X
More than type cumulative frequency
p12 + q12
⇒ = – 2 ⇒ p12 + q12 = – 2p6 q6 table:
p6 q6
⇒ p12 + q12 + 2p6 q6 = 0 Marks No. of Marks cf point
students more
⇒ (p6 + q6)2 = 0 ⇒ p6 + q6 = 0. Proved.
than
33. We have to prove or
sec θ – 1 sec θ + 1 equal
+ = 2 cosec θ
sec θ + 1 sec θ – 1 to
LHS = 0-10 7 0 100 (0, 100)
sec θ – 1 sec θ – 1 sec θ + 1 sec θ + 1 10-20 10 10 97 (10, 97)
× + ×
sec θ + 1 sec θ – 1 sec θ – 1 sec θ + 1 20-30 23 20 83 (20, 83)
30-40 51 30 60 (30, 60)
(sec θ – 1)2 ( sec θ + 1)2
= + 40-50 6 40 9 (40, 9)
sec 2 θ – 1 sec2 θ – 1
50-60 3 50 3 (50, 3)
2 2
( sec θ – 1) ( sec θ + 1)
= +
tan θ 2
tan 2 θ Y

sec θ – 1 sec θ + 1 (0, 100) (60, 100)


= + 100
tan θ tan θ (10, 97) (50, 97)
95
sec θ – 1 + sec θ + 1 (40, 91)
= 90 Less than ogive
tan θ
85
2 (20, 83)
80
2 sec θ cos θ 2
= = = 75
tan θ sin θ sin θ
70
cos θ
65
= 2 cosec θ = RHS. Hence proved.
60 (30, 60)
34. To draw the less than type and more
55
than type ogives, we prepare the
cumulative frequency table by less than 50
and more than methods as given below: 45
Less than type cumulative frequency 40 (30, 40)
table:
35
Marks No. of Marks cf point 30
students less
25
than
20
0-10 7 10 7 (10, 7) (20, 17)
15 More than ogive
10-20 10 20 17 (20, 17)
20-30 23 30 40 (30, 40) 10 (40, 9)
(10, 7)
30-40 51 40 91 (40, 91) 5
(50, 3)
40-50 6 50 97 (50, 97) 0
(32, 0)
X
50-60 3 60 100 (60, 100) 10 20 30 40 50 60

P R A C T I C E P A P E R S 131
We plot the points as given in both of the coordinates of this point of
the tables on a graph, taking marks on intersection, we draw a perpendicular
the x-axis and the cumulative frequencies from this point on the x-axis. The abscissa
on the y-axis. On joining these points by of the foot of this perpendicular is the
free hand smooth curve, we obtain the required median. Here the coordinates
less than and more than type ogives as of the foot of the perpendicular are (32,0),
shown in the figure. where 32 is the approximate value.
Median: The abscissa of the point of Hence, the required median is nearly 32
intersection of the two ogives determines marks.
the median of the given data. To obtain ‰‰

132 M A T H E M A T I C S – X

Вам также может понравиться